Sie sind auf Seite 1von 104

Labor Law II Article 219 (f), Labor Code. Definitions.

- (f)
“Employee” includes any person in the employ of
an employer. The term shall not be limited to the

[Quan] employees of a particular employer, unless the


Code so explicitly states. It shall include any
individual whose work has ceased as a result of or
in connection with any current labor dispute or
because of any unfair labor practice if he has not

Right to Self- obtained any other substantially equivalent and


regular employment.

Organization S.S. Ventures International v. S.S.


Ventures Labor Union, July 23, 2008
Concept and Scope SS Ventures Labor Union filed a petition for
certification election in behalf of the rank and file
employees of Ventures which the latter petitioned to
Article 253, Labor Code. Coverage and
Employees' Right To Self-Organization - All persons be cancelled on the ground that the Union
employed in commercial, industrial, and deliberately included the names of 82 employees no
agricultural enterprises and in religious, charitable, longer connected with Ventures in its list of members
medical, or educational institutions (whether who attended the organizational meeting and in the
operating for profit or not) shall have the RIGHT: adoption.ratification of its Constitution and By-Laws.
(1) to self-organization and SC ruled that the right to form, join or assist a union
(2) to form, join, or assist labor
is specifically protected by Article XIII, Section 3 of
organizations of their own choosing for
purposes of collective bargaining. the Constitution, and such right according to Article
III, Section 8 of the Constitution and Article 246 of the
Ambulant, intermittent and itinerant workers, self- Labor Code shall not be abridged. Once registered
employed people, rural workers and those without with the DOLE, a union is considered a legitimate
any definite employers may form labor labor organization endowed with the right and
organizations for their mutual aid and protection privileges granted by law to such organization.

While a certificate of registration confers a union with


Article 257, Labor Code. Non-Abridgment of
legitimacy with the concomitant right to participate in
Right to Self-Organization. - It shall be unlawful for
any person to restrain, coerce, discriminate against or ask for certification election in a bargaining unit,
or unduly interfere with the employees and workers the registration may be canceled or the union may be
in their exercise of the right to self-organization. decertified as the bargaining unit, in which case the
Such right shall include the right to form, join, or union is divested of the status of a legitimate labor
assist labor organizations for the purpose of organization. Among the grounds for cancellation is
collective bargaining through representatives of the commission of any of the acts enumerated in Art.
their own choosing and to engage in lawful
concerted activities for the same purpose for their 239(a) of the Labor Code, such as fraud and
mutual aid and protection, subject to the provisions misrepresentation in connection with the adoption or
of Article 264 of this Code. (As amended by B.P. ratification of the union’s constitution and like
Blg. 70, May 1, 1980). documents. The relevancy of the 82 individual's’
active participation in the Union’s organizational
meeting and the signing ceremonies thereafter
Article 292 (c), Labor Code. Miscellaneous comes in only for purposes of determining whether or
Provisions. - (c) Any employee, whether employed not the Union, even without the 82, would still meet
for a definite period or not, shall, beginning on his
what Art. 234(c) of the Labor Code requires to be
first day of service, be considered as an employee
for purposes of membership in any labor union. (As submitted.
amended by Sec. 33, Rep. Act No. 6715).
NUWHRAIN-Manila Pavilion Hotel
Chapter v. Secretary, July 31, 2009
Article 219 (e), Labor Code. Definitions. - (e)
“Employer” includes any person acting in the A certification election was conducted among the
interest of an employer, directly or indirectly. The rank and file employees of Holiday Inn Manila. 11
term shall not include any labor organization or any votes were initially segregated because they were
of its officers or agents except when acting as cast by dismissed employees, and thereafter, 6 other
employer. votes were segregated because the employees who
1
cast them were already occupying supervisory right. Also inherent in the right to self-organization is
positions. Another five other votes were segregated the right to choose whether to form a union for
on the ground that they were cast by the purposes of collective bargaining or a workers’
probationary employees and that based on the CBA, association for purposes of providing mutual aid and
such employees cannot vote. On appeal to the SOLE, protection. There is no provision in the Labor Code
NUHWRAIN argued that the votes of the probationary that states that employees with definite employers
employees should have been opened considering may form, join or assist unions only.
that a probationary employee’s vote was tallied, and
that HIMPHLU should not be immediately certified as NOTE: Labor unions are labor organizations but not
the bargaining agent as the votes it garnered would all labor organizations are unions. Employer-
be one less than the majority if the 17 segregated employee relationship is not needed to form a
votes would be opened. SOLE ruled that even if 17 worker’s association.
votes were counted, it would not overturn the votes
garnered by HIMPHLU. CA affirmed. Atlas Lithographic Services v.
The inclusion of the probationary employees vote was
Laguesma, January 6, 1992
proper because probationary employees have the KAMPIL-KATIPUNAN filed, on behalf of the supervisor’s
right to vote in the certification election. The votes of union, a petition for certification election for it to be
the 6 other probationary employees should also be the sole and exclusive bargaining agent of the
counted. In a certification election, all rank and file supervisory employees. ALSI opposed on the ground
employees, whether probationary or permanent, are that the former cannot represent the supervisory
entitled to vote which principle is stated in Article employees for CBA purposes because it also
255. Hence, all rank and file have a substantial represents the rank and file employees’ union.
interest in the selection of the bargaining
representative as the Code makes no distinction as to Managerial employees are not eligible to join, assist
their employment status as basis for eligibility in or form any labor organization. Supervisory
supporting the petition for certification election. All employees shall not be eligible for membership in a
the employees need to be eligible is to belong to a labor organization of the rank and file employees but
bargaining unit. may join, assist or form separate labor organizations
of their own. It is construed to mean that supervisors
shall not be given an occasion to bargain together
Samahan ng Manggagawa sa Hanjin with the rank and file against the interests of the
Shipyard v. BLR, October 14, 2015 employer regarding the terms and conditions of work.
SAMAHAN filed an application for registration of the The Adamson v CIR case relied upon by KAMPIL is
name “Samahan ng Mga Manggagawa sa Hanjin inapplicable because in this case, the rank and file
Shipyard” with the DOLE, but Hanjin filed a petition employees are directly under the supervisors
praying for the cancellation of registration of organized by the same federation and said local
SAMAHAN’s association on the ground that its union is actively represented by the national
members did not fall under any of the types of federation.
workers enumerated in the second sentence of Article
243. Hanjin filed a supplemental petition, adding that
the alternative ground that SAMAHAN committed Labor Organizations and
misrepresentation in connection with the list of
members and/or voters who took part in the Registration of Unions
ratification of their constitution and by-laws in its
application for registration. DOLE Regional Director Article 219 (g), Labor Code. LABOR
and CA ruled to cancel. ORGANIZATION - means any union or association of
employees which exists in whole or in part for the
In the case at bench, the Court cannot sanction the purpose of collective bargaining or of dealing with
employers concerning terms and conditions of
opinion of the CA that Samahan should have formed
employment.
a union for purposes of collective bargaining instead
of a worker’s association because the choice
belonged to it. The right to form or join a labor Article 219 (h), Labor Code. LEGITIMATE LABOR
organization necessarily includes the right to refuse ORGANIZATION - means any labor organization
or refrain from exercising the said right. It is self- duly registered with the DOLE, and includes any
evident that just as no one should be denied the branch or local thereof.
exercise of a right granted by law, so also, no one
should be compelled to exercise such a conferred

2
Article 237, Labor Code. Registry of Unions and workers who participated in such meetings;
File of Collective Bargaining Agreements. - The (c) In case the applicant is an
Bureau shall keep a registry of legitimate labor INDEPENDENT UNION - the names of all its
organizations. The Bureau shall also maintain a file members comprising at least 20% of all the
of all collective bargaining agreements and other employees in the bargaining unit where it
related agreements and records of settlement of seeks to operate;
labor disputes and copies of orders and decisions of (d) If the applicant union has been in
voluntary arbitrators. The file shall be open and existence for one or more years - copies of
accessible to interested parties under conditions its annual financial reports; and
prescribed by the Secretary of Labor and (e) 4 copies of the constitution and by-laws
Employment, provided that not specific information of the applicant union, minutes of its
submitted in confidence shall be disclosed unless adoption or ratification, and the list of the
authorized by the Secretary, or when it is at issue members who participated in it.
in any judicial litigation, or when public interest or
national security requires.
Article 241, Labor Code. Chartering and
Within 30 days from the execution of a Collective Creation of a Local Chapter — A duly registered
Bargaining Agreement, the parties shall submit federation or national union may directly create a
copies of the same directly to the Bureau or the LOCAL CHAPTER by issuing a charter certificate
Regional Offices of the Department of Labor and indicating the establishment of the local chapter.
Employment for registration, accompanied with The chapter shall:
verified proofs of its posting in two conspicuous (1) acquire legal personality ONLY for
places in the place of work and ratification by the purposes of filing a petition for certification
majority of all the workers in the bargaining unit. election FROM the date it was issued a
The bureau or Regional Offices shall act upon the charter certificate.
application for registration of such CBA within 5 (2) be entitled to all other rights and
calendar days from receipt thereof. The Regional privileges of a legitimate labor organization
Offices shall furnish the Bureau with a copy of the ONLY upon the submission of the following
CBA within 5 days from its submission. documents IN ADDITION to its charter
certificate:
The Bureau or Regional Office shall assess the (a) The names of the chapter's
employer for every CBA a registration fee of not officers, their addresses, and the
less than one thousand pesos (P1, 000.00) or in principal office of the chapter; and
any other amount as may be deemed appropriate (b) The chapter's constitution and
and necessary by the Secretary of Labor and by-laws: PROVIDED, That
Employment for the effective and efficient where the chapter's constitution
administration of the Voluntary Arbitration and by-laws are the SAME as that
Program. Any amount collected under this provision of the federation or the national
shall accrue to the Special Voluntary Arbitration union, this fact shall be indicated
Fund. accordingly.
The additional supporting requirements
The Bureau shall also maintain a file and shall shall be certified under oath by the
undertake or assists in the publication of all final secretary or treasurer of the chapter AND
decisions, order and wards of the Secretary of attested by its president
Labor and Employment, Regional Directors, and the
Commission. (As amended by Sec. 15, Rep. Act.
No. 6715, March 21, 1989).
Tagaytay Highlands vs. Tagaytay
Highlands Employees Union, January 22,
Article 240, Labor Code. Requirements of
Registration —A federation, national union or
2003
industry or trade union center or an independent The Union filed a petition for certification election
union shall:: before the DOLE Med Arbiter who gave due course to
(1) acquire legal personality and the petition as the Union is a legitimate labor
(2) shall be entitled to the rights and federation and its local chapter was duly reported to
privileges granted by law to legitimate DOLE as one of its affiliate locals/chapters. After a
labor organizations
certificate of registration is issued to a union, its legal
UPON issuance of the certificate of registration personality cannot be subject to collateral attack. It
based on the following requirements: may be questioned only in an independent petition
(a) Fifty pesos (P50.00) registration fee; for cancellation in accordance with Section 5 of Rule
(b) The names of its officers; their V, Book IV of the "Rules to Implement the Labor
addresses; the principal address of the Code. The inclusion in a union of disqualified
labor organization; the minutes of the employees is not among the grounds for cancellation,
organizational meetings; and the list of the
unless such inclusion is due to misrepresentation,
3
false statement or fraud under the circumstances dual unionism is not a ground for cancelling
enumerated in Sections (a) and (c) of Article 239 of registration.
above-quoted Article 239 of the Labor Code.
The discrepancies cannot be taken as an indication
SMCEU-PTGWO vs. SMPPEU-PDMP, that PIGLAS misrepresented the information
contained in the documents submitted. Charges of
September 12, 2007 fraud should be clearly established by evidence and
SMC Employees Union is the incumbent bargaining surrounding circumstances because once it is proved,
agent for the regular monthly paid rank and file the labor union requires none of the rights accorded
employees of the three divisions of SMC for over 20 to registered organizations. The Labor Code and
years. SMC Packaging was registered as a chapter of implementing rules do not require that the number of
the PDMP which issued the former’s charter members appearing on the documents should
certificate. Upon submission of its charter certificate completely dovetail. For as long as the documents
and other documents, SMC Packaging was issued by and signatures are shown to be genuine and regular
BLR a certificate of creation of local or chapter. SMC an the constitution and by-laws are ratified, the union
then filed a petition to cancel SMC Packaging’s is deemed to have complied with registration
registration as it was not a legitimate labor requirements.
organization but a trade union center.

The procedure for registration of a local or chapter of


Eagle Ridge Golf and Country Club v.
a labor organization is provided in CA, March 18, 2010
the Implementing Rules, which enunciates a two-fold Eagle Ridge Employees Union filed a petition for
procedure for the creation of a chapter or a local. The certification election which Eagle Ridge opposed as
first involves the affiliation of an independent union there was a discrepancy in its declaration that it has
with a federation or national union or industry union. 30 members when it only had 26 members on the
The second involves the direct creation of a local or a organizational meeting. The Union contends that the
chapter through the process of chartering. In the case discrepancies are not real as 4 additional members
at bar, the Union was not a legitimate labor did join the union 2 days after the organizational
organization, having been created via chartering by meeting. When the Union applied for registration,
PDMP which is a trade union center. The PDMP failed there were 30 employees so it complied with the 20%
to comply with the registration requirements to membership requirement under Article 234(c). 20%
become a local or chapter. Although PDMP as a trade of 112 rank and file employees in Eagle Ridge would
union center is a legitimate labor organization, it has require a union membership of at least 22
no power to directly create a local or chapter, such as employees, thus when the certification of registration
RESPONDENT. Thus, RESPONDENT cannot be created was granted, the Union had complied with said
under the more lenient requirements for chartering, requirement. Furthermore, the admission of new
but must have complied with the more stringent rules members is neither prohibited by law nor was it
for creation and registration of an independent union, concealed in its application for registration.
including the 20% membership requirement.
SMCC-SUPER v. Charter Chemical and
Heritage Hotel Manila vs. Piglas- Coating Corp., March 16, 2011
Heritage, October 30, 2009 The union filed a petition for certification election
HHE filed a petition for certification election which among the regular rank and file employees of Charter
Heritage Hotel opposed on the ground that HHE Chemical. A motion to dismiss was filed by
misrepresented itself to be an independent union respondent on the ground that petitioner is not a
when it was actually a local chapter of NUWHRAIN. legitimate labor organization. The Supreme Court
Another union, PIGLAS, was formed by rank and file ruled that the failure to certify under oath the local
employees and filed a petition for certification chapter certificate issued by the mother federation
election, while HHE filed a petition for cancellation of and the list of the members who attended the
its own union registration. Heritage Hotel opposed organizational meeting is not a ground for the
PIGLAS’ petition on the ground that the new union cancellation of the union’s legal personality. The SC
members were also those that comprised the old said that the charter certificate is prepared and
union so its establishment was a circumvention of the issued by the national union and not the local
earlier injunction issued by CA. DOLE denied Heritage chapter, thus is will not make sense to have the local
Hotel’s petition to cancel registration because the chapter’s officers certify or attest to a document
discrepancies in the number of members in the which they had no hand in the preparation. It valid
application’s supporting documents were not material acquired the status of a legitimate labor organization
and did not constitute misrepresentation, and that upon submission of its charter certificate, the names
4
of its officers, their address, and its principal office, the nature of such fraud and misrepresentation must
and its constitution and by-laws. Regarding the be grave as to vitiate the consent of a majority of
alleged mixture, the SC said that while there is a union members.
prohibition against mingling of supervisory and rank
and file employees in one labor organization, the
Labor Code does not provide for the effects thereof. Eligibility for Memberships;
Yokohama Tire Phils. v. Yokohama Special Groups of Employees
Employees Union, March 10, 2010 Article 255, Labor Code. Ineligibility of
The union filed a petition for certification election. managerial employees to join any labor
YTPI filed for the revocation of the union’s organization; right of supervisory employees. -
registration. It was alleged that the union fraudulently Managerial employees are NOT eligible to join,
included the signature of a certain Pineda in the assist or form any labor organization. Supervisory
organizational documents and that the union employees shall NOT be eligible for membership in
a labor organization of the rank-and-file employees
fraudulently obtained the employees signatures by
BUT may join, assist or form separate labor
making them believe that they were signing a organizations of their own. (As amended by Section
petition for a 125% increase in the minimum wage. 18, Republic Act No. 6715, March 21, 1989)
The SC said that the union did not commit fraud or
misrepresentation for failing to remove Pineda’s
name as the Court gave greater credence to Pineda’s Article 219 (m), Labor Code.
handwritten statement wherein he made references MANAGERIAL EMPLOYEE - is one who is vested with
to at least 2 meetings he had attended during which the powers or prerogatives to lay down and
he had signed the organizational documents. YTPI, execute management policies and/or to hire,
transfer, suspend, lay-off, recall, discharge, assign
being the one which filed the petition, had the burden
or discipline employees.
of proving that the union committed fraud and SUPERVISORY EMPLOYEES - are those who (in the
misrepresentation. interest of the employer) effectively recommend
such managerial actions if the exercise of such
Takata Philippines v. BLR and SALAMAT, authority is NOT merely routinary or clerical in
nature BUT requires the use of independent
June 4, 2014 judgment.
Takata filed a petition for cancellation of certificate of All employees NOT falling within any of the above
union registration of SALAMAT on the ground that it definitions are considered RANK-AND-FILE
EMPLOYEES for purposes of this Book.
was guilty of misrepresentation, false statements and
fraud with respect to the number of those who
participated in the organizational meeting. Only 68
Article 256, Labor Code. Effect of Inclusion as
attendees signed the attendance sheet of SALAMAT’s Members of Employees Outside the Bargaining
organizational meeting which only comprised 17% of Unit. - The inclusion as union members of
the total number of the rank and file employees employees outside the bargaining unit shall not be
whom SALAMAt sought to represent, as such it was a ground for the cancellation of the registration of
alleged that it failed to comply with the 20% the union. Said employees are automatically
minimum membership requirement. deemed removed from the list of membership of
said union.
SALAMAT’s registration should not be cancelled on
the grounds of fraud and misrepresentation. Article
Section 1 (hh), Omnibus Rules, Book V, Rule 1
234 of the Labor Code provides for the requirements (as amended by DO 40). MANAGERIAL
of registration and it does not appear from said EMPLOYEE - refers to an employee who is vested
provision that the attendees in the organizational with powers or prerogatives to lay down and
meeting must comprise 20% of the employees in the execute management policies or to hire, transfer,
bargaining unit. The 20% requirement pertains to the suspend, layoff, recall, discharge, assign or
employees’ membership in the union, and not in the discipline employees.
list of workers who participated in the organizational
meeting. Here, the total number of employees in the
Section 1 (xx), Omnibus Rules, Book V, Rule 1
bargaining unit was 396 and 20% of which was about
(as amended by DO 40). SUPERVISORY
79 -- SALAMAt, in the document it submitted, showed EMPLOYEE - refers to an employee who (in the
the names of 119 employees as union members thus interest of the employer) effectively recommends
complying with the requirement. Furthermore, for managerial actions, and the exercise of such
fraud and misrepresentation to be grounds for authority is NOT merely routinary or clerical BUT
cancellation of union registration under Article 239,
5
requires the use of independent judgment. Pepsi Cola Products v. Secretary of
Labor, August 10, 1999
The Union filed a petition for certification election to
Section 1 (nn), Omnibus Rules, Book V, Rule 1 be the exclusive bargaining agent of the supervisors
(as amended by DO 40). RANK-AND-FILE
of PEPSI which was granted. PEPSI filed a petition to
EMPLOYEE - refers to an employee whose functions
are neither managerial nor supervisory in nature. set aside and revoke charter of the Union as a
supervisor’s union cannot affiliate with a federation
whose members include rank and file employees of
Cathay Pacific Steel Corp. v. CA, August the same company, and that confidential employees
30, 2006 cannot join the labor union of the rank and file.
CAPASCO hired Tamondong as Assistant to the Although confidential employees are not explicitly
Personnel Manager of its Cainta plant and was later disqualified or mentioned in the Labor Code from
promoted as Personnel Superintendent. The joining or forming any labor organization, the SC
supervisory personnel of CAPASCO later formed a applied the doctrine of necessary implication that
union known as CUSE. Tamondong joined and later “while Article 245 singles out managerial employees
elected as an officer. CAPASCO sent a memo to him as ineligible to join, assist, or form andy labor
requiring him to discontinue his activities in the organization, under the doctrine of necessary
union. However, Tamondong did not listen and thus implication, confidential employees are similarly
was later terminated. CAPASCO contended that he is disqualified”. Route Managers, Chief Checkers and
considered as a managerial employee. The SC Warehouse Operations Managers are supervisors,
however ruled that he is not a managerial employee. while Credit and Collection and Accounting Managers
Tamondong was not at liberty to lay down and are highly confidential employees. The mere fact that
execute major business and operational policies for an employee is designated “manager” does not make
and in behalf of CAPASCO. He does not have the him one.
power to hire, transfer, terminate, or discipline erring
employees of the company. According to the Court, San Miguel Corp. Supervisors and
Tamondong is a supervisory employee. Supervisory
Exempt Union v. Laguesma, August 15,
employees are those who effectively recommend
such managerial actions, if the exercise of such 1997
authority is not merely routinary or clerical in nature The union filed a petition for certification election
but requires the use of independent judgment. among supervisory and exempt employees of SMC
Magnolia Poultry plants of Cabuyao, San Fernando,
and Otis. The Med-Arbiter granted this and ordered
De La Salle University Medical Center v.
the conduct of certification among the supervisory
Laguesma, August 12, 1999 and exempt employees of the 3 plants as one
FFW granted a certification to FFW-DLSUMCCMSUC bargaining unit. SMC Magnolia Poultry said it was an
recognizing it as its local chapter and also filed on error for the Med-Arbiter to group the 3 plants. In
behalf of the latter, a petition for certification addition, there were supervisory levels 3 and above
election. DLSU opposed as it alleged that some of employees whose positions are confidential who were
the employees who signed the petition were included.
managerial employees and that FFW-DLSUMCCMSUC
was comprised of both rank and file and supervisory The SC said that the supervisory levels 3 and above
employees. The SC ruled that unions formed employees and the exempt employees are not
independently by supervisory and rank and file confidential employees. Confidential employees are
employees of a company may validly affilitate with those who (1) assist or act in a confidential capacity,
the same federation. The exception is when, as in (2) to persons who formulate, determine, and
Atlas Lithographic v. Laguesma, the rank and file effectuate, management policies in the field of labor
employees are under the authority of the supervising relations (‘‘The confidential relationship must exist
employee and the national federation is actively between the employees and his supervisor’’). An
involved in the union activities of the company. In this important element of the confidential employee rule
case, although FFW showed active involvement in the is the employees need to use labor relations
union activities of the company through the filing of information. Thus, in determining the confidentiality
the petition for certification election, DLSU failed to of certain employees, a key question frequently
show that the rank and file employees are under the considered is the employees necessary access to
authority of the supervisory employees. confidential labor relations information. Access to
confidential information does not in itself render an
employee a confidential employee. The information

6
must relate to the employers labor relations policies. and APOSTEU, both federations have a common set
An employee must have access to confidential labor of officers and actively participate in each other’s
relations information with respect to the employer, affairs giving rise to possible conflict of interest. As
the union, or the association, to be regarded as a long as they are affiliated with each other, the unions
confidential employee. do not meet the criteria to attain the status of
legitimate labor organizations and cannot separately
SCBEU-NUBE v. Standard Chartered petition for certification election.

Bank, April 22, 2008 NOTE: Does the federation always actively
The bank and the union negotiated for a new CBA but participate in an enterprise level? NO. The prohibition
due to a deadlock, the union filed a notice of strike. applies to supervisor’s union applying for
The CBA provisions in dispute are the exclusion of membership in a national federation the members of
certain employees from the appropriate bargaining which include local unions of rank and file employees.
unit and the adjustment of remuneration for For said prohibition to apply, it is not enough that the
employees serving in an acting capacity. The union supervisory union and rank and file union are
sought to exclude all managers who are vested with affiliated with a single federation. The supervisors
the right to hire and employees and confidential must have direct authority over the rank and file
employees, Chief Cashiers, Assistant Cashiers, employees.
personnel of Telefax department and HR staff. The SC
held that the disqualification of managerial and A chartered local union acquires legal personality
confidential employees from joining a bargaining unit once issued a charter certificate by a duly registered
for rank and file is already well-entrenched in national union or federation. A local union does not
jurisprudence. As regard the qualification of bank owe its existence to the federation it is affiliated with
cashiers as confidential employees, NATU v Torres as this only gives rise to a contract of agency
already declared that they are confidential whereby the federation is the agent and the local
employees. Golden Farms v Ferrer-Calleja also stated union is the principal.
that radio and telegraph operators and accounting
personnel are likewise confidential employees and in
Philipps Industrial Development v NLRC, personnel
Tunay na Pagkakaisa ng Manggagawa sa
staff in which HR staff may be qualified are Asia Brewery v. Asia Brewery, August 3,
confidential employees. 2010
Asia Brewery entered into a CBA with BLMA, the
Coastal Subic Bay Terminal v. DOLE, bargaining representative of the rank and file
November 20, 2006 employees of ABI. A dispute arose when ABI
The rank and file union and the supervisory union management stopped deducting union dues from the
filed separate petitions for certification election 81 employees believing that their membership in
insisting that they are legitimate because ALU and BLMA violated the CBA as 18 of them were QA
APSOTEU have issued them certificates of charter Sampling Inspectors, 20 are checkers and the rest are
respectively. However, Coastal Subic Bay opposed secretaries. The SC ruled that the 81 employees may
alleging that the said unions are not legitimate labor not be excluded from the bargaining unit as ABI failed
organizations. The Med-Arbiter ruled that ALU and to indicate who among the secretaries and clerks had
APOSTEU are one and the same federation having a access to confidential data relating to management
common set of officers thus the two unions were in policies and thus are not confidential employees.
fact affiliated with only one federation. DOLE With respect to the Sampling Inspectors and Gauge
Secretary reversed as both unions have separate Machine Technician, they also perform routine and
legal personalities. Coasta Bay contends further that mechanical tasks preparatory to the delivery of
APOSTEU is not properly registered as it secured finished products. While it may be argued that the
registration from DOLE Regional Director (not BLR) quality control extends to the post production phase,
thus the supervisory union cannot attain the status of no evidence was presented to prove that these daily
a legitimate labor organization. paid checkers actually form part of the Quality
Control Staff who are exposed to confidential
APOSTEU did not improperly secure registration from information about the products. Hence, the 20
DOLE Regional Director as DO 40-03 provides that checkers may not be considered confidential
“applications for registration of labor organizations employees under Quality Control Staff.
shall be filed either with the Regional Office or with
the BLR”. Being legitimate, they have the authority to
issue a local charter certificate to the supervisory
union. As to the issue of commingling between ALU
7
San Miguel Foods v. SMC Supervisors Manila Electric Company v. Secretary of
and Exempt Union, August 1, 2011 Labor and Employment, May 20, 1991
This case is in relation to SMC Supervisors and The union in MERALCO filed a PCE, seeking to
Exempt Union v Laguesma wherein the court held represent the employees. MERALCO opposed saying
that supervisory employees 3 and 4 and exempt that security guards personnel are prohibited from
employees are not to be considered confidential joining or assisting rank-and-file empees. The Court
employees because the same do not not pertain to ruled that the Implementing Rules of the Labor Code
labor relations and negotiation and settlement of which sought to disqualify security guards from those
grievances so they are allowed to form a bargaining joining labor organizations is null and void, for being
unit. SMC filed an objection to the petition for not germane to the object and purposes of the Labor
certification election because certain employees, Code. Under RA 6715, security guards are now
such as Payroll Master, HR Assistant and Personnel allowed to join labor organizations of the rank-and-file
Assistant, are confidential employees. employees or supervisory employees depending on
their rank.
Confidential employees are those entrusted with
confidential on delicate or with the custody, handling Benguet Electric Cooperative v. Ferrer-
or care and protection of the employer’s property.
The criteria to be a confidential employee (1. Assist
Calleja, Dec. 29, 1989
or act in a confidential capacity; 2. To persons who BWLU-ADLO filed a petition for certification election
formulate, determine and effectuate management as the exclusive bargaining unit of the rank and file
policies in the field of labor relations) must be met to employees of BENECO, which was opposed by BELU
be considered one. Accounting personnel is a as it contended that it was certified as the exclusive
confidential employee but such does not apply to a bargaining unit of the rank and file employees of
Payroll master since the nature of his work does not BENECO. BENECO filed a MTD claiming that the
pertain to company rules and regulations. employees sought to be represented by BWLU-ADLO
Furthermore, the positions of HR Assistant and are not eligible to form, join or assist labor
Personnel Assistant belong to the category of organizations because they are members and joint
confidential employees and thus excluded from the owners of the cooperative. The SC held that the right
bargaining unit. to collective bargaining is not available to an
employee of a cooperative who at the same time is a
member and co-owner. With respect to employees
Heritage Hotel v. Secretary, July 23, who are neither members or owners of the
2014 cooperative, they are entitled to the rights of self-
NUWHRAIN filed a petition for certification election organization and collective bargaining. The fact that
seeking to represent the supervisory employees of the member employees of BENECO do not participate
Heritage Hotel but the latter filed a petition for in the actual management of the cooperative does
cancellation of certification election on the ground not make them eligible to form, assist, or join a labor
that membership of NUWHRAIN consisted of organization for the purpose of collective bargaining.
managerial, confidential and rank and file employees It is the fact of ownership of the cooperative and not
so they had no legal right to file a petition for involvement in the management which disqualifies a
certification election pursuant to Toyota Motors v member from joining any labor organization in the
Toyota Motor Union. Mixed membership is not a cooperative. A cooperative’s owners and members
ground for cancelling the union registration, except are the ones who run and operate the business while
when such was done through misrepresentation, false the others are its employees.
representation or fraud under the circumstances laid
out in Article 239. Among the amendments of DO 9 Kapatiran sa Meat and Canning Division
was the removal of the requirement of indicating in
the petition for certification election that there was no
v. Ferrer-Calleja, June 20, 1988
co-mingling of rank and file and supervisory TUPAS was the sole and exclusive bargaining
employees in the membership of the labor union representative of the workers in MCD-URC. It filed an
which was prevailing during the pronouncement of amended notice of strike as a means of pressuring
Tagaytay Highlands (mixed membership has no the company to extend or renegotiate a CBA.
bearing on legitimacy of registered labor organization Thereafter, NEW ULO, composed or workers
unless the same falls under Article 239). Presently, belonging to INC, registered as a labor union, filed a
mixed membership does not result in the illegitimacy petition for certification election at the BLR claiming
of the registered union unless the same is due to that it has the majority of the daily wage rank and file
Article 239. employees. TUPAS moved to dismiss the petition for
being defective in form and that the members of the
8
NEW ULO were mostly members of the Iglesia ni to the prejudice of innocent school children nor does
Kristo sect which three (3) years previous refused to it legalize an illegal work stoppage.
affiliate with any labor union. It also accused the
company of using the NEW ULO to defeat TUPAS’ International Catholic Migration
bargaining rights. The right of the members of a
religious sect not to join a labor union for being
Commission v. Ferrer-Calleja, Sept. 28,
contrary to their religious beliefs, does not bar the 1990
members of that sect from forming their own union. The unions of ICMC and IRRI (both international
The fact that TUPAS was able to negotiate a new CBA organizations/special agencies recognized by PH)
with ROBINA within the 60-day freedom period of the filed separate petitions for certificate election. ICMC
existing CBA, does not foreclose the right of the rival and IRRI contend that the unions cannot file petitions
union, NEW ULO, to challenge TUPAS’ claim to for certification election because ICMC and IRRI are
majority status, by filing a timely petition for international organizations which have diplomatic
certification election on October 13, 1987 before immunities. The SC ruled in ICMC and IRRI’s favor.
TUPAS’ old CBA expired on November 15, 1987 and There are laws which provide that the said
before it signed a new CBA with the company on international organizations enjoy diplomatic
December 3, 1987. This is because a certification immunities from legal processes in the PH. The grant
election is the best forum in ascertaining the majority of immunity from local jurisdiction to ICMC and IRRI is
status of the contending unions wherein the workers clearly necessitated by their international character
themselves can freely choose their bargaining and respective purposes. The objective is to avoid the
representative thru secret ballot. danger of partiality and interference by the host
country in their internal workings. The exercise of
Acosta v. CA, June 28, 2000 jurisdiction by the DOLE in these instances would
Petitioners are teachers from different public schools defeat the very purpose of immunity, which is to
in Metro Manila who, on various dates in September shield the affairs of international organizations, in
and October 1990, participated in mass actions by accordance with international practice, from political
public school teachers at the Liwasang Bonifacio for pressure or control by the host country to the
the purpose of petitioning the government for redress prejudice of member States of the organization, and
of their grievances. They refused to comply with the to ensure the unhampered performance of their
“return to work order” issued by the Secretary of functions. However, their immunity from local
DECS so they were administratively charged with jurisdiction does not deprive labor of its basic rights,
misconduct. Furthermore, they contend that their as guaranteed by the Constitution. A UN Convention
participation in the mass actions was an exercise of requires such international organizations to make
their constitutional rights to peaceably assemble and provision for appropriate modes of settlement of
petition the government for redress of grievances and disputes arising out of contracts or other disputes of
that they never went on strike because they never private character to which the specialized agency is a
sought to secure changes or modification of the party. Moreover, pursuant to the MoA between ICMC
terms and conditions of their employment. and the PH Government, whenever there is any
abuse of privilege by ICMC, the Government is free to
The ability to strike is not essential to the right of withdraw the privileges and immunities accorded.
association. In the absence of statute,
public employees do not have the right to engage in Republic Planters Bank General Services
concerted work stoppages for any purpose. The Employees Union National Association
“mass actions” in which the petitioners participated
were, for all intents and purposes, a strike as they
of Trade Unions v. Laguesma, Nov. 21,
constituted a concerted and unauthorized stoppage 1996
of, or absence from work which it was the teachers’ The petitioner union filed a petition for certification
sworn duty to perform, undertaken for essentially election to determine the sole and exclusive
economic reasons. What is being penalized is the bargaining representative of all regular employees
manner in which petitioners exercised their right to outside the bargaining unit of Republic Planters Bank.
assemble which suspended public service and classes The proposed bargaining unit is composed of clerks,
in public schools in Metro Manila, no matter how messengers, janitors, plumbers, telex operators,
temporary, effectively derailing services to the public, mailing and printing personnel, drivers, mechanics
which is one of the reasons why the right to strike is and computer personnel. Allegedly, these are regular
denied of government employees. While they may employees but are considered as contractual
have valid grievances and noble intentions in staging employees by private respondent. This was opposed
the “mass action”, it does not justify their absences to by the private respondent because the union is
comprised of some 30 employees of Superior
9
Maintenance Services Inc (SMSI) who are assigned to obligation to acquire or retain union membership as a
the bank under a Contract of Services and there is condition affecting employment. The purpose of a
already a bargaining unit represented by RBEU. union shop or other union security arrangement is to
guarantee the continued existence of the union
If the union members are not employees, no right to through enforced membership for the benefit of the
organize for purposes of bargaining, nor to be workers.
certified as bargaining agent can be recognized.
Since the persons involved are not employees of the Kinds of Union Security Clause
company, they are not entitled to the constitutional
1. Union shop → all new regular employees are
right to join or form a labor organization for purposes
required to join the union within a certain
of collective bargaining. As to the certification
period for their continued employment
election, no petition for certification election may be
2. Maintenance of Membership Shop →
entertained if filed outside the 60-day period
employees, who are union members or who
immediately before the expiration of the collective
thereafter become members, must maintain
bargaining agreement. The purpose of the prohibition
union membership as a condition for
against the filing of a petition for certification election
continued employment until they are
outside the so-called freedom period is to ensure
promoted or transferred out of the bargaining
industrial peace between the employer and its
unit or the agreement is terminated.
employees during the existence of the CBA. In the
3. Closed Shop → No person may be employed
case at bar, the petition for certification election was
in any or certain agreed departments of the
filed on January 21, 1991. The collective bargaining
enterprise unless he or she is, becomes, and,
agreement between the duly certified bargaining
for the duration of the agreement, remains a
agent, Republic Planters Bank Employees Union, and
member in good standing of a union entirely
private respondent was effective from June 30, 1988
comprised of or of which the employees in
to June 30, 1991. It is crystal clear that the filing of
interest are a part
the petition for certification election was premature.

BPI v. BPI Employees Union, August 10,


Union Security Clause 2010
The union is the exclusive bargaining agent of the
Article 259 (e), Labor Code. Unfair labor rank and file employees. FEBTC rank and file
practices of employers. It shall be unlawful for an
employees, before the merger between BPI and
employer to commit any of the following unfair
labor practice: FEBTC, did not belong to any union. Said Union
xxx invited the FEBTC employees to a meeting, before the
(e) To discriminate in regard to wages, hours of merger, regarding the union shop clause, which in
work and other terms and conditions of Section 2 provides that “new employees falling within
employment in order to encourage or discourage the bargaining unit as defined in Article I of this
membership in any labor organization. Nothing in Agreement, who may hereafter be regularly
this Code or in any other law shall stop the parties
from requiring membership in a recognized employed by the Bank shall, within thirty (30) days
collective bargaining agent as a condition for after they become regular employees, join the Union
employment, except those employees who are as a condition of their continued employment. It is
already members of another union at the time of understood that membership in good standing in the
the signing of the collective bargaining agreement. Union is a condition of their continued employment
Employees of an appropriate bargaining unit who with the Bank”. Pursuant to merger, FEBTC’s
are not members of the recognized collective
employees were hired by BPI. The union sent notices
bargaining agent may be assessed a reasonable
fee equivalent to the dues and other fees paid by of hearing to the former FEBTC employees who
members of the recognized collective bargaining refused to join and requested BPI to implement the
agent, if such non-union members accept the union security clause to terminate the employment of
benefits under the collective bargaining such once they refused to attend the hearing. Med-
agreement: Provided, that the individual Arbiter ruled that the FEBTC employees were not
authorization required under Article 242, paragraph covered by the union security clause since the new
(o) of this Code shall not apply to the non-members
employees who were hired and subsequently
of the recognized collective bargaining agent;
regularized, but were absorbed employees by
operation of law because the former employees of
Union Security, defined FEBTC can be considered assets and liabilities of the
absorbed corporation.
Union security is a generic term for any form of
agreement which imposes upon employees the
10
Non-compliance with a union security clause is a valid respondents. Respondents further did not receive any
ground for an employee’s dismissal, but other communication from GMC except the first
jurisprudence dictates that such a dismissal must still written notice of termination.
be done in accordance with due process. There is
nothing in the said provision which requires that a PICOP Resources, Inc. v. Tañeca, August
new regular employee first undergo a temporary or
probationary status before being deemed as such
9, 2010
under the union shop clause of the CBA. All PRI has a CBA with its collective bargaining agent for
employees in the bargaining unit covered by a Union the rank and file employees, NAMAPRI-SPFL.
Shop Clause in their CBA with management are Contained in their CBA was a union security clause
subject to its terms. However, under law and wherein all employees must be members of the union
jurisprudence, the following kinds of employees are at the time of the signing of CBA as a condition for
exempted from its coverage: their continued employment with the company. Later,
1. Employees who at the time the union shop Atty. Fuentes sent a letter to the management of PRI
agreement takes effect are bona fide demanding the termination of employees who
members of a religious organization which allegedly campaigned for, supported, and signed the
prohibits its members from joining labor Petition for Certification Election of the Federation of
unions on religious grounds; Free Workers Union (FFW) during the effectivity of the
2. Employees already in the service and already CBA. A memorandum was issued addressed to the
members of a union other than the majority concerned employees requiring them to explain in
at the time the union shop agreement took writing why their employment should not be
effect; terminated. A number of employees sent their written
3. Confidential employees who are excluded explanation while others did not. After evaluation,
from the rank and file bargaining unit Atty. Fuentes advised the management of PRI that the
employees excluded from the union shop by union found the explanations to be unsatisfactory and
express terms of the agreement thus ordered for their termination. Respondents then
accused PRI of Unfair Labor Practice claiming their
acts to not be acts of disloyalty.
General Milling Corp. v. Casio, March 10,
2010 The SC said that in terminating the employment of an
IBM Union entered into a CBA with GMC. Under employee by forcing the union security clause, the
Section 3 of the CBA, each member should be of good employer needs to determine and prove that: (1) the
standing as a condition for his employment or union security clause is applicable; (2) the union is
continued employment. A letter was sent to requesting for the enforcement of the union security
respondents charging them with acts inimical to the provision in the CBA; and (3) there is sufficient
interest of the union. Respondents refused to evidence to support the decision of the union to expel
acknowledge this letter and later ordered the the employee from the union. In this case however,
expulsion of the respondents from the union. the third requisite is lacking. The mere signing of the
Pressured by the threatened filing of unfair labor authorization in support of the Petition for
practice, GMC terminated the employment of Certification Election of FFW before the freedom
respondents through a memorandum. Thus, period is not sufficient ground to terminate the
respondents filed a notice of strike and later filed for employment of respondents inasmuch as the petition
illegal dismissal. The SC said the dismissal was itself was actually filed during the freedom period.
procedurally infirmed and thus not valid. Respondents did not resign or withdraw their
membership from the union to which they belong.
In terminating the employment of an employee by Respondents continued to pay their union dues and
enforcing the union security clause, the employer never joined FFW. An authorization letter to file a
needs only to determine and prove that: (1) the union Petition for Certification Election is different from an
security clause is applicable; (2) the union is actual Petition for Certification Election. Also, an
requesting for the enforcement of the union security existing CBA cannot constitute a bar to a filing of a
provision in the CBA; and (3) there is sufficient petition for certification election. When there is a
evidence to support the decision of the union to expel representational issue, the status quo provision in so
the employee from the union. In this case however, far as the need to await the creation of a new
the termination letter made no mention at all of the agreement will not apply. Otherwise, it will create an
evidence supporting the decision to expel absurd situation where the union members will be
respondents from the union. GMC never alleged nor forced to maintain membership by virtue of the union
attempted to prove that the company actually looked security clause and support another union when filing
into the evidence of the of IBM-Local 31 for expelling a petition for certification election. If we apply it,

11
there will always be an issue of disloyalty whenever correctly observed that the recognition of the tenets
the employees exercise their right to self- of the sect should not infringe on the basic right of
organization. self-organization granted by the constitution to
workers, regardless of religious affiliation.
Victoriano v. Elizalde Rope Workers
Union, 59 SCRA 54
Victoriano, a member of INC, has been employed in
the Elizalde Rope Company and a member of the
Union which had a collective bargaining agreement
containing a closed shop provision which provides
that “membership in the union shall be required as a
condition of employment for all permanent
employees’ workers covered by this Agreement”. RA
3350 was enacted introducing an amendment to RA
875 which states “but such agreement shall not cover
members of any religious sects which prohibits
affiliation of their members in any such labor
organization”. Being a member of a religious sect that
prohibits the affiliation of its members with any labor
organization, Victoriano resigned. The union wrote a
letter to the company asking it to separate Victoriano
from the service in view of the fact that he was
resigning from the union as a member.

By virtue, therefore, of a closed shop agreement,


before the enactment of Republic Act No. 3350, if any
person, regardless of his religious beliefs, wishes to
be employed or to keep his employment, he must
become a member of the collective bargaining union.
Hence, the right of said employee not to join the
labor union is curtailed and withdrawn. To that all-
embracing coverage of the closed shop arrangement,
Republic Act No. 3350 introduced an exception, when
it added to Section 4 (a) (4) of the Industrial Peace
Act the following proviso: “but such agreement shall
not cover members of any religious sects which
prohibit affiliation of their members in any such labor
organization”. Republic Act No. 3350 merely excludes
ipso jure from the application and coverage of the
closed shop agreement the employees belonging to
any religious sects which prohibit affiliation of their
members with any labor organization. What the
exception provides is that members of said religious
sects cannot be compelled or coerced to join labor
unions even when said unions have closed shop
agreements with the employers; that in spite of any
closed shop agreement, members of said religious
sects cannot be refused employment or dismissed
from their jobs on the sole ground that they are not
members of the collective bargaining union.

[SUPRA] Kapatiran sa Meat and Canning


Division v. Ferrer-Calleja, 162 SCRA 367
The right of members of a religious sect not to join a
labor union for being contrary to their religious
beliefs, does not bar the members of that sect from
forming their own union. The public respondent
12
Conditions of Membership and any disbursement of its money or funds
UNLESS he is duly authorized pursuant to
Rights of Members its constitution and by-laws;

H. Every payment of fees, dues, or other


Article 250, Labor Code. Right and Conditions of contributions by a member shall be
Membership in a Labor Organization - The evidenced by a receipt signed by the
following are the rights and conditions of officer or agent making the collection and
membership in a labor organization: entered into the record of the organization
A. No arbitrary or excessive initiation fees to be kept and maintained for the purpose;
shall be required of the members of a
legitimate labor organization NOR shall I. The funds of the organization shall NOT be
arbitrary, excessive, or oppressive fine and applied for any purpose or object OTHER
forfeiture be imposed; THAN those expressly provided by its
constitution and by-laws or those expressly
B. The members shall be ENTITLED to full and authorized by written resolution adopted by
detailed reports from their officers and the majority of the members at a general
representatives of all financial transactions meeting duly called for the purpose;
(as provided for in the constitution and by-
laws of the organization); J. Every income or revenue of the
organization shall be evidenced by a record
C. The members shall DIRECTLY ELECT their showing its source, and every expenditure
officers (including those of the national of its funds shall be evidenced by a receipt
union or federation, to which they or their from the person to whom the payment is
union is affiliated, by secret ballot at made (which shall state the date, place and
intervals of 5 years). No qualification purpose of such payment) Such record or
requirements for candidacy to any position receipt shall form part of the financial
shall be imposed other than membership in records of the organization.
good standing in subject labor organization.
The secretary or any other responsible Any action involving the funds of the
union officer shall furnish the SOLE with a organization shall prescribe AFTER 3 years
list of the newly-elected officers, together from the date of submission of the annual
with the appointive officers or agents who financial report to the DOLE or from the
are entrusted with the handling of funds, date the same should have been submitted
within 30 calendar days after the election as required by law, whichever comes
of officers or from the occurrence of any earlier: PROVIDED, That this provision shall
change in the list of officers of the labor apply only to a legitimate labor
organization; (As amended by Section 16, organization which has submitted the
Republic Act No. 6715, March 21, 1989) financial report requirements under this
Code: PROVIDED FURTHER that failure of
D. The members shall determine by secret any labor organization to comply with the
ballot (after due deliberation) any question periodic financial reports required by law
of major policy affecting the entire and such rules and regulations
membership of the organization, UNLESS promulgated thereunder 6 months after the
the nature of the organization or force effectivity of this Act shall automatically
majeure renders such secret ballot result in the cancellation of union
impractical, In which case: the board of registration of such labor organization; (As
directors of the organization may make the amended by Section 16, Republic Act No.
decision in behalf of the general 6715, March 21, 1989)
membership;
K. The officers of any labor organization shall
E. NO labor organization shall knowingly NOT be paid any compensation other than
admit as members or continue in the salaries and expenses due to their
membership any individual who belongs to positions as specifically provided for in its
a subversive organization or who is constitution and by-laws, or in a written
engaged directly or indirectly in any resolution duly authorized by a majority of
subversive activity; all the members at a general membership
meeting duly called for the purpose. The
F. NO person (who has been convicted of a minutes of the meeting and the list of
crime involving moral turpitude) shall be participants and ballots cast shall be
eligible for election as a union officer or for subject to inspection by the SOLE or his
appointment to any position in the union; duly authorized representatives. Any
irregularities in the approval of the
G. NO officer, agent or member of a labor resolutions shall be a ground for
organization shall collect any fees, dues, or impeachment or expulsion from the
other contributions in its behalf or make
13
organization; members on the provisions of its
constitution and by-laws, collective
L. The treasurer of any labor organization bargaining agreement, the prevailing labor
AND every officer thereof who is relations system, and all their rights and
responsible for the account of such obligations under existing labor laws.
organization or for the collection,
management, disbursement, custody or For this purpose, the registered labor organizations
control of the funds, moneys and other may assess REASONABLE DUES to finance labor
properties of the organization, shall render relations seminars and other labor education
to the organization and to its members a activities.
true and correct account of: (1) all moneys
received and paid by him since he assumed Any VIOLATION of the above rights and conditions
office or since the last day on which he of membership shall be a ground for cancellation of
rendered such account, (2) and of all union registration or expulsion of officers from
bonds, securities and other properties of office (whichever is appropriate). At least 30% of
the organization entrusted to his custody or the members of a union or any member or
under his control. The rendering of such members specially concerned may report such
account shall be made: violation to the Bureau. The Bureau shall have the
1. At least ONCE a year WITHIN 30 power to hear and decide any reported violation to
days after the close of its fiscal mete the appropriate penalty.
year;
2. At such other times as may be Criminal and civil liabilities arising from violations
required by a resolution of the of above rights and conditions of membership shall
majority of the members of the continue to be under the jurisdiction of ORDINARY
organization; and COURTS.
3. Upon vacating his office. The
account shall be duly audited and
verified by affidavit and a copy Article 289, Labor Code. Visitorial Power - The
thereof shall be furnished the Secretary of Labor and Employment or his duly
Secretary of Labor. authorized representative is hereby empowered to:
(1) inquire into the financial activities of
M. The books of accounts and other records of legitimate labor organizations
the financial activities of any labor (2) examine their books of accounts and
organization shall be OPEN to inspection by other records to determine compliance or
any officer or member thereof DURING non-compliance with the law AND
office hours; (3) to prosecute any violations of the law
and the union constitution and by-laws
N. NO special assessment or other
extraordinary fees may be levied upon the UPON the filing of a complaint under oath and duly
members of a labor organization UNLESS supported by the written consent of at least 20% of
authorized by a written resolution of a the total membership of the labor organization
majority of all the members in a general concerned
membership meeting duly called for the
purpose. The secretary of the organization PROVIDED, That such inquiry or examination shall
shall record the minutes of the meeting NOT be conducted during the 60-day freedom
including: (1) the list of all members period NOR within the 30 days immediately
present, (2) the votes cast, (3) the purpose preceding the date of election of union officials. (As
of the special assessment or fees and (4) amended by Section 31, Republic Act No. 6715,
the recipient of such assessment or fees. March 21, 1989)
The record shall be attested to by the
president.

O. Other than for mandatory activities under Article 228(b), Labor Code. Appearances and
the Code, NO special assessments, Fees.
attorney’s fees, negotiation fees or any xxx
other extraordinary fees may be checked b. NO attorney’s fees, negotiation fees or similar
off from any amount due to an employee charges of any kind arising from any collective
WITHOUT an individual written bargaining agreement shall be imposed on any
authorization duly signed by the employee. individual member of the contracting union:
The authorization should specifically state PROVIDED, However, that attorney’s fees may be
the amount, purpose, and beneficiary of charged against union funds in an amount to be
the deduction; and agreed upon by the parties.

P. It shall be the DUTY of any labor Any contract, agreement or arrangement of any
organization and its officers to inform its sort to the contrary shall be NULL and VOID. (As
amended by Presidential Decree No. 1691, May 1,

14
immediate termination of the expelled union officers -
1980)
threatened by this, the company terminated the
employees. Petitioners then filed a complaint for
Article 259(e), Labor Code. Unfair labor unfair labor practice.
practices of employers. - It shall be UNLAWFUL for
an employer to commit any of the following UNFAIR The SC said the company was not justified in
LABOR PRACTICE: dismissing petitioner employees. A union security
xxx clause embodied in the CBA may be validly enforced
(e) To discriminate in regard to wages, hours of and that dismissals pursuant thereto may likewise be
work and other terms and conditions of
valid, however this does not eroe the fundamental
employment, in order to encourage or discourage
membership in any labor organization. requirement of due process. The reason is that the
sanctity and inviolability of contracts cannot override
NOTHING in this Code or in any other law shall stop ones right to due process. The enforcement of union
the parties from requiring membership in a security clauses is authorized by law provided such
recognized collective bargaining agent as a enforcement is not characterized by arbitrariness.
condition for employment, EXCEPT those The SC further held that the act of the local union to
employees who are already members of another
disaffiliate was proper. A local union has the right to
union at the time of the signing of the collective
bargaining agreement. disaffiliate from its mother union and declare its
autonomy. A local union, being a separate and
Employees of an appropriate bargaining unit who voluntary association, is free to serve the interests of
are NOT members of the recognized collective all its members including the freedom to disaffiliate
bargaining agent may be assessed a reasonable or declare its autonomy from the federation to which
fee equivalent to the dues and other fees paid by it belongs when circumstances warrant, in
members of the recognized collective bargaining
accordance with the constitutional guarantee of
agent, IF such non-union members accept the
benefits under the collective bargaining freedom of association. The purpose of affiliation by
agreement: PROVIDED, that the individual a local union with a mother federation is to increase
authorization required under Article 250, paragraph by collective action the bargaining power in respect
(o) of this Code shall not apply to the non-members of the terms and condition of labor. Thus, a local
of the recognized collective bargaining agent; union which was affiliated itself with a federation is
free to sever such affiliation anytime and such
NOTE: Please read Omnibus Rules, Book V, Rule XI, disaffiliation cannot be considered disloyalty. In the
XII, XIII, XVIII, XX, as amended by D.O. 40 absence of specific provisions in the federations
constitution prohibiting disaffiliation or the
declaration of autonomy of a local union, a local may
Local Unions and Federations dissociate with its parent union.

MSMG-UWP v. Ramos, GR 113907, Philippine Skylanders v. NLRC, GR


February 28, 2000 127374, January 31, 2002
MSMG-UWP is an affiliate of private respondent PSEA, a local labor union affiliated with PAFLU, won in
federation ULGWP. A local union election was held the certification election conducted among rank and
wherein the herein petitioners were proclaimed as file employees of PSI. PSEA’s rival union, PSEA-WATU,
winners. The defeated candidates filed for a petition immediately protested the result of the election
for impeachment with the national federation but was before the Secretary of Labor. PSEA sent PAFLU a
dismissed. The local union held a general notice of disaffiliation because of PAFLU’s alleged
membership meeting and several union members deliberate and habitual dereliction of duty toward its
failed to attend thus asking the respondent company members. PSEA then affiliated itself with NCW and
to deduct the union fines from the wages/salaries of changed its name to PSEA-NCW. PSEA-NCW then
those union members who failed to attend the entered into a CBA with petitioner PSI. PAFLU asked
general membership meeting. The imposition of the for an audited financial statement from PSI which was
Php 50 fine became the subject of bitter rejected by the latter as PSEA already disaffiliated
disagreement between the Federation and the local itself from PAFLU. PAFLU then filed a complaint for
union causing the union’s declaration of unfair labor practice.
autonomy.The federation then expel the local union
officers from the federation and advised respondent The SC said that the disaffiliation was valid pending
company about this and demanded their separation the settlement of the election protest. This Court has
from employment. The federation filed a notice of upheld the right of local unions to separate from their
strike to compel the employer to effect the mother federation on the ground that as separate

15
and voluntary associations, local unions do not owe employees. The petition was dismissed by the Med-
their creation and existence to the national federation Arbiter due to the lack of legal personality of
to which they are affiliated, but instead, to the will of petitioner. Thereafter, respondent filed a petition for
their members. There is nothing shown in the records cancellation of petitioner’s registration. The charter
nor it is claimed by PAFLU that the local union was certificate of petitioner was eventually revoked on the
expressly forbidden to disaffiliate from the federation ground of prohibited mixture of supervisory and rank
nor were there any conditions imposed for a valid and file employees and non-compliance with the
breakaway. attestation clause under Art 235 of the Labor Code.
The Acting DOLE secretary reversed the earlier ruling
of the Med Arbiter while this was later reversed by
Cancellation of Registration the CA. The SC ruled that the erroneous inclusion of
one supervisory employee in the union is not a
NOTE: Please read Omnibus Rules, Book V, Rule XIV- ground to impugn the legitimacy of a legitimate labor
XV, as amended by D.O. 40-03 organization which had the right to file a petition for
Article 245, Labor Code. Cancellation of certification election. With the certificates of
registration. - The certificate of registration of any registration issued in favor of petitioner, they are
legitimate labor organization, whether national or clothed with legal personality as a legitimate labor
local, may be cancelled by the Bureau, after due organizations. Moreover, such legal personality
hearing, only on the grounds specified in Article cannot thereafter be subject to collateral attack, but
239 [renumbered 247] hereof.
may be questioned only in an independent petition
for cancellation of certificates of registration. Even
though the DOLE initially revoked the petitioner’s
Article 246, Labor Code. Effect of a petition for
cancellation of registration. - A petition for charter certificate, petitioner has moved for its
cancellation of union registration shall not suspend reconsideration of such resolution.
the proceedings for certification election nor shall it
prevent the filing of a petition for certification Mariwasa v. Sec. of DOLE, December 21,
election.
2009
In case of cancellation, nothing herein shall restrict SMMSC-Independent was issued a certificate of
the right of the union to seek just and equitable registration as a legitimate labor organization by the
remedies in the appropriate courts. DOLE. Mariwasa led a petition for cancellation of
union registration against SMMSC-Independent,
claiming that the latter violated Article 234 for not
Article 247, Labor Code. Grounds for
complying with the 20% minimum requirement
cancellation of union registration.- The following
may constitute grounds for cancellation of union because of the disaffiliation of 102 employees who
registration: executed affidavits recanting their union membership
after the union filed a petition for certification
(a) Misrepresentation, false statement or fraud in election and that it committed misrepresentation in
connection with the adoption or ratification of the violation of Article 239. The Labor Code, Article 234
constitution and by-laws or amendments thereto, merely requires a 20% minimum requirement during
the minutes of ratification, and the list of members
who took part in the ratification; the application for union registration and does not
(b) Misrepresentation, false statements or fraud in mandate that a union maintain the 20% minimum
connection with the election of officers, minutes of membership requirement all throughout its existence.
the election of officers, and the list of voters; In this case, the total number at the time of
(c) Voluntary dissolution by the members. registration was 169 out of 528 rank and file
employees -- which was 32%. As to the allegation of
NOTE: Article 247 is for petition for cancellation of misrepresentation, the bare fact that two signatures
union registration, and not for opposing certification appeared twice on the list of those who participated
election. in the organizational meeting would not provide a
valid reason to cancel SMMSC’s certificate of
registration. In this case, the failure of SMMSC to
SAMMA-LIKHA vs. SAMMA Corporation,
indicate with mathematical precision the total
March 13, 2009 number of employees in the bargaining unit is of no
Petitioner SAMMA-LIKHA filed a petition for moment as it was able to comply with the 20%
certification election. This was opposed by minimum membership requirement .
respondent claiming among others that petitioner
filed to prove existence as a local chapter and it had
a prohibited mixture of supervisory and rank and file

16
Heritage Hotel Manila v NUWHRAIN- Republic of the Philippines, represented
HHMSC, January 12, 2011 by DOLE v. Kawashima Textile, July 23,
The union filed a petition for certification election but 2008
it was only after 5 years that the pre-election was KFWU filed with the DOLE Regional Office a petition
made. Heritage Hotel discovered that the union did for certification election to be conducted but
not submit its annual financial reports since the year Kawashima opposed on the ground that KFWU did not
it was registered thus it filed a petition for acquire legal personality because its membership of
cancellation of the union’s registration on the ground mixed rank and file and supervisory employees and
of failure to submit said annual financial reports and failure to submit books of account. In the case at bar,
list of members to the BLR. The SC held that the the union’s membership list contains the names of at
union’s registration cannot be cancelled on such least 27 supervisory employees, the union could not,
ground. The union had submitted its annual financial prior to purging itself of its supervisory employee
reports and list of members although belatedly, thus members, attain the status of a legitimate labor
it was considered sufficient compliance as the organization. Not being one, it could not have
Regional Director is given license to treat late filing as possessed the requisite personality to file a petition
such. The union members belonging to the for certification election based on RA 6716. However,
bargaining unit should not be deprived of a on June 21, 997, DO no. 9 removed the requirement
bargaining agent because of the negligence of the that the petition for certification election indicate that
union officers who were merely responsible for the the bargaining unit of rank and file has not mingled
submission of the documents to the BLR. with supervisory employees thus the decision of
DOLE granting the certification election petition was
NOTE: When this case was filed, non-submission of reinstated. As to the issue of whether an employer
financial reports was a ground for cancellation, but can collaterally attack the legitimacy of a labor
presently, it is not one of the grounds under Article organization by filing a MTD, an employer is a mere
239. bystander to any petition for certification election
except when it is requested to bargain collectively.
Legend International Resorts Limited v. Such proceeding is non-adversarial and merely
Kilusang Manggagawa ng Legenda, investigative, for the purpose is to determine which
organization will represent the employees in their
February 23, 2011 collective bargaining with the employer. The
The union filed a petition for certification election employer’s only right in the proceeding is to be
which Legend International seeks to dismiss alleging notified and informed.
that the former is not a legitimate labor organization
due to its membership being a mixture of supervisory
and rank and file employees. The union countered
DHL Phils. United Rank and File
that the certification election should still proceed as it Association v. Buklod ng Manggagawa
complied with the required number. At the time the ng DHL Phils, July 22, 2004
petition for certification election is filed, the A certification election was conducted among the
petitioning union is presumed to possess the legal regular rank and file employees of DHL whereby the
personality to file the same. Thus, a certification contending choices were the petitioner and no union.
election may still be conducted during the pendency BUKLOD filed a petition for the nullification of the
of the cancellation proceedings. There is thus no certification election and the officers of the union
basis for Legend’s allegation that the cancellation of were charged with committing fraud and deceit in the
certificate of registration should retroact to the time election proceedings as they misrepresented to the
of issuance or that it effectively nullified all the voter-employees that it was an independent union
union’s activities. Furthermore, the legitimacy of the when it was in fact an affiliate of FFW. The SC held
legal personality of a union cannot be collaterally that the certification election was invalid as a number
attacked in a petition for certification election of employees were lured by their officers into
proceeding. Such legal personality may not be believing that the petitioner was an independent
subject to a collateral attack but only through a union. The making of false statements or
separate action instituted particularly for the purpose misrepresentations that interfere with the free choice
of assailing it. of the employees is a valid ground for protest. A
certification election may be set aside for
misstatements made during the campaign, where (1)
a material fact has been misrepresented in the
campaign; (2) an opportunity for reply has been
lacking; (3) the misrepresentation has had an impact
17
on the free choice of the employees participating in and faculty.” Moreover, while the CA may have ruled
the election. Misrepresentation is likely to have an that there is no mutuality or commonality of interests
impact on their free choice, if it comes from a party among the members of BIGKAS, this is not enough
who has special knowledge or is in an authoritative reason to cancel its registration. The only grounds for
position to know the true facts. cancellation are those found in Art 247 of the Labor
Code. The inclusion in a union of disqualified
Asian Institute of Management v. Asian employees is not among the grounds for cancellation,
unless such inclusion is due to misrepresentation,
Institute of Management Faculty false statement or fraud under the circumstances
Association, January 23, 2017 enumerated in Sections (a) and (c) of Article 247.
Respondent filed a petition for certification election Thus, for purposes of decertifying a union, it is not
seeking to represent a bargaining unit in AIM enough to establish that the rank and file union
consisting of 40 faculty members. Petitioner opposed includes ineligible employee in its membership.
claiming that respondent’s members are managerial Pursuant to paragraph (a) and (b) of Article 247 of the
employees. Later, petitioner filed a petition for Labor Code, it must be shown that there was
cancellation of respondent’s certificate of registration misrepresentation, false statement or fraud in
on the grounds of misrepresentation in registration connection with: (1) the adoption or ratification of the
and that respondent is composed of managerial constitution and by-laws or amendments thereto; (2)
employees who are prohibited from organizing as a the minutes of ratification ; the election of officers;
union. The SC said that petitioner was correct in filing (3) election of officers; (4) the minutes of the election
of a petition for cancellation of respondent's of officers; and (5) the list of voters.
certificate of registration. Petitioner’s sole ground for
seeking cancellation of respondent’s certificate of
registration - that its members are managerial
employees and for this reason, its registration is thus Bargaining Unit
patent nullity for being an absolute violation of Art
245 of the Labor Code, which declares that Bargaining Unit, defined
managerial employees are ineligible to join any labor a group of employees of a given employer, comprised
organization - is in a sense, an accusation that
of all or less than all of the entire body of employees,
respondent is guilty of misrepresentation for which the collective interest of all the employees,
registering under the claim that its members are not
consistent with equity to the employer, indicate to be
managerial employees. the best suited to serve the reciprocal rights and
duties of the parties under the collective bargaining
De Ocampo Memorial Schools, Inc. v. provisions of the law.
Bigkis Manggagawa sa De Ocampo
Memorial School, Inc., March 15, 2017 Rule 1, Sec. 1(d), Omnibus Rules, Book V, as
Petitioner filed a petition for cancellation of certificate amended by DO 40-03. BARGAINING UNIT -
refers to a group of employees sharing mutual
registration against BIGKIS based on
interests within a given employer unit, comprised
misrepresentation false statement, and fraud in of all or less than all of the entire body of
connection with its creation and registration as a employees in the employer unit or any specific
labor union as it shared the same set of officers and occupational or geographical grouping within such
members with another union, LAKAS. Further, there employer unit.
was an alleged mixture of supervisory
employees/managerial employees with a rank and file
union. Rule 1, Sec. 1(t), Omnibus Rules, Book V, as
amended by DO 40-03. EXCLUSIVE BARGAINING
REPRESENTATIVE - refers to a legitimate labor
The SC said that for fraud and misrepresentation to
union duly recognized or certified as the sole and
constitute as a ground for cancellation of union exclusive bargaining representative or agent of all
registration, the nature of the fraud and the employees in a bargaining unit.
misrepresentation must be grave and compelling
enough to vitiate the consent of a majority of the
union members. In this case, BIGKIS did not commit San Miguel Corp. v. Laguesma,
the alleged acts. In its application, even if it has the
September 21, 1994
same set of officers and members, the applicant
A petition for certification election was filed by the
indicated in the portion “description of bargaining
union among all regular sales personnel of Magnolia
unit” that is composed of “rank and file” and under
Dairy Products in the North Luzon Sale Area. SMC
the “occupational classification”, it marked “technical
18
opposed claiming that its bargaining history in its example, is the Dean. Non-academic rank and file
sales offices, plants and warehouses is to have a employees of UP shall constitute a bargaining unit to
separate bargaining unit for each sales office. The the exclusion of the academic employees of the
union won the election. SMC argues that its prior institution who may also organize themselves into a
collective bargaining history is the most persuasive separate collective bargaining unit as the test in
criterion in determining the appropriateness of the determining the appropriate bargaining unit is the
collective bargaining unit. The SC said that the union mutuality of interest test. Applying this test, teachers
represents an appropriate bargaining unit. would find almost nothing in common with the non-
academic personnel as regards working conditions,
A bargaining unit is a "group of employees of a given compensation rates, skills and intellectual pursuits.
employer, comprised of all or less than all of the
entire body of employees, consistent with equity to Golden Farms v. Secretary, July 26, 1994
the employer, indicate to be the best suited to serve PFL filed a petition before the Med-Arbiter praying for
the reciprocal rights and duties of the parties under the holding of a certification election among the
the collective bargaining provisions of the law." The monthly paid office and technical rank and file
fundamental factors in determining the appropriate employees of Golden Farms. It filed a MTD when PFL
collective bargaining unit are: failed to show that it was organized as a chapter
(1) the will of the employees (Globe within GF’s establishment and that there was already
Doctrine); an existing CBA between the rank and file employees
(2) affinity and unity of the employees' represented by NFL and GF. PFL opposed, contending
interest, such as substantial similarity of work that monthly paid office and technical employees
and duties, or similarity of compensation and should be allowed to form a separate bargaining unit
working conditions (Substantial Mutual because
Interests Rule); The monthly paid office and technical rank-and- file
(3) prior collective bargaining history; and employees of petitioner Golden Farms enjoy the
(4) similarity of employment status. constitutional right to self-organization and collective
bargaining. The monthly paid rank-and- file
The Court held that the existence of a prior collective employees of petitioner primarily perform
bargaining history is neither decisive nor conclusive administrative or clerical work. In contradistinction,
in the determination of what constitutes an the petitioner’s daily paid rank-and- file employees
appropriate bargaining unit. The test of grouping is mainly work in the cultivation of bananas in the
the mutuality or commonality of interests. The fields. It is crystal clear the monthly paid rank-and-
employees sought to be represented must have file employees of petitioner have very little in
substantial mutual interests in terms of employment common with its daily paid rank-and-employees in
and working conditions as evidenced by the type of terms of duties and obligations. This dissimilarity of
work they perform. interests warrants the formation of a separate and
distinct bargaining unit for the monthly paid rank-
University of the Philippines v. Ferrer- and-file employees of GF. To rule otherwise would
Calleja, July 14, 1992 deny this distinct class of employees the right to self-
ONAPUP filed a petition with the BLR for certification organization for purposes of collective bargaining.
election among all the non-academic employees of
UP but All UP Worker’s Union intervened in the Mechanical Department Labor Union sa
petition, alleging that its membeship covers both
academic and non-academic personnel and that it
PNR v. CIR, August 30, 1968
aims to unit all rank and file employees in one union. Philtranco Services Enterprises v. BLR,
Director Calleja, citing EO 180 and Rule IV, Section 1 June 28, 1989
of the Rules implementing EO 180, ruled that the The Locomotive and Motor Car Crew of the Caloocan
appropriate organizational unit should include all rank shops are represented by the Union de Maquinistas,
and file employees, teaching and non-teaching of UP. while the workers under the Operations and Shops
UP thereafter sought exclusion from the Rolling Stocks are represented by the Mechanical
organizational unit certain employees holding Department Labor Union. Said Rolling Stocks
supervisory positions among non-academic personnel Maintenance Division seek to be separated from the
and those with ranks of Assistant Professor. Relevant rest of the workers of the department and to be
laws and the University Charter show that associate represented by Samahan ng mga Manggagawa sa
professors, assistant professors and associate high Caloocan shops. Thereafter, Samahan filed a petition
level employeesare not managerial but are rank and calling attention to the fact that there were unions in
employees. What is a managerial employee, for the Caloocan shops of the PNR and that no
19
certification election had been held in the last 12 employees. It follows that the members of KASAMA
months for which reason a certification election was KO who are professional, technical, administrative
needed to determine the proper collective bargaining and confidential personnel of PHILTRANCO performing
representative. Said petition was opposed by the managerial functions are not qualified to join, much
management and the Mechanical Labor Union as it less form a union. Managers by any name may not
had previously been certified as the sole and join the rank and file union. On the other hand, those
exclusive bargaining agent of the employees of PNR’s who are rank and file workers may join the existing
mechanical department and that before expiration of bargaining unit instead of organizing another
the CBA, a renewal had been negotiated. The CIR bargaining unit and compelling the employer to deal
judge held that based on the “Globe” doctrine, the with it. Furthermore, there are no compelling reasons
employees in the Caloocan Shops should be given a in this case such as a denial to the KASAMA KO group
chance to vote on whether their group should be of the right to join the certified bargaining unit or
separated from that represented by the Mechanical substantial distinctions warranting the recognition of
Department Labor Union, and ordered a plebiscite a separate group of rank and file workers. Precisely,
held for the purpose. NAMAWUMIF intervened to make it clear it has no
objections to qualified rank and file workers joining its
Those in the Caloocan shops, contrary to union.
Mechanical’s argument that these said workers do
not require different skills from the rest of the workers Philippine Scout Veterans Security and
in the Mechanical Department, not only have a
community of interest and working conditions but
Investigation Agency v. Secretary, July
perform major repairs of railway rolling stock, using 21, 1993
heavy equipment and machineries found in said The Union filed a petition for direct certification
shops, while the others only perform minor repairs. It election among the rank and file employees of PVSIA,
is easy to understand, therefore, that the workers in GVM and ASDA. These said companies filed a single
the Caloocan shops require special skill in the use of comment alleging that they have separate and
heavy equipment and machinery sufficient to set distinct personalities while PGA Security Agency is
them apart from the rest of the workers. Said appeal not a business or corporate entity and does not
of Mechanical as to whether the Globe Doctrine is possess any personality, and filed a mTD on the
applicable is premature at best since the result of the ground that the 721 supporting signatures do not
ordered plebiscite among the workers of the Caloocan meet the 20% minimum requirement for certification
shops who desire to form a new bargaining unit may election as the number of employees total 2,374 and
be adverse to the formation of a separate unit. that there are no implementing rules yet of RA 6715.
Med-Arbiter ruled that the companies should be
deemed as a single entity and bargaining unit for the
Philtranco Service Enterprises Inc v. purpose of union organizing and the holding of a
BLR, G.R. no. 85343, June 28, 1989 certification election.
KASAMA KO, a registered labor organization, filed a
petition for certification election with DOLE seeking to As to the allegation that the Union failed to meet the
represent all professional and confidential employees 20% requirement that must support the petition,
and personnel of Philtranco and that no certification there was no need for the union to prove the 20%
election has been held in the past 3 years prior to the requirement. Under the amendments of the Labor
filing of the petition. NAMAWU-MIF filed a motion for Code, when a duly organized labor union files a
intervention alleging that it was the bargaining agent petition for certification election, Med-Arbiter has the
of the said workers but the Arbiter dismissed said duty to automatically conduct an election and has no
petition stating that the eligible employees must be discretion on the matter. This is the mandate of
included in the existing bargaining unit. Article 257 which states that: “In any establishment
where there is no certified bargaining agent, a
The Labor Code recognizes two (2) principal groups of certification election shall automatically be conducted
employees, namely, the managerial and the rank and by the Med-Arbiter upon the filing of a petition by a
file groups. Thus, Art. 212 (k) of the Code provides: legitimate labor organization. Further, an employer
“(k) ‘Managerial employee’ is one who is vested with has nothing to do with a certification election except
powers or prerogatives to lay down and execute when it is requested to bargain collectively.
management policies and/or to hire, transfer,
suspend, layoff, recall, discharge, assign or discipline
employees, or to effectively recommend such
managerial actions. All employees not falling within
this definition are considered rank and file
20
International School Alliance of
Educators v. Quisumbing, June 8, 2000 [SUPRA] San Miguel Foods v. San Miguel
The School hires both foreign and local heirs but Corp. Supervisors and Exempt Union,
grants the foreign heirs certain benefits not accorded Aug. 1, 2011
to the former, such as housing, transportation, While the existence of a bargaining history is a factor
shipping costs, taxes, salary rate 25% more than that may be reckoned with in determining the
local-hires and home-leave travel allowance. CBA was appropriate bargaining unit, the same is not decisive
held where ISAE contested the difference in salary or conclusive. Other factors must be considered. The
rates. Foreign heirs do not belong to the same test of grouping is community or mutuality of
bargaining unit as local heirs. The factors in interest. Certain factors, such as specific line of work,
determining the appropriate collective bargaining unit working conditions, location of work, mode of
are (1) the will of the employees; (2) affinity and unity compensation, and other relevant conditions do not
of the employees’ interest, such as substantial affect or impede their commonality of interest.
similarity of work and duties, or similarity of Although they seem separate and distinct from each
compensation and working conditions (3) prior other, the specific tasks of each division are actually
collective bargaining history; and (4) similarity of interrelated and there exists mutuality of interests
employment status. The basic test of an asserted which warrants the formation of a single bargaining
bargaining unit’s acceptability is whether or not it is unit.
fundamentally the combination which will best assure
to all employees the exercise of their collective
bargaining rights. It does not appear that foreign-
Lepanto Consolidated Mining v. The
hires have indicated their intention to be grouped Lepanto Capataz Union, February 18,
together with local-hires for purposes of collective 2013
bargaining. Although foreign-hires perform similar The Union filed a petition for consent election with
functions under the same working conditions as the the Industrial Relations Division of the CAR of the
local-hires, foreign-hires are accorded certain benefits DOLE, proposing to represent 139 capatazes of
not granted to local-hires. To include foreign-hires in a Lepanto. Lepanto opposed the petition, contending
bargaining unit with local-hires would not assure that the Union was in reality seeking a certification
either group the exercise of their respective collective election, not a consent election, and would thereby
bargaining rights. be competing with the Lepanto Employees Union
(LEU), the current collective bargaining agent, as the
De La Salle v. De La Salle University capatezes were already members of the LEU.
Employees’ Association, Apr. 12, 2000 Capatezes or foremen are not rank-and file
DLSU Union entered into a CBA. During the 60 day employees because they are an extension of the
freedom period (before expiration of said CBA), the management, and as such they may influence the
Union initiated negotiations with DLSU for a new rank-and- file workers under them to engage in
collective bargaining agreement which turned out to slowdowns or similar activities detrimental to the
be unsuccessful, like the issue of the scope of the policies, interests or business objectives of the
bargaining unit if it should include employees of CSB. employers. were performing functions totally different
During the freedom period, the parties may not only from those performed by the rank-and- file
renew the existing collective bargaining agreement employees, and that the capatazes were supervising
but may also propose and discuss modifications or and instructing the miners, mackers and other rank-
amendments. Express exclusion of the computer and-file workers under them, assessing and
operator and discipline officers from the bargaining evaluating their performance, making regular reports
unit of rank and file employees in the 1986 CBA does and recommending new systems and procedure of
not bar any re-negotiation for the future inclusion of work, as well as guidelines for the discipline of
the said employees in the bargaining unit. employees. Hence, Med-Arbiter Lontoc concluded and
Furthermore, computer operators and discipline affirmed by SC, that the capatazes differed from the
officers are not confidential employees as the rank-and- file and could by themselves constitute a
former’s duties are basically clerical and non- separate bargaining unit.
confidential in nature. The employees of the College
of St. Benilde should be excluded from the bargaining Fulache vs. ABS-CBN Broadcasting
unit of the rank-and- file employees of DLSU because Corporation, G.R. 183810, January 21,
the two educational institutions have their own
separate juridical personality and no sufficient
2010
evidence was shown to justify the piercing of the veil Fulache filed two separate complaints for
of corporate fiction. regularization and illegal dismissal, alleging that ABS-
21
CBN and the Union executed a CBA but they were 22 rank and file professional and technical employees
excluded from it. They claim that they had already of COCA-COLA. The company, however, opposed the
rendered more than a year’s service and should be petition as several employees in the IPTEU are
recognized as regular employees. Only regular confidential employees, but the petition was granted
employees who fall under the coverage of the by the Med-Arbiter. Thereafter, COCA COLA and IPTEU
bargaining unit are therefore entitled to CBA benefits mutually agreed to conduct a certification election
as a matter of law and contract. SC ruled that Fulache and 16 employees voted. COCA-COLA again filed a
et al were regular employees as they were engaged protest arguing that the 16 employees were
in the performance of activities usually necessary or confidential employees and that there is already the
desirable in ABS-CBN’s trade of business. As regular IMU which represented the rank and file.
employees, they fall within the coverage of the Subsequently, IPTEU was proclaimed as the EBR for
bargaining unit and are entitled to benefits as a the rank and file exempt workers of COCA-COLA. The
matter of law. Mediator-Arbiter ruled that employees who encounter
or handle trade secrets and financial information are
Holy Child Catholic School v. HCCS- not automatically classified as confidential
employees. It was admitted that the subject
TELU-PIGLAS, July 23, 2013 employees encounter and handle financial as well as
HCCS-TELU-PIGLAS filed a petition for certification physical production data and other information which
election but such filing was opposed by Holy Child are considered. The SOLE, which the CA affirmed,
Catholic School on the grounds that the members of likewise held that the questioned voters do not have
the union do not belong to the same class of access to confidential labor relations information.
employees, which included managerial, confidential,
rank and file, teaching and non-teaching personnel, The SC defers to the findings of fact of the Mediator-
and that it is an illegitimate labor organization lacking Arbiter, the SOLE, and CA. An employee must assist
in personality to file such and inappropriate or act in a confidential capacity and obtain
bargaining unit for want of community of interest. confidential information relating to labor relations
policies. Exposure to internal business operations of
In determining the proper collective bargaining unit the company is not per se a ground for the exclusion
and what unit would be appropriate to be the in the bargaining unit.
collective bargaining agency, several factors should
be considered: (1) will of employees (Globe Doctrine);

Bargaining Agent and


(2) affinity and unity of employees’ interest, such as
substantial similarity of work and duties, or similarity
of compensation and working conditions; (3) prior
collective bargaining history; and (4) employment
status, such as temporary, seasonal and probationary
Certification Election
employees. We stressed, however, that the test of the
grouping is community or mutuality of interest,
Proceedings
because “the basic test of an asserted bargaining Article 267, Labor Code. Exclusive bargaining
unit’s acceptability is whether or not it is representation and workers’ participation in policy and
fundamentally the combination which will best assure decision-making. - The labor organization designated
to all employees the exercise of their collective or selected by the majority of the employees in an
bargaining rights. The teaching and non-teaching appropriate collective bargaining unit shall be the
personnel of the school must form separate exclusive representative of the employees in such
unit for the purpose of collective bargaining.
bargaining units. Thus, the order for the conduct of
However, an individual employee or group of
two separate certification employees shall have the right at any time to
elections, one involving teaching personnel and the present grievances to their employer.
other involving non- teaching personnel based on the
UP case which prohibited commingling teaching and Any provision of law to the contrary
non-teaching personnel. notwithstanding, workers shall have the right,
subject to such rules and regulations as the
Secretary of Labor and Employment may
Coca-Cola Bottlers Philippines, Inc. vs. promulgate, to participate in policy and decision-
Ilocos Professional and Technical making processes of the establishment where they
are employed insofar as said processes will directly
Employees Union (IPTEU), September affect their rights, benefits and welfare. For this
09, 2015 purpose, workers and employers may form labor-
management councils: Provided, That the
IPTEU filed a verified petition for certification election representatives of the workers in such labor-
seeking to represent the bargaining unit consisting of
22
management councils shall be elected by at least
the majority of all employees in said establishment. Article 271, Labor Code. EMPLOYER as
BYSTANDER — In all cases (whether the petition for
certification election is led by an employer OR a
Article 268, Labor Code. Representation Issue legitimate labor organization), the employer shall
in Organized Establishments. - In ORGANIZED NOT be considered a party thereto with a
establishments, when a verified petition concomitant right to oppose a petition for
questioning the majority status of the incumbent certification election.
bargaining agent is filed before the DOLE within the The employer's participation in such proceedings
60-day period before the expiration of the shall be LIMITED to:
collective bargaining agreement, the Med-Arbiter (1) being notified or informed of petitions of
shall automatically order an election by secret such nature; and
ballot WHEN the verified petition is supported by (2) submitting the list of employees during
the written consent of at least 25% of all the the pre-election conference should the
employees in the bargaining unit to ascertain the Med-Arbiter act favorably on the petition.
will of the employees in the appropriate bargaining
unit.
Article 272, Labor Code. APPEAL from
To have a valid election, at least a majority of all certification election orders. - Any party to an
eligible voters in the unit must have cast their election may appeal the order or results of the
votes. The labor union receiving the majority of the election as determined by the Med-Arbiter directly
valid votes cast shall be certified as the exclusive to the SOLE on the ground that the rules and
bargaining agent of all the workers in the unit. regulations or parts thereof established by the
SOLE for the conduct of the election have been
WHEN an election which provides for 3 or more violated. Such appeal shall be decided within 15
choices results in NO CHOICE receiving a majority calendar days. (As amended by Section 25,
of the valid votes cast, a RUN-OFF ELECTION Republic Act No. 6715, March 21, 1989)
shall be conducted BETWEEN the labor unions
receiving the 2 highest number of votes:
PROVIDED, that the total number of votes NOTE: Read Omnibus Rules, Book V, Rule I, Sec. 1 (d,
for all contending unions is at least 50% of h, j, o, p, q, t, ll, ss, bbb, a, tt, ww), Rules VI-X, as
the number of votes cast. amended by D.O. 40, D.O. 40-F-03, series of 2008,
and further amended by D.O. 40-I-15, series of 2015
At the EXPIRATION of the freedom period, the
employer shall CONTINUE to recognize the
majority status of the incumbent bargaining agent
WHERE NO petition for certification election is filed. Duty to Bargain Collectively
(As amended by Section 23, Republic Act No. 6715,
March 21, 1989) Lakas ng Manggagawang Makabayan v.
Marcelo Enterprises, November 19,
Article 269, Labor Code. Petitions in 1982
UNORGANIZED establishments. - In any Confronted with the problem of whom to recognize as
establishment where there is NO certified the bargaining representative of all its workers, the
bargaining agent, a certification election shall company asked for proof of authority to represent
automatically be conducted by the Med-Arbiter
MFWU and MACATIFU and apprised LAKAS of the
UPON the filing of a petition by a legitimate labor
organization. (As amended by Section 24, Republic existing conflicting demands for recognition as
Act No. 6715, March 21, 1989) bargaining representative in the units involved. The
company expressed conformity to negotiate as soon
as LAKAS can present evidence of authority and four
Article 270, Labor Code. When an EMPLOYER conferences were held but LAKAS refused all of the
may file petition. - When requested to bargain management’s offers. LAKAS thereafter declared a
collectively, an employer may petition the Bureau strike against all of the Marcelo companies which was
for an election. IF there is NO existing certified attended by violence. A return to work agreement
collective bargaining agreement in the unit, the
was executed whereby Marcelo Companies resumed
Bureau shall (after hearing) order a certification
election. operations and strikers went back to work, except
four who chose not to because of the criminal
All certification cases shall be decided WITHIN 20 charges filed against them. LAKAS declared another
working days. The Bureau shall conduct a strike. Some strikers returned to work and were
certification election WITHIN 20 days in accordance requested to indicate their availability for work in
with the rules and regulations prescribed by the
order that they may be scheduled but some refused
Secretary of Labor.
to return as they did not want to comply with the

23
requirement Thus, LAKAS filed an unfair labor the respondents this is because they were of the
practice alleging non-readmission of the impression that before a union could have that
striking members despite unconditional offer to capacity, it must first be certified by the Court of
return to work after the strike. Industrial Relations as the duly authorized bargaining
unit. Thus, the fact that the Company had agreed to
Marcelo Companies did not ignore the demand for only some of the demands shows that she did not
collective bargaining nor did it refuse to bargain at refuse to bargain collectively with the complaining
all. What it did was apprise LAKAS of the existing union.
conflicting demands for recognition as bargaining
representative in the units involved and suggested Liberty Flour Mills Employees
the settlement of the issue by filing a petition for
certification election. Thus, where the issue of
Employees v. Liberty Flour Mills,
legitimate representation in dispute is viewed for not December 29, 1989
only by one legitimate labor organization but two or Union and Company entered into a CBA. The CBA had
more, there is every ground warranting the holding of a union shop clause, that employees should maintain
a certification election. Although an employer has the membership in good standing in the Union as a
undoubted right to bargain with a bargaining agent condition for continued employment in the company.
whose authority has been established, without the Later, 2 employees (petitioners) veered away from
requirement that the bargaining agent be officially the Union and created a New Union. Through New
certified by the National Labor Relations Board as Union, they filed a petition for Certification Election.
such, if the informally presented evidence leaves a So, Union expelled the 2 employees. While this was
real doubt as to the issue, the employer has a right to happening, the above-mentioned CBA was certified.
demand a certification and to refuse to negotiate The company then dismissed the 2 employees from
until such official certification is presented. work pursuant to the union shop clause. They filed a
case against their dismissal. The issue here is
It is essential to the right of a putative bargaining whether the 2 employees could be validly dismissed
agent to represent the employees that it be the based on a union shop clause in a CBA that was NOT
delegate of a majority of the employees and, yet certified. The Court ruled that the 2 employees
conversely, an employer is under duty to bargain were actually dismissed after certification. But,
collectively only when the bargaining agent assuming arguendo that they were dismissed before
representative of the majority of the employees. A the certification of the CBA, they could still be validly
natural consequence of these principles is that the dismissed. The certification of the collective
employer has the right to demand of the asserted bargaining agreement by the BLR is not required to
bargaining agent proof of its representation of its put a stamp of validity to such contract. Once it is
employees. Having the right to demonstration of this duly entered into and signed by the parties, a
fact, it is not an unfair labor practice for an employer collective bargaining agreement becomes effective as
to refuse to negotiate until the asserted bargaining between the parties regardless of whether or not the
agent has presented reasonable proof of majority same has been certified by the BLR.
representation.
Colegio de San Juan de Letran v.
NOTE: The duty to bargain collectively exists when
there is a certification as to who to bargain with.
Association of Employees and Faculty of
Letran, September 13, 2000
National Union of Restaurant Workers Letran unilaterally suspended CBA re-negotiations
with existing EBR after purportedly receiving
(PTUC) V. C1R, April 30, 1964 information that another labor organization filed a
A complaint for unfair labor practice was lodged petition for certification election. The Court held
against the owners of Tres Hermanas Letran is guilty for unfair labor practice for not
Restaurant on the ground that the company refused fulfilling their duty to collectively bargain. Letran’s
to bargain collectively with the union. The Company excuse is not a valid ground to suspend CBA
made a counter offer wherein the petitioner would be renegotiations. In order to allow the employer to
accepted if the same would become a company validly suspend the bargaining process there must be
union. Also, an employee was separated from the a valid petition for certification election raising a
service because he was found to be the organizer and legitimate representation issue. Hence, the mere
adviser of the complaining union. SC ruled that the filing of a petition for certification election does not
Company did not refuse to bargain collectively with ipso facto justify the suspension of negotiation by the
the Union. While it is true that it denied the capacity employer. The petition must first comply with the
of the complaining union to bargain collectively with provisions of the Labor Code and its Implementing
24
Rules. Foremost is that a petition for certification Manila Electric Co. v. Quisumbing,
election must be filed during the 60 day freedom
period. The "Contract Bar Rule" under Section 3, Rule
January 27, 1999
XI, Book V, of the Omnibus Rules, as based on the MERALCO and MEWA had an existing CBA from 1992-
Labor Code, provides that: ". . . If a collective 1997, and they sought to negotiate the conditions
bargaining agreement has been duly registered in covering the remaining period from Dec. 1 , 1995 to
accordance with the Code, a petition for certification Nov. 30, 1997. They failed to agree on the terms
election or a motion for intervention can only be resulting in a deadlock. The Secretary later assumed
entertained within 60 days prior to the expiry date of jurisdiction over their labor dispute. The Secretary
such agreement." No petition for certification election later issued an order (fixing benefits, wage increases,
for any representation issue may be filed after the and other economic benefits AND also determined
lapse of the 60 freedom period. The old CBA is the retroactivity of the CBA) which was opposed by
extended until a new one is signed. The rule is that MERALCO alleging grave abuse of discretion. Court
despite the lapse of the formal effectivity of the CBA ruled that the retroactivity of arbitral awards shall
the law still considers the same as continuing in force commence at such time as granted by the Secretary.
and effect until a new CBA shall have been validly As provided under the law, if no agreement is reach
executed. In this case, the petition was filed by the within 6 months from the expiry date of the 3-years
other union outside the freedom period. As such, the that follow the CBA execution, the law expressly gives
suspension of negotiations by Letran was invalid. the parties, not anybody else, the discretion to fix the
effectivity of the agreement. In case no agreement is
reached, the (1) principle of hold over shall apply,
San Miguel Corporation Employees which provides that “in the absence of a new CBA,
Union-PTGWO v. Confesor, September the parties shall maintain the status-quo and must
19, 1996 continue in full force and effect the terms and
Union entered into a CBA with SMC. Later, the SMC conditions of the existing agreement until a new
informed its employees that the company will agreement is reached. The (2) arbitrated CBA may
undergo a restructuring. Thus, two corporations were also take on the nature of a judicial or quasi-judicial
formed Magnolia and SMF. Notwithstanding the spin- award and may be executed. (READ NEXT CASE)
offs, the CBA still remained in force. During
negotiation, the union insisted that the bargaining Manila Electric Co. v. Quisumbing,
unit of SMC shall still include the employees of the February 22, 1999
spun-off corporations, Magnolia and SMF. Also, they Motion for reconsideration from previous case; main
contended that the renegotiated terms of the CBA issue is whether the grant of arbitral awards are
shall be effective only for the remaining period of 2 retroactive. Court ruled that CBA arbitral awards
years. SMC however says otherwise. The SOLE ruled granted after 6-months from the expiration of the last
that the employees of SMF and Magnolia are no CBA shall retroact to such time agreed upon by both
longer covered. employer and the employees or their union. Absent
agreement for retroactivity, the award shall retroact
The SC ruled that bargaining unit of SMC does not to the first day after the 6-month period following the
include the employees of SMF and Magnolia. It ruled expiration of the last day of the CBA. In the absence
that Magnolia and SMF became distinct entities with of a CBA, the Secretary’s determination of the date of
separate juridical personalities. In determining an retroactivity as part of his discretionary powers over
appropriate bargaining unit, the test of grouping is arbitral awards shall control. Although an arbitral
mutuality or commonality of interest. The employees award cannot be categorized as an agreement
sought to be represented by the collective bargaining voluntarily entered into by the parties because it
agent must have substantial mutual interests in requires the interference and imposing power of the
terms of employment and working conditions as State thru the Secretary, the arbitral award can be
evidenced by the type of work they perform. In this considered as an approximation of a CBA which would
case, SMC is engaged in the business of beer otherwise have been entered into by the parties. The
manufacturing while Magnolia is involved in the terms or periods (for retroactivity) set forth in the
manufacturing and processing of dairy products and Labor code pertains to a CBA, there is nothing that
SMF is involved in the production of feeds and would prevent such rules to apply to an arbitral
processing of chicken. award by analogy.

25
New Pacific Timber v. NLRC, March 17, Samahang Manggagawa sa Top Form v.
2000 NLRC, September 7, 1998
The union started to negotiate with petitioner Petitioner was the certified collective bargaining
company which was met with resistance. Thus, union representative of all regular rank and file employees
filed a complaint for ULP against the company for of Top Form. A collective bargaining negotiation was
refusal to bargain collectively. The union won its case held. In the minutes of the meeting, an across the
against the company before the Executive LA. The board wage increase was tackled but it was not
case was then placed under the arbitration branch of stated anymore in the CBA since the union dropped
the NLRC for execution. LA ordered the company to such proposals relying to the undertakings made by
pay benefits, thereafter, the union manifested that it the officials of the company. The union eventually
will no longer appeal this decision of the LA. However, requested the implementation of the wage order
it filed a petition for relief saying that 186 employees however they demanded that the increase be on an
were wrongfully excluded from receiving benefits who across-the-board basis. The company refused and
were entitled under the existing CBA. Company eventually implemented a scheme of increase
counters that these employees were not entitled purportedly to avoid wage distortion. The SC held
because they were hired after the terms of the CBA-- that the company did not commit an unfair labor
saying that the provision for wage increase already practice. The Court held that the across-the-board
ended in 1984, and that no new CBA was instituted. wage increase is not part of the CBA. It could only be
Court ruled that the term of the CBA as to its demandable in law if incorporated in the CBA. The
economic provisions can be extended beyond the Minutes only reflects the proceedings and discussions
term expressly stipulated in the absence of a new undertaken in the process of bargaining for worker
CBA, or in other words, even beyond its 3-year benefits. Petitioner union had the right and the
period. Art. 253 of the LC directs that the company opportunity to insist on the foreseeable fulfilment of
(in this case) is duty bound to keep the status quo the company’s promise by demanding its
and to continue in full force and effect the terms of incorporation in the CBA, but they did not. Because
the existing CBA until a new CBA is made. Court also the proposal was never embodied in the CBA, the
ruled, thus, that the 186 employees hired after the promise has remained just that, a promise, the
stipulated term of the CBA are entitled to their implementation of which cannot be validly demanded
benefits because of the status quo. under the law.

Mindanao Terminal and Brokerage Rivera v. Espiritu, January 23, 2002


Service. Inc. v. Confesor, May 5, 1997 PAL pilots affiliated with ALPAP went on a 3 week
Company and the union entered into a CBA for a strike causing serious losses to the airline company
period of 5 years. On the 3rd year, they met to company. Faced with bankruptcy, PAL adopted a
renegotiate the provisions of the CBA. They however rehabilitation plan and downsized its labor force.
failed to resolve some of their differences and thus a PALEA went on a strike to protest the retrenchment
deadlock developed. The union and the company measures adopted the airline company. To address
went back to the bargaining table and eventually the problems of the airline company, the PAL
agreed to certain benefits. The company claimed management and PALEA reached an agreement in
however that the wage increases which it had agreed which shares of stocks will be given to some
to give to the employees should be creditable as employees. The agreement further provides that
compliance with future mandated wage increases. In PALEA agreed to suspend their CBA for 10 years but
addition, it mandated that such increases in the PAL will continue to recognize the said CBA. The SC
renegotiated terms should not be retroactive. The SC said that there is nothing wrong with the agreement.
held that the renegotiated terms are retroactive. The SC said there is nothing in Art 253-A which
Under the Labor Code, all other terms of the CBA prohibits parties from waiving or suspending the
shall be renegotiated not later than 3 years after its mandatory timetables and agreeing on the remedies
execution. Any agreement on such other provisions of to enforce the same. The right to free collective
the CBA entered into within 6 months from the date bargaining after all includes the right to suspend it.
of expiry, shall retroact to the day immediately
following such date. If any such agreement is entered
into beyond 6 months, the parties shall agree on the Jurisdictional Requirements
duration of retroactivity thereof.
Kiok Loy v. NLRC, January 22, 1986
The Union won and became the exclusive bargaining
agent of the rank and file employees of the company
and thus proposed a CBA but was ignored. This
26
prompted the union to file a notice of strike with the bargaining unit. Furthermore, at the time of the
BLR on the ground of unresolved economic issues in supposed recognition, the Company was obviously
collective bargaining. SC ruled that the company was aware that there were other unions existing in the
guilty to unfair labor practice as the Union had made unit (they even went on strike). The unusual
a request to bargain and that the company did not promptitude in the recognition of ALU by the
act on it. Such inaction and refusal by the company to Company as the exclusive bargaining representative
make a counter proposal may indicate bad faith. of the workers under the fluid and amorphous
While it is a mutual obligation of the parties to circumstances then obtaining, was decidedly
bargain, the employer, however, is not under any unwarranted and improvident.
legal duty to initiate contract negotiation. The
mechanics of collective bargaining is set in motion
only when the following jurisdictional preconditions Bargaining Agent and
are present, namely (which are present in this case):
1. possession of the status of majority Certification Election
representation of the employees’
representative in accordance with any of the
Proceedings
means of selection or designation provided
for by the Labor Code; Certification Election Proceedings
2. proof of majority representation; and
3. a demand to bargain under Article 251, par. SEBA GRANT: Not
(a) of the New Labor Code Unorganized certifi Appealable
Labor Certifi
cation DENY:
to Sec. of
Organization cation Appealable
Labor
Electi
ALU v. Ferrer-Calleja, May 5, 1989 (No EBR in BU)
Organized
Certifi
on
to Secretary
GRANT or
(automaticall
Company had 4 existing labor organizations for their cation of Labor CE)
DENY:
y conduct
Labor 2 OR
Conse
Electi Appealable
rank and file employees, one of which was ALU. ALU Organization MORE
nt
on to
sent a letter to the Company informing it that (has EBR in BU) LOs
Certification Elections Electi Secretary of
majority of its employees authorized ALU as its on Labor
1. DOUBLE MAJORITY RULE
exclusive bargaining representative; and so, the
Company recognized it as such and proceeded to a. 1st Majority: Majority of employees in
bargaining unit voted
negotiate and with them for a CBA. The CBA as
executed and filed with the Ministry of Labor. b. 2nd Majority: Union → has majority of
valid votes cast
Contesting this, the other labor organizations
conducted strikes, and filed a petition for Certification NOTE:
● If NO Double Majority = FAILURE of
Election. The question in this case is whether a
petition for CE is proper considering the existence of Elections
● Includes invalid votes when counting
a CBA (contract bar rule)? The Court held that the
petition is proper, and the contract bar rule cannot be votes for (a)
2. If election is valid (satisfies 1st majority), but
applied, since the mechanics of collecting bargaining
were not set in motion. The mechanics of collective no one won and can do Run-Off → then do
Run-Off
bargaining are set in motion only when the following
jurisdictional preconditions are present, namely, (1) 3. If election is valid, but can’t do Run-off →
after __ number of days, can move for
possession of the status of majority representation by
the employees' representative in accordance with conduct of another CE
4. If election fails, wait for another 6 months
any of the means of selection and/or designation
provided for by the LC; (2) proof of majority before another CE can be conducted
representation; and (3) a demand to bargain under
Article 251, paragraph (a), of the New Labor Code. In Requisites to Conduct Run-Off Elections
this case, the only express recognition of ALU as said 1. Valid election
employees’ bargaining representative in the records 2. 3 or more choices (including no union)
is in the CBA the company and ALU hurriedly entered 3. No one gets majority of valid votes cast
into. Other than that, looking at the content of the the 4. Contending unions got at least 50% of the
letters between ALU and the Company, there was votes cast (including invalid votes)
precipitate haste on the part of the Company in 5. No unresolved challenge or election protest
recognizing ALU, which recognition (as exclusive
bargaining representative) appears to have been ● If all requisites are present, can conduct Run-
based on its own self-serving claim that it had the Off Elections:
support of the majority of the employees in the

27
○ The 2 unions who got the 2 highest The inclusion in a union of disqualified employees is
votes will compete with each other not among the grounds for cancellation of union
for the most votes in the elections registration, unless such inclusion is due to
○ No more “no union” as an option in misrepresentation, false statement or fraud under the
this election circumstances enumerated in Sections (a) and (c) of
Article 239 of the Labor Code. Except when it is
FOR EXAMPLE: 200 ELIGIBLE VOTERS IN THE requested to bargain collectively, an employer is a
BARGAINING UNIT → you need at least 101 mere bystander to any petition for certification
election. The employer’s participation in such
proceedings shall be limited to: (1) being notified or
A 40 Valid Election → 50% + 1 of BU
informed of petitions of such nature; and (2)
(40+30+20+80+30) = 101
B 30 Majority of Valid Votes Cast → submitting the list of employees during the pre--
50% + 1 of VVC (40+30+20+80) = election conference should the MedArbiter act
C 20 86 favorably on the petition.

X 80 X union did not get majority votes, NOTE: Commingling is not a ground for cancellation
union so run-off. If X union got 86, then no
as provided by RA 6715.
run-off.
SPOIL 30
ED For Run-Off to Apply→ 50% + 1 St. James School of Quezon City v.
of 200 = 101 BUT 40+30+20 = 90
SO NO RUN-OFF
Samahang Manggagawa sa St. James,
November 23, 2005
The members of Samahang Manggagawa are
Re-run Elections
employees in the Tandang Sora campus. Under its
- Election conducted to break a tie between constitution and by-laws, Samahang Manggagawa
contending unions (including “no union”) seeks to represent the motor pool, construction and
- Also refers to an election conducted after a transportation employees of the Tandang Sora
failure of election has been declared by the campus. Thus, the computation of the quorum should
election officer and/or affirmed by the be based on the rank and file motor pool,
mediator-arbiter (conducted within 6 months construction and transportation employees of the
after failure of elections) Tandang Sora campus and not on all the employees
in St. James’ five campuses. x x x The motor pool,
construction and transportation employees of the
A 50 1. First, Run-off A, B, C
2. Second, if there is still a tie, Tandang Sora campus had 149 qualified voters at the
B 50 A and B time of the certification election. Hence, the 149
qualified voters should be used to determine the
C 60 existence of a quorum. Since a majority or 84 out of
the 149 qualified voters cast their votes, a quorum
X 5 existed in the certification election.
union

Coastal Subic Bay Terminal v. DOLE,


Republic of the Philippines, represented November 20, 2006
by DOLE, v. Kawashima Textile, July 23, The petitioner contends that applying by analogy, the
2008 doctrine of piercing the veil of corporate fiction,
KFWU filed PCE to be conducted in the bargaining APOSTEU and ALU are the same federation. Private
unit composed of rank-and-file employees of respondents disagree. First, as earlier discoursed,
respondent. Attached to its petition are a Certificate once a labor union attains the status of a legitimate
of Creation of Local/Chapter stating that KFWU labor organization, it continues as such until its
submitted a Charter Certificate issued to it by the certificate of registration is cancelled or revoked in an
national federation PH Transport & General Workers independent action for cancellation. In addition, the
Organization (PTGWO). Kawashima filed a petition on legal personality of a labor organization cannot be
the ground that KFWU did not acquire any legal collaterally attacked. Thus, when the personality of
personality because its membership of mixed rank- the labor organization is questioned in the same
and-file and supervisory employees violated Article manner the veil of corporate fiction is pierced, the
245 of the Labor Code, and its failure to submit its action partakes the nature of a collateral attack.
books of account. Hence, in the absence of any independent action for
cancellation of registration against either APSOTEU or

28
ALU, and unless and until their registrations are Having been misled, a majority of them eventually
cancelled, each continues to possess a separate legal disaffiliated themselves from it and formed an
personality. The CSBTI-RFU and CSBTI-SU are independent union, respondent herein, which
therefore affiliated with distinct and separate thereafter protested the conduct of the election.
federations, despite the commonalities of APSOTEU Having been formed just after such exercise by the
and ALU. defrauded employees who were former members of
petitioner, respondent could not have reasonably
Under the rules implementing the Labor Code, a filed its protest within five days from the close of the
chartered local union acquires legal personality election proceedings. The circumstances in the
through the charter certificate issued by a duly present case show that the employees did not sleep
registered federation or national union, and reported on their rights. Hence, their failure to follow strictly
to the Regional Office in accordance with the rules the procedural technicalities regarding the period for
implementing the Labor Code. A local union does not filing their protest should not be taken against them.
owe its existence to the federation with which it is Mere technicalities should not be allowed to prevail
affiliated. It is a separate and distinct voluntary over the welfare of the workers. What is essential is
association owing its creation to the will of its that they be accorded an opportunity to determine
members. Mere affiliation does not divest the local freely and intelligently which labor organization shall
union of its own personality, neither does it give the act on their behalf. Having been denied this
mother federation the license to act independently of opportunity by the betrayal committed by petitioner’s
the local union. It only gives rise to a contract of officers in the present case, the employees were
agency, where the former acts in representation of prevented from making an intelligent and
the latter. Hence, local unions are considered independent choice.
principals while the federation is deemed to be
merely their agent. As such principals, the unions are
Sugbuanon Rural Bank, Inc. v
entitled to exercise the rights and privileges of a
legitimate labor organization, including the right to Laguesma, February 2, 2000
seek certification as the sole and exclusive bargaining One of the rights of a legitimate labor organization
agent in the appropriate employer unit. under Article 242(b) of the Labor Code is the right to
be certified as the exclusive representative of all
DHL Phils. United Rank and File employees in an appropriate bargaining unit for
purposes of collective bargaining. Having complied
Association v. Buklod ng Manggagawa with the requirements of Art. 234, it is our view that
ng DHL Phils., July 22, 2004 respondent union is a legitimate labor union. Article
Petitioner argues that the CA gravely erred in 257 of the Labor Code mandates that a certification
rendering that the withdrawal of a great majority of election shall automatically be conducted by the
the members of petitioner provided a compelling MedArbiter upon the filing of a petition by a
reason to conduct a certification election anew in legitimate labor organization. Nothing is said therein
order to determine, once and for all, which union that prohibits such automatic conduct of the
reflected their choice, considering that no protest or certification election if the management appeals on
challenge had been formalized within five days, or the issue of the validity of the union’s registration. On
raised during the election proceedings and entered in this score, petitioner’s appeal was correctly
the minutes thereof. Petitioner adds that respondent dismissed.
did not file any protest, either, against the alleged
fraud and misrepresentation by the former’s officers Sta. Lucia East Commercial Corporation
during the election. We disagree. When the
medarbiter admitted and gave due course to v. Hon. Secretary of Labor, August 14,
respondent’s Petition for nullification of the election 2009
proceedings, the election officer should have deferred CLUP instituted a petition for certification election
issuing the Certification of the results thereof. Section among rank and file employees of SLECC and its
13 of the Implementing Rules cannot strictly be affiliates. This was dismissed by the Med-Arbiter, thus
applied to the present case. the union re-organized as CLUP-SLECCWA limiting
their membership to rank and file employees of
Respondent’s contention is that a number of SLECC only. The new union then filed with the DOLE a
employees were lured by their officers into believing petition for certification election. A motion to dismiss
that petitioner was an independent union. Since the was filed by SLECC alleging that it has voluntarily
employees had long desired to have an independent recognized SMSLEC as exclusive bargaining agent of
union that would represent them in collective its rank and file employees and that collective
bargaining, they voted “yes” in favor of petitioner. bargaining negotiations already commenced between
29
them. The Med-Arbiter dismissed the petition, which was intially dismissed claiming that there was not
was however reversed by the SOLE and later affirmed employer-employee relationship between the parties
by the CA. as admitted by the union that the members are
agency employees. When the union filed a second
The SC said that the union’s certification should not petition for certification election, it was dismissed
be cancelled for representing the wrong bargaining stating that it was barred by prior judgment. On its
unit. The union initially had a problem because they third petition for certification election, the SOLE
are a legitimate labor organization but represented a granted the petition subject to the usual pre-election
non-appropriate bargaining unit. When the union conference. When the case eventually reached the
subsequently re-registered limiting its members to SC, the SC held that the case was not barred by res
rank and file employees of SLECC, SLECC cannot now judicata. It ruled that there was no identity of parties,
ignore that the union was a legitimate labor subject matter, and causes of action. In this case, the
organization. Representing the wrong bargaining unit SOLE dismissed the first petition as it was filed
is not a ground for cancellation unless it was outside the 60 day freedom period. During that time,
attended by misrepresentation, false statement, or the union has no cause of action since they are not
fraud. The Court further held that the recognition of yet legally allowed to challenge openly and formally
SMSLEC by SLECC was not valid. An employer may the status of SMCGC-SUPER. The dismissal has not
voluntarily recognize the representation status of a bearing in the case since the third petition was filed
union in an unorganized establishments. However, within the 60 day freedom period.
SLECC is not an unorganized establishment when it
voluntarily recognized SMSLEC as its exclusive [SUPRA] National Union of Workers in
bargaining representative.
Hotels, Restaurants and Allied
NOTE: Voluntary recognition and SEBA certification is Industries- Manila Pavilion Hotel Chapter
only allowed in unorganized unions. v. Secretary of Labor, July 31, 2009
Under the so-called “double majority rule” for there
[SUPRA] Samma–Likha v. Samma to be a valid certification election, majority of the
Corporation, March 13, 2009 bargaining unit must have voted and the winning
The erroneous inclusion of one supervisory employee union must have garnered majority of the valid votes
in the union is not a ground to impugn the legitimacy cast; Majority is 50% + 1.—As to whether HIMPHLU
of a legitimate labor organization which had the right should be certified as the exclusive bargaining agent,
to file a petition for certification election. With the the Court rules in the negative. It is well- settled that
certificates of registration issued in favor of under the so-called “double majority rule,” for there
petitioner, they are clothed with legal personality as a to be a valid certification election, majority of the
legitimate labor organizations. Moreover, such legal bargaining unit must have voted AND the winning
personality cannot thereafter be subject to collateral union must have garnered majority of the valid votes
attack, but may be questioned only in an cast. From the Court’s ruling that all the probationary
independent petition for cancellation of certificates of employees’ votes should be deemed valid votes while
registration. Even though the DOLE initially revoked that of the supervisory employees should be
the petitioner’s charter certificate, petitioner has excluded, it follows that the number of valid votes
moved for its reconsideration of such resolution. cast would increase—from 321 to 337.

Art. 256 of the Labor Code provides that the union


Chris Garments Corporation v. Hon. obtaining the majority of the valid votes cast by the
Patricia A. Sto Tomas and Chris eligible voters shall be certified as the sole and
Garments Workers Union-PTGWO, exclusive bargaining agent of all the workers in the
appropriate bargaining unit. This majority is 50% + 1.
January 12, 2009 Hence, 50% of 337 is 168.5 + 1 or at least 170.
The union filed a petition for certification election Furthermore, period of reckoning in determining who
with the Med-Arbiter. The union sought to represent shall be included in the list of eligible voters is in
petitioner’s rank and file employees not covered by cases where a timely appeal has been filed from the
its CBA with SMCGC-SUPER. Petitioner moved to Order of the MedArbiter, the date when the Order of
dismiss the petition arguing that it has an existing the Secretary of Labor and Employment, whether
CBA with SMCGC-SUPER which bars any petition for affirming or denying the appeal, becomes final and
certification prior to the 60 day freedom period. It executory. The provision in the CBA disqualifying
also argued that the union members are not its probationary employees from voting cannot override
regular employees since they are direct employees of the Constitutionally- protected right of workers to self
qualified and independent contractors. The petition organization, as well as the provisions of the Labor
30
Code and its Implementing Rules on certification questioned only in an independent petition for
elections and jurisprudence. cancellation based on Section 5, Rule V of the
Implementing Rules of Book V.
[SUPRA] Eagle Ridge Golf and Country
Club v. CA, March 18, 2010 Bars to Certification Election
hen the Union applied for registration, there were 30
employees so it complied with the 20% membership NOTE: Please read Omnibus Rules, Book V, Rule VIII,
requirement under Article 234(c). 20% of 112 rank Sections 14-15, Rule XVII, Section 7 as amended by
and file employees in Eagle Ridge would require a D.O. 40-03
union membership of at least 22 employees, thus
Article 238, Labor Code. Prohibition on
when the certification of registration was granted, the
Certification Election. - The Bureau shall not
Union had complied with said requirement. entertain any petition for certification election or
Furthermore, the admission of new members is any other action which may disturb the
neither prohibited by law nor was it concealed in its administration of duly registered existing collective
application for registration. bargaining agreements affecting the parties except
under Articles 253, 253-A, and 256 of this Code.
(As amended by Sec. 15, Rep. Act. No. 6715, March
[SUPRA] PICOP Resources, Inc. v. 21, 1989).
Tañeca, August 9, 2010
At the expiration of the freedom period (60 days from
WHAT ARE THE BARS?
the termination of CBA), the employer shall continue
to recognize the majority status of the incumbent 1. CONTRACT BAR RULE - provides that while a
bargaining agent where no petition for certification valid and registered CBA is subsisting, the
election is filed. While it is incumbent for the Bureau is not allowed to hold an election
employer to continue to recognize the majority status contesting the majority status of the
of the incumbent bargaining agent even after the incumbent union.
expiration of the freedom period, they could only do 2. CERTIFICATION YEAR BAR RULE OR ELECTION
so when no petition for certification election was filed. BAR RULE - provides that no petition for a CE
representative.The provision for status quo is may be filed within one year from the date of
conditioned on the fact that no certification election a valid certification, consent, or run-off
was filed during the freedom period. In the instant election. Thus if an election had been held
case, a petition for certification election was already but not one of the unions won, a PCE may be
ordered by the Med Arbiter so that at the expiration filed again but only after 12 mos. The same
of the freedom period, PRI’s obligation to recognize ban shall apply even if “NO UNION” won in
NAMAPRI-APFL as the incumbent bargaining agent the previous election. The purpose of this bar
does not hold true when petitions for certification is to give a chance to the union to conclude a
election were filed like this case. Moreover, the last CBA with the employer within 1-year. If the 1-
sentence of Article 253 which provides for automatic year lapsed and no CBA was concluded BUT
renewal pertains only to the economic provisions of without the fault of the union, then the bar
the CBA. When there is a representational issue, the still applies.
status quo provision in so far as the need to await the 3. DEADLOCK BAR RULE - that a PCE can only be
creation of a new agreement will not apply. entertained if there is no pending bargaining
deadlock submitted to conciliation or
[SUPRA]Legend International Resorts v. arbitration or had become the subject of a
valid notice of strike or lockout. The principal
Kilusang Manggagawa ng Legend, purpose is to ensure stability in the
February 23, 2011 relationship of the workers and the
An order to hold a certification election is proper management.
despite the pendency of the petition for cancellation
of the registration certificate of the respondent union.
The rationale for this is that at the time the
respondent union filed its petition, it still had the legal
Collective Bargaining
personality to perform such act absent an order NOTE: Please read Omnibus Rules, Book V, Rule I,
directing a cancellation. Thus, The cancellation of Section 1 (d, h, j, t, bbb), Rules XVI-XVII, as amended
KMLs certificate of registration should not retroact to by D.O. 40-03; and
the time of its issuance. The legitimacy of the legal Omnibus Rules, Book V, Rules XIX-XXI, as amended by
personality of KML cannot be collaterally attacked in D.O. 40-03;
a petition for certification election, but may be
31
Article 261, Labor Code. Procedure in collective notice to terminate or modify the agreement at
bargaining. - The following procedures shall be least sixty (60) days prior to its expiration date. It
observed in collective bargaining: shall be the duty of both parties to keep the status
quo and to continue in full force and effect the
(a) When a party desires to negotiate an terms and conditions of the existing agreement
agreement, it shall serve a written notice upon the during the 60-day period and/or until a new
other party with a statement of its proposals. The agreement is reached by the parties.
other party shall make a reply thereto not later
than ten (10) calendar days from receipt of such
notice; Article 265, Labor Code. Terms of a collective
(b) Should differences arise on the basis of such bargaining agreement. - Any Collective Bargaining
notice and reply, either party may request for a Agreement that the parties may enter into shall,
conference which shall begin not later than ten (10) insofar as the representation aspect is concerned,
calendar days from the date of request. be for a term of five (5) years. No petition
(c) If the dispute is not settled, the Board shall questioning the majority status of the incumbent
intervene upon request of either or both parties or bargaining agent shall be entertained and no
at its own initiative and immediately call the certification election shall be conducted by the
parties to conciliation meetings. The Board shall Department of Labor and Employment outside of
have the power to issue subpoenas requiring the the sixty-day period immediately before the date of
attendance of the parties to such meetings. It shall expiry of such five-year term of the Collective
be the duty of the parties to participate fully and Bargaining Agreement. All other provisions of the
promptly in the conciliation meetings the Board Collective Bargaining Agreement shall be
may call; renegotiated not later than three (3) years after its
(d) During the conciliation proceedings in the execution.
Board, the parties are prohibited from doing any
act which may disrupt or impede the early Any agreement on such other provisions of the
settlement of the disputes; and Collective Bargaining Agreement entered into
(e) The Board shall exert all efforts to settle within six (6) months from the date of expiry of the
disputes amicably and encourage the parties to term of such other provisions as fixed in such
submit their case to a voluntary arbitrator. Collective Bargaining Agreement, shall retroact to
the day immediately following such date. If any
such agreement is entered into beyond six months,
Article 262, Labor Code. Duty to bargain the parties shall agree on the duration of
collectively in the absence of collective bargaining retroactivity thereof. In case of a deadlock in the
agreements. - In the absence of an agreement or renegotiation of the Collective Bargaining
other voluntary arrangement providing for a more Agreement, the parties may exercise their rights
expeditious manner of collective bargaining, it shall under this Code.
be the duty of employer and the representatives of
the employees to bargain collectively in
accordance with the provisions of this Code. Article 266, Labor Code. Injunction prohibited. -
No temporary or permanent injunction or
restraining order in any case involving or growing
Article 263, Labor Code. Meaning of duty to out of labor disputes shall be issued by any court or
bargain collectively. - The duty to bargain other entity, except as otherwise provided in
collectively means the performance of a mutual Articles 218 and 264 of this Code.
obligation to meet and convene promptly and
expeditiously in good faith for the purpose of
negotiating an agreement with respect to wages, Article 237, Labor Code. Registry of unions and
hours of work and all other terms and conditions of file of collective bargaining agreements. - The
employment including proposals for adjusting any Bureau shall keep a registry of legitimate labor
grievances or questions arising under such organizations. The Bureau shall also maintain a file
agreement and executing a contract incorporating of all collective bargaining agreements and other
such agreements if requested by either party but related agreements and records of settlement of
such duty does not compel any party to agree to a labor disputes and copies of orders and decisions of
proposal or to make any concession. voluntary arbitrators. The file shall be open and
accessible to interested parties under conditions
prescribed by the Secretary of Labor and
Article 264, Labor Code. Duty to bargain Employment, provided that no specific information
collectively when there exists a collective submitted in confidence shall be disclosed unless
bargaining agreement. - When there is a collective authorized by the Secretary, or when it is at issue
bargaining agreement, the duty to bargain in any judicial litigation, or when public interest or
collectively shall also mean that neither party shall national security so requires.
terminate nor modify such agreement during its
lifetime. However, either party can serve a written Within thirty (30) days from the execution of a

32
Collective Bargaining Agreement, the parties shall
submit copies of the same directly to the Bureau or For this purpose, parties to a Collective Bargaining
the Regional Offices of the Department of Labor Agreement shall name and designate in advance a
and Employment for registration, accompanied Voluntary Arbitrator or panel of Voluntary
with verified proofs of its posting in two Arbitrators, or include in the agreement a
conspicuous places in the place of work and procedure for the selection of such Voluntary
ratification by the majority of all the workers in the Arbitrator or panel of Voluntary Arbitrators,
bargaining unit. The Bureau or Regional Offices preferably from the listing of qualified Voluntary
shall act upon the application for registration of Arbitrators duly accredited by the Board. In case
such Collective Bargaining Agreement within five the parties fail to select a Voluntary Arbitrator or
(5) calendar days from receipt thereof. The panel of Voluntary Arbitrators, the Board shall
Regional Offices shall furnish the Bureau with a designate the Voluntary Arbitrator or panel of
copy of the Collective Bargaining Agreement within Voluntary Arbitrators, as may be necessary,
five (5) days from its submission. pursuant to the selection procedure agreed upon in
the Collective Bargaining Agreement, which shall
The Bureau or Regional Office shall assess the act with the same force and effect as if the
employer for every Collective Bargaining Arbitrator or panel of Arbitrators has been selected
Agreement a registration fee of not less than one by the parties as described above.
thousand pesos (P1,000.00) or in any other amount
as may be deemed appropriate and necessary by
the Secretary of Labor and Employment for the Article 274, Labor Code. Jurisdiction of
effective and efficient administration of the Voluntary Arbitrators or panel of Voluntary
Voluntary Arbitration Program. Any amount Arbitrators. - The Voluntary Arbitrator or panel of
collected under this provision shall accrue to the Voluntary Arbitrators shall have original and
Special Voluntary Arbitration Fund. exclusive jurisdiction to hear and decide all
unresolved grievances arising from the
The Bureau shall also maintain a file and shall interpretation or implementation of the Collective
undertake or assist in the publication of all final Bargaining Agreement and those arising from the
decisions, orders and awards of the Secretary of interpretation or enforcement of company
Labor and Employment, Regional Directors and the personnel policies referred to in the immediately
Commission. preceding article. Accordingly, violations of a
Collective Bargaining Agreement, except those
which are gross in character, shall no longer be
Article 219(n), Labor Code. Definitions - treated as unfair labor practice and shall be
(n) "Voluntary Arbitrator" means any person resolved as grievances under the Collective
accredited by the Board as such or any person Bargaining Agreement. For purposes of this article,
named or designated in the Collective Bargaining gross violations of Collective Bargaining Agreement
Agreement by the parties to act as their Voluntary shall mean flagrant and/or malicious refusal to
Arbitrator, or one chosen with or without the comply with the economic provisions of such
assistance of the National Conciliation and agreement.
Mediation Board, pursuant to a selection procedure
agreed upon in the Collective Bargaining The Commission, its Regional Offices and the
Agreement, or any official that may be authorized Regional Directors of the Department of Labor and
by the Secretary of Labor and Employment to act Employment shall not entertain disputes,
as Voluntary Arbitrator upon the written request grievances or matters under the exclusive and
and agreement of the parties to a labor dispute. original jurisdiction of the Voluntary Arbitrator or
panel of Voluntary Arbitrators and shall
immediately dispose and refer the same to the
Article 273, Labor Code. Grievance machinery Grievance Machinery or Voluntary Arbitration
and voluntary arbitration. - The parties to a provided in the Collective Bargaining Agreement.
Collective Bargaining Agreement shall include
therein provisions that will ensure the mutual
observance of its terms and conditions. They shall Article 275, Labor Code. Jurisdiction over other
establish a machinery for the adjustment and labor disputes. - The Voluntary Arbitrator or panel
resolution of grievances arising from the of Voluntary Arbitrators, upon agreement of the
interpretation or implementation of their Collective parties, shall also hear and decide all other labor
Bargaining Agreement and those arising from the disputes including unfair labor practices and
interpretation or enforcement of company bargaining deadlocks.
personnel policies.

All grievances submitted to the grievance Article 292, Labor Code. Miscellaneous
machinery which are not settled within seven (7) provisions. -
calendar days from the date of its submission shall (f) A special Voluntary Arbitration Fund is hereby
automatically be referred to voluntary arbitration established in the Board to subsidize the cost of voluntary
prescribed in the Collective Bargaining Agreement.

33
arbitration in cases involving the interpretation and PAL v. PALEA, March 12, 2008
implementation of the Collective Bargaining Agreement, PAL and PALEA entered into a CBA. The agreement
including the Arbitrator’s fees, and for such other provides for the 13th month bay and the Christmas
related purposes to promote and develop voluntary bonus of rank and file employees. Prior to the
arbitration. The Board shall administer the Special
payment, PAL released a guideline to which it
Voluntary Arbitration Fund in accordance with the
guidelines it may adopt upon the recommendation of the provides the eligibility of an employee for the benefit.
Council, which guidelines shall be subject to the approval However, PALEA assailed the implementation of the
of the Secretary of Labor and Employment. Continuing guideline. It was alleged by PALEA that all employees,
funds needed for this purpose in the initial yearly amount regular as well as non-regular must be paid their 13th
of fifteen million pesos (P15,000,000.00) shall be provided month pay. PALEA filed a labor complaint for unfair
in the 1989 annual general appropriations acts.
labor practice. The complaint interposed that cut-off
The amount of subsidy in appropriate cases shall period for regularization should not be used as
be determined by the Board in accordance with parameter for the granting of 13th month pay. The SC
established guidelines issued by it upon the held that the CBA between the parties clearly reveals
recommendation of the Council. that Art. 1, Section 3 of the said agreement made its
provisions applicable to all employees, without
The Fund shall also be utilized for the operation of distinguishing whether regular or not, in the
the Council, the training and education of Voluntary
bargaining unit. The CBA extends to the laborers and
Arbitrators, and the Voluntary Arbitration Program.
employees in the collective bargaining agreement,
(g) The Ministry shall help promote and gradually including those who do not belong to the chosen
develop, with the agreement of labor organizations bargaining labor organization. Otherwise there would
and employers, labor-management cooperation be discrimination. Hence, the benefits of the CBA
programs at appropriate levels of the enterprise should be given to all employees who are members of
based on the shared responsibility and mutual the bargaining unit, not necessarily of the labor
respect in order to ensure industrial peace and
organization designated as the bargaining unit.
improvement in productivity, working conditions
and the quality of working life.
San Miguel Foods v. San Miguel
(h) In establishments where no legitimate labor
organization exists, labor-management committees Corporation Employees Union, October
may be formed voluntarily by workers and 5, 2007
employers for the purpose of promoting industrial Union filed a case for ULP against the Company for
peace. The Department of Labor and Employment
violating the CBA, specifically in violating the Job
shall endeavor to enlighten and educate the
workers and employers on their rights and Security provision by committing wanton
responsibilities through labor education with discrimination in matters of promotion within the
emphasis on the policy thrusts of this Code. FInance Department and in violating the Grievance
Machinery provision by not immediately acting upon
the complaint the union filed. The Company
Union of Filipro Employees v. Nestle questions the jurisdiction of the LA or the NLRC in
Phils., March 3, 2008 hearing the case, reasoning that the issues raised in
The Union and Nestle negotiated for a new CBA but the case are grievance issues. The Court ruled in
failed to reach agreement after 15 meetings. The favor of the Union, and held that the LA/NLRC can
Union argues that Nestle’s refusal to bargain on a hear this case. It held that for a ULP case to be
very important CBA economic provision constitutes cognizable by the LA, and the NLRC to exercise its
unfair labor practice. The Court held that the purpose appellate jurisdiction, the allegations in the complaint
of collective bargaining is the reaching of an should show prima facie the concurrence of 2 things,
agreement resulting in a contract binding on the namely:
parties; but the failure to reach an agreement after (1) gross violation of the CBA; AND
negotiations have continued for a reasonable period (2) the violation pertains to the economic
does not establish a lack of good faith. The duty to provisions of the CBA.
bargain does not include the obligation to reach an
agreement. Here, Nestle never refused to bargain In this case, the Union charges the Company to have
collectively with the union. The corporation simply violated the grievance machinery provision in the
wanted to exclude the Retirement Plan from the CBA. The grievance machinery provision in the CBA is
issues to be taken up during CBA negotiations, on the not an economic provision, however, hence, the
postulation that such was in the nature of a second requirement for a LA to exercise jurisdiction of
unilaterally granted benefit. a ULP is not present. However, the Union likewise
charges the Company to have violated the Job
Security provision in the CBA, specifically the
34
seniority rule. Since the seniority rule in the members on account of religious affiliation. The
promotion of employees has a bearing on salary and company said there is no basis for negotiation
benefits, it may, following a liberal construction of because the union lost its legal personality on
Article 261 of the Labor Code, be considered an account of disaffiliation or resignation of union
economic provision of the CBA. This charge is a gross members. The SC said that the company should have
or flagrant violation of the seniority rule under the entered into collective bargaining with the union. The
CBA. And so, it is a ULP over which the LA has law mandates that the representation provisions of
jurisdiction. the CBA last for 5 years. The relationship between
labor and management should be undisturbed until
the last 60 days of the 5th year. When the union
Capitol Medical Center v. Trajano, June requested for a renegotiation of the economic terms
30, 2005 of the CBA, it was still the certified collective
Capital Medical Center argues that its petition for bargaining agent of the workers since it was seeking
cancellation of the Union’s certificate of registration renegotiation within 5 years from the date of
involved a prejudicial question to be settled first effectivity of the CBA. The union’s proposal was
before the Secretary of Labor could order the parties submitted within the prescribed 3 year period from
to bargain collectively. The Court held in the negative. the date of effectivity of the CBA. GMC has no valid
Pendency of a petition for cancellation does not reason to refuse to negotiate in good faith with the
preclude collective bargaining. Unless the certificate union. For refusing to send a counter-proposal to the
of registration and its status as the certified union and to bargain anew on the economic terms of
bargaining agent are revoked, Capitol Medical Center the CBA, the company committed an unfair labor
is duty bound to collectively bargain with the Union. practice. MC’s failure to make a timely reply to the
Reason for this ruling: the Union still has legal proposals presented by the union is indicative of its
personality. utter lack of interest in bargaining with the union.

Standard Chartered Bank Employees Halagueña, et al., and other flight


Union v. Confesor, June 16, 2004 attendants of Philippine Airlines v.
Before the commencement of negotiations, the Union Philippine • Airlines, October 2, 2009
suggested that the bank lawyers should be excluded Female flight attendants, employed under PAL,
from the negotiating team. The Bank agreed but also argued that the provision on compulsory retirement
suggested that the President of the Union be for females is discriminatory (55 y/o: Females; 60 y/o:
excluded from the Union’s negotiating panel. Males). They filed a special civil action for declaratory
However, the President was retained as member. relief with the RTC to question the constitutionality of
Both parties failed to agree on some economic SEc. 144, Part A of the CBA. RTC held that it has
provisions, thus, the Union declared a deadlock. The jurisdiction over the case; however, the CA reversed
Bank filed a case of ULP against the Union. However, it. Thus, Halaguena, et. al. filed this case before SC,
the Union argues that the company’s interference in arguing that the RTC has jurisdiction over this case.
the choice of the bargaining panel is tantamount to SC held that the RTC has jurisdiction because the
ULP. Court held that it is NOT ULP because it does not issue of constitutionality of the CBA provision is
show that the suggestion to exclude the President incapable of pecuniary estimation - exclusively
was an anti-union conduct. cognizable by the RTC. Also, this dispute is between
PAL and the female flight attendants, not the Union.
ULP is committed when the ER interferes in the Thus, referral to the grievance machinery and
selection of its negotiators or coerces the Union to voluntary arbitration would not serve the best
exclude from its panel of negotiators a representative interest of Halaguena, et. al.
of the Union, and if it can be inferred that the ER
adopted the said act to yield adverse effects on the
free exercise to right to self-organization or on the PASSI v. Boclot, September 28, 2007
right to collective bargaining of the EEs. Boclot claims that he has attained the status of a
regular employee pursuant to the CBA provision
which states that the company agrees to convert to
General Milling Corporation v. CA, regular status all incumbent probationary or casual
February 11, 2004 employees and workers who have served the
The GMC and the union executed a CBA. A day before company for an accumulated service term of
the expiration of the CBA, the union sent a proposal. employment of not less than 6 months. Court held
A counter-proposal was not given by the company that he has attained regular status based on the CBA,
claiming that it receive withdrawals from union and not on the Labor Code.

35
Thus, are regular rank-and-file employees, they fall
FVCLUPTGWO v. Sama-Samang within the CBA coverage under the CBA’s express
terms, and are entitled to its benefits.
Nagkakaisang Manggagawa Sa FVC-
Solidarity Of Independent And General Employees Union of Bayer v. Bayer
Labor Organizations (SANAMA-FVC- Phils., December 6, 2010
SIGLO), November 27, 2009 A group of employees dissented from Union 1 which
At the end of the 3rd year of the CBA, FVC (the union) was the EBR of Bayer, and made their own union
and the company re-negotiated and agreed to extend (Union 2) with the support of majority of Union 1’s
the duration of the CBA to 4 more months: original is members. This resulted to a conflict between Union 1
Jan. 30, 2003 and now it is May 31, 2003. Then, and Union 2. Bayer, then, became confused as to
another Union, Sanama, filed a petition for which union they should remit the union dues. In the
certification election with the DOLE 9 days before Jan. end, they remitted this to Union 2. Bayer also signed
30, 2003. FVC argues that its exclusive bargaining a new CBA with Union 2, despite the existence of
representation status should be in accord with the their current CBA with Union 1. And so, Union 1 filed a
term of its CBA and that such status can only be case for ULP against Bayer. The Court held that Bayer
challenged within 60 days before the expiration of is guilty of ULP. It must be remembered that a CBA is
said term. Court held that even if there is an entered into in order to foster stability and mutual
agreement, the exclusive bargaining status if cooperation between labor and capital. An employer
effective for only 5 years. Although said amendment should not be allowed to rescind unilaterally its CBA
in the CBA is legal, it will have no effect on FVC’s with the duly certified bargaining agent it had
representation status which will be only up to Jan. 30, previously contracted with, and decide to bargain
2003. Representation status is a legal matter not for anew with a different group if there is no legitimate
the workplace parties to agree upon. reason for doing so and without first following the
proper procedure. If such behavior would be
RFM Corporation v. KAMPI-NAFLU-KMU, tolerated, bargaining and negotiations between the
employer and the union will never be truthful and
February 4, 2009 meaningful, and no CBA forged after arduous
Sec. 3 of their CBA provides that the company agrees negotiations will ever be honored or be relied upon.
to make payment to all daily paid employees, in
respect of any of the days enumerated hereunto if The Court also defined an intra-union dispute.
declared as special holidays by the national Accordingly, this refers to any conflict between and
government. Now, during the 1st year of the among union members, including grievances arising
effectivity of the CBA, Dec. 31, which fell on a Sunday from any violation of the rights and conditions of
(rest day) was declared by the national government membership, violation of or disagreement over any
as a special holiday. The Union claimed payment provision of the union's constitution and by-laws, or
invoking Sec. 3 but RFM refused, saying that Dec. 31 disputes arising from chartering or disaffiliation of the
was NOT compensable since it was a rest day. union. The case against Bayer by Union 1 is not an
intra-union dispute. It is a ULP, so the LA has
Court held that if the terms of a CBA are clear and jurisdiction over the same. However, the case against
have no doubt upon the intention of the contracting Union 2 is an intra-union dispute; and was correctly
parties, as in this case, the literal meaning thereof dismissed for lack of jurisdiction.
shall prevail. Thus, as daily-paid employees must be
paid their regular salaries on the holidays which are The Court also ruled that, despite the issue being
so declared by the national government, regardless of moot and academic since the Company later on
whether they fall on rest days. The CBA is the law recognized Union 1 as the EBR of its R&F employees,
between the parties, hence, they are obliged to the complaint should still not be dismissed. A
comply with its provisions. legitimate labor organization cannot be construed to
have abandoned its pending claim against the
[SUPRA] Fulache v. ABS-CBN, GR No. employer by returning to the negotiating table to
183810, January 21, 2010 fulfill its duty to represent the interest of its
Petitioners claim to be regular employees of ABS CBN members, except when the pending claim has been
and, thus, are entitled to the CBA benefits. The CBA expressly waived or compromised in its subsequent
expressly provides that regular rank-and-file negotiations with the management. Also, Bayer’s
employees are its coverage. Court held that they are later action did not obliterate the fact that the
regular employees as their functions are necessary management of Bayer had withdrawn its recognition
and indispensable in the business of the company. of Union 1 and supported Union 2.

36
disposed of by the LA by referring the same to the
General Milling Corp. Independent Labor grievance machinery and voluntary arbitration as
may be provided in said agreements. Thus, the LA
Union v. General Milling, June 15, 2011 should have referred the matter to the grievance
The CA here imposed to the employer the proposed machinery provided in the CBA.
CBA of the union, stating that the provision of such
agreement will take effect in the remaining two years
of the old CBA (to expire in 1993). Thereafter the
Insular Hotel Employees Union v.
union asked for a computation of their benefits before Waterfront Insular Hotel, September 22,
the Executive LA. The union questioned the exclusion 2010
of beneficiaries made by the Executive LA, given that If the individual members of the Union have no
she excluded new employees hired after Nov. 30, authority to file the case, does the federation to
1993. Court ruled that the computation was proper, which the local union is affiliated have the standing to
stating that the covered employees should be limited do so? On this note, Coastal Subic Bay Terminal, Inc.
only to those who were employed by GMC. So those v. Department of Labor and Employment, 507 SCRA
hired after the expiration of the two year period 300 (2006), is enlightening, thus: x x x A local union
should be excluded from the computation, pursuant does not owe its existence to the federation with
to the ruling of the CA. The grievance procedure of which it is affiliated. It is a separate and distinct
the CBA also provided for those who are covered by voluntary association owing its creation to the will of
the CBA, which served as basis for the Executive LA its members. Mere affiliation does not divest the local
in its exclusion of beneficiaries. union of its own personality, neither does it give the
mother federation the license to act independently of
Malayan Employees Association v. the local union. It only gives rise to a contract of
Malayan Insurance Co., February 2, agency, where the former acts in representation of
the latter. Hence, local unions are considered
2010 principals while the federation is deemed to be
A provision in the CBA allows union officials to avail of merely their agent. x x x Based on the foregoing, this
union leaves with pay for 90 days per year for Court agrees with approval with the disquisition of
purposes of attending grievance meetings and other the CA when it ruled that NFL had no authority to file
union activities. The company issued a rule requiring the complaint in behalf of the individual employees.
prior notice and prior approval by the department
heads before members can avail. This was placed Petitioners have not been duly authorized to
without any objection from the union until an officer’s represent the union. In Atlas Farms v. NLRC: X x x
file for leave was disapproved. He still took his leave Pursuant to Art 260, the parties to a CBA shall name
but he was eventually suspended from work. Court or designate their respective representatives to the
held that this regulation was accepted by the union, grievance machinery and if the grievance is unsettled
and the rule on its face is not unreasonable, in that level, it shall automatically be referred to the
oppressive nor violative of the CBA terms. The union voluntary arbitrators designated in advance by
members have also willingly applied for approval as parties to a CBA.
the rule requires. The rule cannot be removed except
with the company’s consent, or by negotiation and The CBA recognizes that DIHFEU-NFL is the exclusive
express agreement in future CBAs. bargaining representative of all permanent
employees. The inclusion of the word "NFL" after the
Santuyo v. Remerco Garments, March name of the local union merely stresses that the local
union is NFL's affiliate. It does not, however, mean
22, 2010 that the local union cannot stand on its own. The
Petitioners were questioning the legality of RGMI’s local union owes its creation and continued existence
adoption of new salary scheme. The labor arbiter to the will of its members and not to the federation to
assumed jurisdiction over the case and rendered a which it belongs.
decision granting the claims of the union. RGMI
appealed to the NLRC which reversed the LA.
However, the CA reversed the NLRC and held that the
Cirtek Employees Labor Union v. Cirtek
Labor Arbiter had no jurisdiction over the complaint. Electronics, November 15, 2010
SC held that the LA did NOT have jurisdiction over the The Secretary of Labor (SoL) resolved the CBA
complaint. Under Art. 217 of the Labor Code, cases deadlock by awarding a wage increase of from P6.00
arising from the interpretation or implementation of to P10.00 per day effective Jan. 1, 2004 and from
CBAs and those arising from the interpretation or P9.00 to P15.00 per day effective Jan. 1, 2005 and
enforcement of company personnel policies shall be adopting all other benefits as embodied in the MOA.

37
SC held that the SoL may resolve all issues involved Memorandum which scheduled the employees’ VLs.
in the controversy including the award of wage The Court ruled that PNCC has the sole discretion to
increases and benefits, pursuant to Art. 263(g), LC. schedule the vacation leave of the Union. If the terms
While an arbitral award cannot per se be categorized of a CBA are clear and leave no doubt upon the
as an agreement voluntarily entered into by the intention of the contracting parties, the literal
parties because it requires the intervention and meaning of its stipulation shall prevail. In fine, the
imposing power of the State thru the SoL when he CBA must be strictly adhered to and respected if its
assumes jurisdiction, the arbitral award can be ends have to be achieved, being the law between the
considered an approximation of a CBA which would parties. The rule of construction, that labor contracts
otherwise have been entered into by the parties, should be construed in favor of the laborer, does not
hence, it has the force and effect of a valid contract benefit the Union because, as stated, there is no
obligation. room for interpretation.

Eastern Telecoms v. Eastern Telecoms However, the Court ruled that PNCC has to pay for
the costs of their employees’ renewal of their
Employees Union, February 8, 2012 Security Licenses. This is because, while the CBA
Respondent union has a CBA with petitioner states that the expenses for the renewal of licenses
corporation. The agreement is about to expire in must be on the personal account of the employee,
2004, with a side agreement signed on 2001. The the 1994 Revised Rules and Regulations
company deferred the payment of the 14th, 15th, Implementing R.A. No. 5487 provides that this is the
and 16th month bonuses because of the deteriorating primary responsibility of the company. As such, PSC
financial status. The deferment was opposed by the should be liable for the same. Although it is a rule
union claiming that bonuses are mandated in the side that a contract freely entered into between the
agreement and that the grant of bonuses has ripened parties should be respected, since a contract is the
into company practice. On the other hand, the law between the parties, there are, however, certain
company held that the grant of bonus is conditional exceptions to the rule, specifically Art. 1306 of the
and based on the successful business performance Civil Code, which provides: The contracting parties
and the availability of company profits. The SC held may establish such stipulations, clauses, terms and
that the company is liable. A reading of the CBA side conditions as they may deem convenient, provided
agreement reveals that the giving of the bonuses they are not contrary to law, morals, good customs,
were without qualification. The said provision does public order, or public policy.
not state that the subject bonuses shall be made to
depend on the company’s financial standing or the
payment was contingent upon the realization of
Supreme Steel v. Nagkakaisang
profits. By virtue of its incorporation in the CBA side Manggagawa sa Supreme, March 28,
agreements, the grant of the bonuses has become 2011
more than just an act of generosity on the part of the Union filed a notice of strike with the National
company but a contractual obligation it has Conciliation and Mediation Board against petitioner
undertaken. Considering that the company had been for alleged violations of their CBA which prohibited
continuously suffering huge losses from 2000 to contracting-out labor or the hiring of contractual
2002, its business losses in the year 2003 were not employees. Petitioner admitted that it hired
exactly unforeseen or unexpected. Thus, it cannot be temporary employees, due to increase of job orders
said that the difficulty in complying with its obligation from abroad, but claimed that the same was a
was manifestly beyond the contemplation of the management prerogative
parties.
It is a familiar and fundamental doctrine in labor law
PNCC Skyway Traffic Management & that the CBA is the law between the parties and
Security Division Workers Organization compliance therewith is mandated by the express
policy of the law. If the terms of a CBA are clear and
v. PNCC Skyway Corp., February 17, there is no doubt as to the intention of the
2010 contracting parties, the literal meaning of its
In the CBA between PNCC and its Union, there was a stipulation shall prevail. Moreover, the CBA must be
provision on VL which states that the company would construed liberally rather than narrowly and
schedule the VLs of the employees, taking into technically and the Court must place a practical and
consideration their preference. There was also a realistic construction upon it. Any doubt in the
provision on their employees’ Security License, interpretation of any law or provision affecting labor
saying that the employees would responsible for the should be resolved in favor of labor. Jurisprudence
costs in renewing the same. Later, PNCC released a recognizes the right to exercise management
38
prerogative. Labor laws also discourage interference
with an employer’s judgment in the conduct of its Unfair Labor Practices
business. For this reason, the Court often declines to
interfere in legitimate business decisions of Article 258, Labor Code. Concept of unfair labor
employers. The law must protect not only the welfare practice and procedure for prosecution thereof. -
of employees, but also the right of employers. Unfair labor practices violate the constitutional
However, the exercise of management prerogative is right of workers and employees to self-
organization, are inimical to the legitimate interests
not unlimited. Managerial prerogatives are subject to
of both labor and management, including their
limitations provided by law, collective bargaining right to bargain collectively and otherwise deal with
agreements, and general principles of fair play and each other in an atmosphere of freedom and
justice. mutual respect, disrupt industrial peace and hinder
the promotion of healthy and stable labor-
management relations.
Wesleyan University v. Wesleyan
University Faculty and Staff Association, Consequently, unfair labor practices are not only
violations of the civil rights of both labor and
March 12, 2014 management but are also criminal offenses against
A memorandum providing guidelines on the the State which shall be subject to prosecution and
implementation of vacation and sick leave credits and punishment as herein provided.
vacation leave commutation was issued by Wesleyan
University. The respondent’s president wrote a letter Subject to the exercise by the President or by the
to the petitioner’s president informing him that it was Secretary of Labor and Employment of the powers vested
in them by Articles 263 and 264 of this Code, the civil
not amenable to the unilateral changes and that said
aspects of all cases involving unfair labor practices, which
guidelines were violative of of the CBA. Like any other may include claims for actual, moral, exemplary and other
contract, a CBA has the force of law between the forms of damages, attorney’s fees and other
parties and should be complied with in good faith. affirmative relief, shall be under the jurisdiction of the
Unilateral changes for suspensions in the limitation of Labor Arbiters. The Labor Arbiters shall give utmost priority
the provisions of the CBA cannot be allowed without to the hearing and resolution of all cases involving unfair
labor practices. They shall resolve such cases within thirty
the consent of both parties.
(30) calendar days from the time they are submitted for
decision.
Philippine Airlines v. Hassaram, June 5,
Recovery of civil liability in the administrative
2017 proceedings shall bar recovery under the Civil
Under the CBA, Hassaram may claim two separate Code.
plans: (a) the amount of P5,000 for every year of
service under the PAL-ALP AP Retirement Plan; and No criminal prosecution under this Title may be
(b) an equity equivalent to 240% of his gross monthly instituted without a final judgment finding that an
unfair labor practice was committed, having been
salary for every year of employment pursuant to the
first obtained in the preceding paragraph. During
Plan. In contrast, Article 287 would entitle a retiring the pendency of such administrative proceeding,
pilot to the equivalent of only 22.5 days of his the running of the period of prescription of the
monthly salary for every year of service. criminal offense herein penalized shall be
considered interrupted: Provided, however, that the
Art. 287 is applicable only to a situation where (l) final judgment in the administrative proceedings
shall not be binding in the criminal case nor be
there is no CBA or other applicable employment
considered as evidence of guilt but merely as proof
contract providing for retirement benefits for an of compliance of the requirements therein set forth.
employee, or (2) there is a CBA or other applicable
employment contract providing for retirement
benefits for an employee, but it is below the Article 259, Labor Code. Unfair labor practices
requirement set by law. The rationale for the first of employers. It shall be unlawful for an employer
situation is to prevent the absurd situation where an to commit any of the following unfair labor
employee, deserving to receive retirement benefits, is practice:
denied them through the nefarious scheme of
(a) To interfere with, restrain or coerce employees
employers to deprive employees of the benefits due
in the exercise of their right to self-organization;
them under existing labor laws. On the other hand, (b) To require as a condition of employment that a
the second situation aims to prevent private person or an employee shall not join a labor
contracts from derogating from the public law. organization or shall with-draw from one to which
he belongs;
(c) To contract out services or functions being
performed by union members when such will

39
interfere with, restrain or coerce employees in the collectively with the employer, provided it is the
exercise of their rights to self-organization; representative of the employees;
(d) To initiate, dominate, assist or otherwise (d) To cause or attempt to cause an employer to
interfere with the formation or administration of pay or deliver or agree to pay or deliver any money
any labor organization, including the giving of or other things of value, in the nature of an
financial or other support to it or its organizers or exaction, for services which are not performed or
supporters; not to be performed, including the demand for fee
(e) To discriminate in regard to wages, hours of for union negotiations;
work and other terms and conditions of (e) To ask for or accept negotiation or attorney’s fees
employment in order to encourage or discourage from employers as part of the settlement of any issue in
membership in any labor organization. Nothing in collective bargaining or any other dispute; or
this Code or in any other law shall stop the parties (f) To violate a collective bargaining agreement.
from requiring membership in a recognized The provisions of the preceding paragraph
collective bargaining agent as a condition for notwithstanding, only the officers, members of
employment, except those employees who are governing boards, representatives or agents or
already members of another union at the time of members of labor associations or organizations
the signing of the collective bargaining agreement. who have actually participated in, authorized or
Employees of an appropriate bargaining unit who ratified unfair labor practices shall be held
are not members of the recognized collective criminally liable.
bargaining agent may be assessed a reasonable
fee equivalent to the dues and other fees paid by
members of the recognized collective bargaining Article 274, Labor Code. Jurisdiction of Voluntary
agent, if such non-union members accept the Arbitrators or panel of Voluntary Arbitrators. - The
benefits under the collective bargaining Voluntary Arbitrator or panel of Voluntary
agreement: Provided, that the individual Arbitrators shall have original and exclusive
authorization required under Article 242, paragraph jurisdiction to hear and decide all unresolved
(o) of this Code shall not apply to the non-members grievances arising from the interpretation or
of the recognized collective bargaining agent; implementation of the Collective Bargaining
(f) To dismiss, discharge or otherwise prejudice or Agreement and those arising from the
discriminate against an employee for having given interpretation or enforcement of company
or being about to give testimony under this Code; personnel policies referred to in the immediately
(g) To violate the duty to bargain collectively as preceding article. Accordingly, violations of a
prescribed by this Code; Collective Bargaining Agreement, except those
(h) To pay negotiation or attorney’s fees to the which are gross in character, shall no longer be
union or its officers or agents as part of the treated as unfair labor practice and shall be
settlement of any issue in collective bargaining or resolved as grievances under the Collective
any other dispute; or Bargaining Agreement. For purposes of this article,
(i) To violate a collective bargaining agreement. gross violations of Collective Bargaining Agreement
shall mean flagrant and/or malicious refusal to
The provisions of the preceding paragraph comply with the economic provisions of such
notwithstanding, only the officers and agents of agreement.
corporations, associations or partnerships who
have actually participated in, authorized or ratified The Commission, its Regional Offices and the
unfair labor practices shall be held criminally liable. Regional Directors of the Department of Labor and
Employment shall not entertain disputes,
grievances or matters under the exclusive and
Article 260, Labor Code. Unfair labor practices original jurisdiction of the Voluntary Arbitrator or
of labor organizations. - It shall be unfair labor panel of Voluntary Arbitrators and shall
practice for a labor organization, its officers, agents immediately dispose and refer the same to the
or representatives: Grievance Machinery or Voluntary Arbitration
provided in the Collective Bargaining Agreement.
(a) To restrain or coerce employees in the exercise
of their right to self-organization. However, a labor
organization shall have the right to prescribe its [SUPRA] Employees Union of Bayer
own rules with respect to the acquisition or
retention of membership;
Phils. v. Bayer Phils., December 6, 2010
(b) To cause or attempt to cause an employer to Bayer is guilty of ULP. A CBA is entered into in order
discriminate against an employee, including to foster stability and mutual cooperation between
discrimination against an employee with respect to labor and capital. An employer should not be allowed
whom membership in such organization has been to rescind unilaterally its CBA with the duly certified
denied or to terminate an employee on any ground bargaining agent it had previously contracted with,
other than the usual terms and conditions under
and decide to bargain anew with a different group if
which membership or continuation of membership
is made available to other members; there is no legitimate reason for doing so and without
(c) To violate the duty, or refuse to bargain first following the proper procedure. If such behavior
would be tolerated, bargaining and negotiations
40
between the employer and the union will never be
truthful and meaningful, and no CBA forged after For a charge of unfair labor practice to prosper, it
arduous negotiations will ever be honored or be relied must be shown that the employer was motivated by
upon. ill-will, bad faith or fraud, or was oppressive to labor.
The employer must have acted in a manner contrary
Prince Transport v. Garcia, January 12, to morals, good customs, or public policy causing
social humiliation, wounded feelings or grave anxiety.
2011 While the law makes it an obligation for the employer
In order to block the continued formation of the and the employees to bargain collectively with each
union, PTI caused the transfer of all union members other, such compulsion does not include the
and sympathizers to one of its sub-companies, Lubas commitment to precipitately accept or agree to the
Transport (Lubas). Despite the transfer, the schedule proposals of the other. All it contemplates is that both
of bus drivers and conductors, their company IDs parties should approach the negotiation with an open
were issued by PTI. The daily time records, tickets mind and make reasonable effort to reach a common
and reports were also filed at the PTI office; and all ground of agreement.
claims for salaries were transacted thereto. Later, the
business of Lubas deteriorated because of the refusal The Union based its contention on the letter request
of PTI to maintain and repair the units being used - by MMC for the suspension of the collective
which resulted to the virtual stoppage of its bargaining negotiations until it resumes operations.
operations and respondents' loss of employment. Verily, it cannot be said that MMC deliberately
avoided the negotiation. It merely sought a
Respondents transfer of work assignments to Lubas suspension and in fact, even expressed its willingness
was designed by petitioners as a subterfuge to foil to negotiate once the mining operations resume.
the formers right to organize themselves into a union. There was valid reliance on the suspension of mining
Under Article 248 (a) and (e) of the Labor Code, an operations for the suspension, in turn, of the CBA
employer is guilty of unfair labor practice if it negotiation. The Union failed to prove bad faith in
interferes with, restrains or coerces its employees in MMC’s actuations.
the exercise of their right to self-organization or if it
discriminates in regard to wages, hours of work and
other terms and conditions of employment in order to
Central Azucarera de Bais Employees
encourage or discourage membership in any labor Union v. Central Azucarera de Bais, Nov.
organization. 17, 2010
For a charge of unfair labor practice to prosper, it
Indeed, evidence of petitioners' unfair labor practice must be shown that CAB was motivated by ill will,
is shown by the established fact that, after “bad faith, or fraud, or was oppressive to labor, or
respondents' transfer to Lubas, petitioners left them done in a manner contrary to morals, good customs,
high and dry insofar as the operations of Lubas was or public policy, and, of course, that social
concerned. The Court ruled that petitioners withheld humiliation, wounded feelings or grave anxiety
the necessary financial and logistic support such as resulted x x x” in suspending negotiations with
spare parts, and repair and maintenance of the CABEU-NFL. Notably, CAB believed that CABEU-NFL
transferred buses until only two units remained in was no longer the representative of the workers. It
running condition. This left respondents virtually just wanted to foster industrial peace by bowing to
jobless. the wishes of the overwhelming majority of its rank
and file workers and by negotiating and concluding in
Manila Mining Employees Corp. v. good faith a CBA with CABELA.” Such actions of CAB
Manila Mining, September 29, 2010 are nowhere tantamount to anti-unionism, the evil
MMC was compelled by the DENR to temporarily shut sought to be punished in cases of unfair labor
down its mining operations due to its failure to secure practices.
an Environmental Compliance Certificate, resulting in
the temporary lay-off of more than 400 employees in Furthermore, basic is the principle that good faith is
the mine site. presumed and he who alleges bad faith has the duty
to prove the same. By imputing bad faith to the
The lay-off is neither illegal nor can it be considered actuations of CAB, CABEU-NFL has the burden of
as unfair labor practice. Unfair labor practice cannot proof to present substantial evidence to support the
be imputed to MMC since, as ruled by the Court of allegation of unfair labor practice. Apparently,
Appeals, the call of MMC for a suspension of the CBA CABEU-NFL refers only to the circumstances
negotiations cannot be equated to “refusal to mentioned in the letter-response, namely, the
bargain.” execution of the supposed CBA between CAB and
41
CABELA and the request to suspend the negotiations, fault of the employee and without prejudice to the
to conclude that bad faith attended CAB’s actions. latter, resorted by management during periods of
The Court is of the view that CABEU-NFL, in simply business recession, industrial depression, or seasonal
relying on the said letter-response, failed to malfunctions or during lulls over shortage of
substantiate its claim of unfair labor practice to rebut materials. The prerogative of an employer to retrench
the presumption of good faith. its employees must be exercised only as a last resort,
considering that it will lead to the loss of the
Moreover, as correctly determined by the LA, the employee’s livelihood. The following are the
filing of the complaint for unfair labor practice was requirements for retrenchment:
premature inasmuch as the issue of collective (a) The retrenchment is reasonably necessary ad
bargaining is still pending before the NCMB. likely to prevent business losses and done in
good faith;
BPI Employees Union-Davao v. BPI, July (b) The employer served written notice to both to
the employees and the DOLE at least 1
24, 2013 month prior to the intended date of
In essence, the primordial issue in this case is retrenchment;
whether or not the act of BPI to outsource the (c) The employer pays the employees separation
cashiering, distribution and bookkeeping functions to pay;
BOMC is in conformity with the law and the existing (d) It is done in good faith for the advancement
CBA. Particularly in dispute is the validity of the of its interes and not to defeat or circumvent
transfer of twelve (12) former FEBTC employees to the employees’ right to security of tenure;
BOMC, instead of being absorbed in BPI after the and
corporate merger. The Union claims that a union shop (e) The employer used fair and reasonable
agreement is stipulated in the existing CBA. It is criteria in ascertaining who would be
unfair labor practice for employer to outsource the dismissed and who would be retained among
positions in the existing bargaining unit, citing the the employees.
case of Shell Oil Workers’ Union v. Shell Company of
the Philippines, Ltd.. The Union’s reliance on the Shell
Case is misplaced. The rule now is covered by Article
Royal Plant Workers Union v. Coca Cola
261 of the Labor Code. Article 261 provides Bottlers, April 15, 2013
Jurisdiction of Voluntary Arbitrators or panel of Union filed a case against Coke (company). They
Voluntary Arbitrators.—x x x Accordingly, violations of claimed that the company’s act of taking away the
a Collective Bargaining Agreement, except those chairs it has provided the employees since 1974 was
which are grossin character, shall no longer be invalid. The Court ruled that Coke’s act was valid. The
treated as unfair labor practice and shall be resolved chairs were not removed indiscriminately. The
as grievances under the Collective Bargaining removal of the chairs was compensated by a
Agreement. For purposes of this article, gross reduction of the operating hours and an increase of
violations of Collective Bargaining Agreement shall the break period. The decision to remove the chairs
mean flagrant and/or malicious refusal to comply with was done with good intentions as Coke wanted to
the economic provisions of such agreement. Clearly, avoid instances of operators sleeping on the job while
only gross violations of the economic provisions of in the performance of their duties and responsibilities
the CBA are treated as ULP. Otherwise, they are mere and because of the fact that the chairs were not
grievances. necessary considering that the operators constantly
move about while working. Hence, Coke’s exercise of
Pepsi Cola Products v. Molon et al., its management prerogative was made in good faith
without doing any harm to the workers’ rights. It did
February 18, 2013 not violate labor laws since there is no law that
Pepis adopted a company wide retrenchment requires employers to provide chairs for bottling
program were 6 elected officers and 29 active operators. It also did not violate their CBA. The CBA
members of ALU were dismissed. ALU filed a notice of between the Union and Coke contains no provision
strike alleging unfair labor practice. Eventually, a whatsoever requiring the management to provide
return to work order was issued. The Company and chairs for the operators in the
the union eventually executed an agreement wherein production/manufacturing line while performing their
the union will receive 100% separation pay and they duties and responsibilities. Also, their CBA expressly
were made to sign a quitclaim. Despite suh, provides that benefits and/or privileges, not expressly
complaints for illegal dismissal were still filed. The SC given therein but which are presently being granted
ruled that the retrenchment program was valid. by the company and enjoyed by the employees, shall
Retrenchment was defined as the termination of be considered as purely voluntary acts by the
employment initiated by the employer through no
42
management and that the continuance of such engaged in conduct which, it may reasonably be said,
benefits and/or privileges, no matter how long or how tends to interfere with the free exercise of
often, shall not be understood as establishing an employees’ rights; and that it is not necessary that
obligation on the company’s part. Since the matter of there be direct evidence that any employee was in
the chairs is not expressly stated in the CBA, it is fact intimidated or coerced by statements of threats
understood that it was a purely voluntary act on the of the employer if there is a reasonable inference that
part of Coke and the long practice did not convert it anti–union conduct of the employer does have an
into an obligation or a vested right in favor of the adverse effect on self–organization and collective
Union. bargaining.

Goya v. Goya Employees Union, January The questioned acts of petitioners: 1) sponsoring a
field trip to Zambales to the exclusion of union
21, 2013 members, before the election; 2) the active campaign
Goya hired contractual employees from PESO by the sales officer of petitioners against the union
Resources Development Corporation. This prompted prevailing as a bargaining agent during the field trip;
Union to request for a grievance conference on the 3) escorting its employees after the field trip to the
ground that the contractual workers do not belong to polling center; 4) the continuous hiring of
the categories of employees stipulated in their CBA subcontractors performing their functions; 5)
and contrary to the union security clause embodied in assigning union members to the site to work as grass
it. The Union argued that Goya is guilty of ULP for cutters; and 6) the enforcement of work on a
gross violation of the CBA. rotational basis for union members, all reek of
interference on the part of petitioners.
Goya’s act of contracting out/outsourcing is
within the purview of management prerogative. The various acts of petitioners reasonably support an
Both did not say, however, that such act is a valid inference that such were all orchestrated to restrict
exercise thereof. Obviously, this is due to the respondents’ free exercise of their right to self–
recognition that the CBA provisions agreed upon organization. Petitioners’ undisputed actions prior and
by Goya and the Union delimit the free exercise immediately before the scheduled certification
of management prerogative pertaining to the election, while seemingly innocuous, unduly meddled
hiring of contractual employees. A collective in the affairs of its employees in selecting their
bargaining agreement is the law between the parties. exclusive bargaining representative.
A collective bargaining agreement or CBA refers to
the negotiated contract between a legitimate labor
organization and the employer concerning wages,
Mendoza v. Officers of MWEU, January
hours of work and all other terms and conditions of 25, 2016
employment in a bargaining unit. As in all contracts, Mendoza was a member of the MWEU consisting of
the parties in a CBA may establish such stipulations, rank and file employees within Manila Water
clauses, terms and conditions as they may deem Company. MWEU informed Mendoza that the union
convenient provided these are not contrary to law, was unable to fully deduct the increased P200 union
morals, good customs, public order or public policy. dues from his salary due to lack of the required
Thus, where the CBA is clear and unambiguous, it check-off authorization from him and that his failure
becomes the law between the parties and compliance to pay the union dues would result in sanctions.
therewith is mandated by the express policy of the Consequently, MWEU scheduled an election of
law. officers and Mendoza filed his certificate of candidacy
for VP, but was disqualified for not being a member in
T&H Shopfitters v. T&H Shopfitters good standing on account of his suspension. Due to
his continued non-payment of union dues, he was
Workers Union, February 26, 2014 expelled from the union. During the freedom period,
ULP relates to the commission of acts that transgress Mendoza joined a new union, WATER-AFWC, wherein
the workers’ right to organize. As specified in now he was elected Union President. LA dismissed
Article 257 and now Article 258 of the Labor Code, Mendoza’s complaint for ULP (wherein he claims that
the prohibited acts must necessarily relate to the he was suspended and expelled by MWEU illegally as
workers’ right to self–organization. In the case of a result of the denial of his right to appeal his case to
Insular Life Assurance Co., Ltd. Employees the general membership assembly in accordance with
Association – NATU v. Insular Life Assurance Co. Ltd., the union’s constitution and by-laws) against MWEU
this Court had occasion to lay down the test of on the ground that the complaint covers intra-union
whether an employer has interfered with and coerced disputes which CA, LA, NLRC do not have jurisdiction,
employees in the exercise of their right to self–
organization, that is, whether the employer has
43
and that Mendoza failed to provide substantial
vicinity actually used by picketing strikers in
evidence on MWEU officers’ violations. moving to and fro before all points of entrance to
and exit from said establishment.
MWEU officers are guilty of ULP. While it is true that
some of the causes of action fall constitute intra-
union cases cognizable by the BLR, the charge of ULP Article 266, Labor Code. Injunction Prohibited.
falls within the exclusive jurisdiction of the LA based No temporary or permanent injunction or
on Article 217. MWEU’s Constitution and By-Laws are restraining order in any case involving or growing
clear that when Mendoza received the letter out of labor disputes shall be issued by any court or
other entity, except as otherwise provided in
informing him of the suspension, he immediately filed
Articles 218 and 264 of this Code.
a written appeal that went unheeded; thereafter,
when he was charged for the third time and expelled,
his timely appeal was still unacted upon. As the Article 278, Labor Code. Strikes, picketing and
officers did not act on his two appeals, he was thus lockouts. - (a) It is the policy of the State to
suspended and deprived of his right to run for the VP encourage free trade unionism and free collective
position and forced to join another union. bargaining.

(b) Workers shall have the right to engage in


concerted activities for purposes of collective
Strikes, Lockouts & bargaining or for their mutual benefit and
protection. The right of legitimate labor
Concerted Actions organizations to strike and picket and of employers
to lockout, consistent with the national interest,
shall continue to be recognized and respected.
Read Omnibus Rules, Book V, Rule XXII, as amended
However, no labor union may strike and no
by D.O. 40-03, and further amended by D.O. 40-A, employer may declare a lockout on grounds
D.O. 40-G-03 (2010), and D.O. 40-H-13 (2013) involving inter-union and intra-union disputes.
Article 219(o), Labor Code. (c) In case of bargaining deadlocks, the duly
(o) "Strike" means any temporary stoppage of work certified or recognized bargaining agent may file a
by the concerted action of employees as a result of notice of strike or the employer may file a notice of
an industrial or labor dispute. lockout with the Ministry at least 30 day before the
intended date thereof. In cases of unfair labor
practice, the period of notice shall be 15 days and
Article 219(p), Labor Code. in the absence of a duly certified or recognized
(p) "Lockout" means any temporary refusal of an bargaining agent, the notice of strike may be filed
employer to furnish work as a result of an industrial by any legitimate labor organization in behalf of its
or labor dispute. members. However, in case of dismissal from
employment of union officers duly elected in
accordance with the union constitution and by-
Article 219(q), Labor Code. laws, which may constitute union busting, where
(q) "Internal union dispute" includes all disputes or the existence of the union is threatened, the 15-
grievances arising from any violation of or day cooling-off period shall not apply and the union
disagreement over any provision of the constitution may take action immediately.
and by laws of a union, including any violation of
the rights and conditions of union membership (d) The notice must be in accordance with such
provided for in this Code. implementing rules and regulations as the Minister
of Labor and Employment may promulgate.

(e) During the cooling-off period, it shall be the


Article 219(r), Labor Code.
duty of the Ministry to exert all efforts at mediation
(r) "Strike-breaker" means any person who
and conciliation to effect a voluntary settlement.
obstructs, impedes, or interferes with by force,
Should the dispute remain unsettled until the lapse
violence, coercion, threats, or intimidation any
of the requisite number of days from the
peaceful picketing affecting wages, hours or
mandatory filing of the notice, the labor union may
conditions of work or in the exercise of the right of
strike or the employer may declare a lockout.
self-organization or collective bargaining.
(f) A decision to declare a strike must be approved
by a majority of the total union membership in the
Article 219(s), Labor Code. bargaining unit concerned, obtained by secret
(s) "Strike area" means the establishment, ballot in meetings or referenda called for that
warehouses, depots, plants or offices, including the purpose. A decision to declare a lockout must be
sites or premises used as runaway shops, of the approved by a majority of the board of directors of
employer struck against, as well as the immediate the corporation or association or of the partners in

44
a partnership, obtained by secret ballot in a Commission, under pain of immediate disciplinary
meeting called for that purpose. The decision shall action, including dismissal or loss of employment
be valid for the duration of the dispute based on status or payment by the locking-out employer of
substantially the same grounds considered when backwages, damages and other affirmative relief,
the strike or lockout vote was taken. The Ministry even criminal prosecution against either or both of
may, at its own initiative or upon the request of any them.
affected party, supervise the conduct of the secret
balloting. In every case, the union or the employer The foregoing notwithstanding, the President of the
shall furnish the Ministry the results of the voting at Philippines shall not be precluded from determining
least seven days before the intended strike or the industries that, in his opinion, are indispensable
lockout, subject to the cooling-off period herein to the national interest, and from intervening at
provided. any time and assuming jurisdiction over any such
labor dispute in order to settle or terminate the
(g) When, in his opinion, there exists a labor same.
dispute causing or likely to cause a strike or lockout
in an industry indispensable to the national (h) Before or at any stage of the compulsory
interest, the Secretary of Labor and Employment arbitration process, the parties may opt to submit
may assume jurisdiction over the dispute and their dispute to voluntary arbitration.
decide it or certify the same to the Commission for
compulsory arbitration. Such assumption or (i) The Secretary of Labor and Employment, the
certification shall have the effect of automatically Commission or the voluntary arbitrator shall decide
enjoining the intended or impending strike or or resolve the dispute, as the case may be. The
lockout as specified in the assumption or decision of the President, the Secretary of Labor
certification order. If one has already taken place at and Employment, the Commission or the voluntary
the time of assumption or certification, all striking arbitrator shall be final and executory ten (10)
or locked out employees shall immediately return- calendar days after receipt thereof by the parties.
to-work and the employer shall immediately
resume operations and readmit all workers under
the same terms and conditions prevailing before Article 279, Labor Code. Prohibited activities. -
the strike or lockout. The Secretary of Labor and (a) No labor organization or employer shall declare
Employment or the Commission may seek the a strike or lockout without first having bargained
assistance of law enforcement agencies to ensure collectively in accordance with Title VII of this Book
compliance with this provision as well as with such or without first having filed the notice required in
orders as he may issue to enforce the same. the preceding Article or without the necessary
strike or lockout vote first having been obtained
In line with the national concern for and the highest and reported to the Ministry.
respect accorded to the right of patients to life and
health, strikes and lockouts in hospitals, clinics and No strike or lockout shall be declared after
similar medical institutions shall, to every extent assumption of jurisdiction by the President or the
possible, be avoided, and all serious efforts, not Minister or after certification or submission of the
only by labor and management but government as dispute to compulsory or voluntary arbitration or
well, be exhausted to substantially minimize, if not during the pendency of cases involving the same
prevent, their adverse effects on such life and grounds for the strike or lockout.
health, through the exercise, however legitimate,
by labor of its right to strike and by management to Any worker whose employment has been
lockout. In labor disputes adversely affecting the terminated as a consequence of any unlawful
continued operation of such hospitals, clinics or lockout shall be entitled to reinstatement with full
medical institutions, it shall be the duty of the backwages. Any union officer who knowingly
striking union or locking-out employer to provide participates in an illegal strike and any worker or
and maintain an effective skeletal workforce of union officer who knowingly participates in the
medical and other health personnel, whose commission of illegal acts during a strike may be
movement and services shall be unhampered and declared to have lost his employment status:
unrestricted, as are necessary to insure the proper Provided, That mere participation of a worker in a
and adequate protection of the life and health of its lawful strike shall not constitute sufficient ground
patients, most especially emergency cases, for the for termination of his employment, even if a
duration of the strike or lockout. In such cases, replacement had been hired by the employer
therefore, the Secretary of Labor and Employment during such lawful strike.
may immediately assume, within twenty four (24)
hours from knowledge of the occurrence of such a (b) No person shall obstruct, impede, or interfere
strike or lockout, jurisdiction over the same or with, by force, violence, coercion, threats or
certify it to the Commission for compulsory intimidation, any peaceful picketing by employees
arbitration. For this purpose, the contending parties during any labor controversy or in the exercise of
are strictly enjoined to comply with such orders, the right to self-organization or collective
prohibitions and/or injunctions as are issued by the bargaining, or shall aid or abet such obstruction or
Secretary of Labor and Employment or the interference.

45
the driver of the service truck decided to return to the
(c) No employer shall use or employ any strike- respondent’s compound. Later that day, the group
breaker, nor shall any person be employed as a staged a picket in front of company’s compound,
strike-breaker. carrying placards with slogans (Containing: “Ibalik
ang pasok sa Finishing Department; “Huwag mong
(d) No public official or employee, including officers ipitin ang mid-year, 13th month pay ng mga
and personnel of the New Armed Forces of the manggagawa sa CKC”, etc.). Respondent filed a
Philippines or the Integrated National Police, or
petition to declare the strike illegal.
armed person, shall bring in, introduce or escort in
any manner, any individual who seeks to replace
strikers in entering or leaving the premises of a A strike is any temporary stoppage of work by the
strike area, or work in place of the strikers. The concerted action of employees as a result of an
police force shall keep out of the picket lines unless industrial or labor dispute. A labor dispute includes
actual violence or other criminal acts occur therein: any controversy or matter concerning terms or
Provided, That nothing herein shall be interpreted conditions of employment or the association or
to prevent any public officer from taking any
representation of persons in negotiating, fixing,
measure necessary to maintain peace and order,
protect life and property, and/or enforce the law maintaining, changing or arranging the terms and
and legal order. conditions of employment, regardless of whether the
disputants stand in the proximate relation of
(e) No person engaged in picketing shall commit any act of employer and employee.
violence, coercion or intimidation or obstruct the free
ingress to or egress from the employer’s premises for The petitioner union, its officers, members and
lawful purposes, or obstruct public thoroughfares.
supporters staged a strike. In order for a strike to be
valid, the following requirements laid down in
Article 280, Labor Code. Improved offer paragraphs (c) and (f) of Article 263 of the Labor
balloting. - In an effort to settle a strike, the Code must be complied with: (a) a notice of strike
Department of Labor and Employment shall must be filed; (b) a strike-vote must be taken; and (c)
conduct a referendum by secret ballot on the the results of the strike-vote must be reported to the
improved offer of the employer on or before the DOLE. It bears stressing that these requirements are
30th day of the strike. When at least a majority of mandatory, meaning, non-compliance therewith
the union members vote to accept the improved
makes the strike illegal. The evident intention of the
offer the striking workers shall immediately return
to work and the employer shall thereupon readmit law in requiring the strike notice and strike-vote
them upon the signing of the agreement. report is to reasonably regulate the right to strike,
which is essential to the attainment of legitimate
In case of a lockout, the Department of Labor and policy objectives embodied in the law.
Employment shall also conduct a referendum by
secret balloting on the reduced offer of the union
on or before the 30th day of the lockout. When at Steel Corporation v. SCP Employees
least a majority of the board of directors or trustees Union, April 16, 2008
or the partners holding the controlling interest in SCPEU-NAFU filed a notice of strike based in refusal
the case of a partnership vote to accept the
to bargain. The Union staged the strike to compel SCP
reduced offer, the workers shall immediately return
to work and the employer shall thereupon readmit to recognize it as the collective bargaining agent, as
them upon the signing of the agreement. the employees’ bargaining representative to work out
a CBA despite the striking union’s doubtful majority
status to merit voluntary recognition and lack of
Article 281, Labor Code. Requirement for arrest formal certification as the exclusive representative in
and detention. - Except on grounds of national the bargaining unit. The certification election that
security and public peace or in case of commission was conducted where SCPEU-NAFLU emerged as
of a crime, no union members or union organizers
winner, not having been recognized as valid, it has no
may be arrested or detained for union activities
without previous consultations with the Secretary authority to represent the RF-EE of SCP. Thus, it could
of Labor. not ask SCP to bargain with it.

The Court ruled that The strike is a legitimate weapon


Bukluran ng Manggagawa sa Clothman in the human struggle for a decent existence. It is
Knitting v. CA, January 17, 2005 considered as the most effective weapon in
Respondent company decided to temporarily shut- protecting the rights of the employees to improve the
down its operations. However, while company’s terms and conditions of their employment. But to be
service truck was about to deliver fabric petitioners valid, a strike must be pursued within legal bounds.
approached the truck and blocked its way. As a result, The right to strike as a means for the attainment of

46
social justice is never meant to oppress or destroy were illegally locked out as a result of their taking
the employer. The law provides limits for its exercise. part in said event. Also they allege that as to their
Here, the strike is illegal for (1) it is a union- putting up of tents, tables and chairs in front of the
recognition- strike which is not sanctioned by labor main gate of the company premises, this was for the
laws; (2) it was undertaken after the dispute had benefit of their members who reported every morning
been certified for compulsory arbitration; and (3) it to see if the management would allow them to work.
was in violation of the Secretary’s return-to- work
order. Stoppage of work due to welga ng bayan is in the
nature of a general strike, an extended sympathy
Even if this Court were to uphold the validity of strike. It affects numerous employers including those
respondent’s purpose or objective in staging a strike, who do not have a dispute with their employees
still, the strike would be declared illegal for having regarding their terms and conditions of employment.
been conducted in utter defiance of the Secretary’s Employees who have no labor dispute with their
return-to-work order and after the dispute had been employer but who, on a day they are scheduled to
certified for compulsory arbitration. Although work, refuse to work and instead join a welga ng
ostensibly there were several notices of strike bayan commit an illegal work stoppage.
successively filed by respondent, these notices were
founded on substantially the same grounds— Even if petitioners’ joining the welga ng bayan were
petitioner’s continued refusal to recognize it as the considered merely as an exercise of their freedom of
collective bargaining representative. expression, freedom of assembly or freedom to
petition the government for redress of grievances,
The powers granted to the Secretary under Article the exercise of such rights is not absolute. For the
263(g) of the Labor Code have been characterized as protection of other significant state interests such as
an exercise of the police power of the State, aimed at the “right of enterprises to reasonable returns on
promoting the public good. When the Secretary investments, and to expansion and growth”
exercises these powers, he is granted “great breadth enshrined in the 1987 Constitution must also be
of discretion” to find a solution to a labor dispute. The considered, otherwise, oppression or self-destruction
most obvious of these powers is the automatic of capital in order to promote the interests of labor
enjoining of an impending strike or lockout or its would be sanctioned. And it would give imprimatur to
lifting if one has already taken place. workers’ joining demonstrations/rallies even before
affording the employer an opportunity to make the
The moment the Secretary of Labor assumes necessary arrangements to counteract the
jurisdiction over a labor dispute in an industry implications of the work stoppage on the business,
indispensable to national interest, such assumption and ignore the novel “principle of shared
shall have the effect of automatically enjoining the responsibility between workers and employers”
intended or impending strike. It was not even aimed at fostering industrial peace. There being no
necessary for the Secretary of Labor to issue another showing that petitioners notified respondents of their
order directing a return to work. The mere issuance of intention, or that they were allowed by respondents,
an assumption order by the Secretary of Labor to join the welga ng bayan, their work stoppage is
automatically carries with it a return-to-work order, beyond legal protection.
even if the directive to return to work is not expressly
stated in the assumption order. Even assuming arguendo that in staging the strike,
petitioners had complied with legal formalities, the
strike would just the same be illegal, for by blocking
Biflex Phils. v. Filflex Industrial & the free ingress to and egress from the company
Manufacturing Corp., Dec. 19, 2006 premises, they violated Article 264(e) of the Labor
In response to the accelerating prices of oil the labor Code which provides that “[n]o person engaged in
sector conducted a welga ng bayan where the picketing shall … obstruct the free ingress to or
petitioner unions led by their officers took part of by egress from the employer’s premises for lawful
conducting a work stoppage which lasted for 3 purposes, or obstruct public thoroughfares.” In fine,
weeks. Respondents filed a petition to declare illegal the legality of a strike is determined not only by
the said work stoppage of petitioners because they compliance with its legal formalities but also by the
did not comply with certain procedural requirements means by which it is carried out.
under the Labor Code which are (1) Filing of a Notice
of Strike and (2) Securing a Strike Vote and (3) Bascon & Cole v. CA, February 5, 2004
Submission of the report of the Strike Vote to the Petitioners were employees of MCCH and members of
DOLE. Petitioner Unions retaliated by claiming that the NAMA-MCCH, a labor union of MCCH employees.
after the end of the welga ng bayan, their members NAMA-MCCH asked MCCH to renew their Collective
47
Bargaining Agreement (CBA) which was set to expire. the deliberate absence of a considerable number of
The National Labor Federation (NFL) opposed the employees Toyota experienced lack of manpower
move by NAMA-MCCH who was their local affiliate. resulting in huge losses. Subsequently, Toyota
NAMA-MCCH staged a series of mass actions inside terminated 227 employees. The terminated
MCCH's premises. They marched around the hospital employees allegedly abandoned their work.This
putting up streamers, placards and posters. The resulted to another rally within Toyota’s premises as
means of ingress to and egress from the hospital the strikers barricaded the entrances of Toyota
were blocked. Employees and patients, including preventing non-strikers from going to work.
emergency cases, were harassed, according to MCCH
management, which also complained that mass The Court ruled that the protest actions undertaken
actions held inside the hospital had created an by the Union officials and members are not valid and
atmosphere of animosity and violence, aggravating proper exercises of their right to assemble but are
the condition of ailing patients illegal strikes in breach of the Labor Code. The
Union’s position is weakened by the lack of permit
The Court held that while a union officer can be from the City of Manila to hold “rallies.” Cloaked as
terminated for mere participation in an illegal strike, demonstrations, they were in reality temporary
an ordinary striking employee, like petitioners herein, stoppages of work perpetrated through the concerted
must have participated in the commission of illegal action of the employees who deliberately failed to
acts during the strike. There must be proof that they report for work on the convenient excuse that they
committed illegal acts during the strike. But proof will hold a rally at the BLR and DOLE offices.
beyond reasonable doubt is not required. Substantial
evidence, which may justify the imposition of the It is obvious that the concerted actions were
penalty of dismissal, may suffice. undertaken without satisfying the prerequisites for a
valid strike under Art. 263 of the Labor Code. The
In this case, the Court of Appeals found that Union failed to comply with the following
petitioners' actual participation in the illegal strike requirements: (1) a notice of strike filed with the
was limited to wearing armbands and putting up DOLE 30 days before the intended date of strike, or
placards. There was no finding that the armbands or 15 days in case of unfair labor practice; (2) strike
the placards contained offensive words or symbols. vote approved by a majority of the total union
Thus, neither such wearing of armbands nor said membership in the bargaining unit concerned
putting up of placards can be construed as an illegal obtained by secret ballot in a meeting called for that
act. purpose; and (3) notice given to the DOLE of the
results of the voting at least seven days before the
In fact, they are within the mantle of constitutional intended strike.
protection under freedom of speech. Evidence on
record shows that various illegal acts were committed These requirements are mandatory and the failure of
by unidentified union members in the course of the a union to comply with them renders the strike illegal.
protracted mass action AFTER petitioners were The evident intention of the law in requiring the strike
terminated (emphasis supplied by author). And we notice and the strike-vote report is to reasonably
commiserate with MCCH, patients, and third parties regulate the right to strike, which is essential to the
for the damage they suffered. attainment of legitimate policy objectives embodied
in the law. As they failed to conform to the law, the
But we cannot hold petitioners responsible for acts strikes were illegal.
they did not commit. The law, obviously solicitous of
the welfare of the common worker, requires, before NUWHRAIN Dusit Hotel Nikko Chapter v.
termination may be considered, that an ordinary
union member must have knowingly participated in
CA, November 11, 2008
the commission of illegal acts during a strike. The Union submitted its CBA negotiation proposals to
the Hotel, however they failed to arrive at an
agreement. The Union filed a Notice of Strike on the
Toyota Motor Phils. Corp. Workers ground of bargaining deadlock. The mediation was
Association v. Toyota Motor Phils, Oct. also unsuccessful and a Strike Vote was conducted by
19, 2007 the Union where they decided to stage a strike. The
Union submitted its CBA proposals to Toyota but the Union held a General Assembly where some members
latter refused to bargain pending its appeal. The sported closely cropped or clean shaven heads. The
Union then filed a notice of strike. Union officers and following day, more male Union members followed
members failed to render the required overtime work, suit. They were prevented by the Hotel from entering
and instead marched to and staged a picket. Due to its premises for violation of the Hotel’s grooming

48
standards.The Union thereafter staged a picket Capitol Medical Center v. NLRC, April 26,
outside the Hotel premises, where later on other
members were also prevented from entering the
2005
Hotel, thus lacking manpower in the Hotel forcing Union filed a Notice of Strike.Petitioner then filed a
them to temporarily cease operations in 3 Letter before the Board requesting that the notice of
restaurants. strike be dismissed, because the Union had
apparently failed to furnish the Regional Branch of
Art. 212(o) of the Labor Code defines a strike as “any the NCMB with a copy of a notice of the meeting
temporary stoppage of work by the concerted action where the strike vote was conducted.. Thereafter, the
of employees as a result of an industrial or labor officers and members of the Union staged a strike.
dispute.” In Toyota Motor Phils. Corp. Workers Petitioner then filed a petition before the NLRC to
Association (TMPCWA) v. National Labor Relations declare the strike illegal. LA: sided with petitioner
Commission, 537 SCRA 171 (2007), we cited the that strike was illegal—because no voting had taken
various categories of an illegal strike, to wit: Noted place; no notice of such voting was furnished to the
authority on labor law, Ludwig Teller, lists six (6) NCMB at least 24-hours prior to the intended holding
categories of an illegal strike, viz.: of the strike vote.
(1) [when it] is contrary to a specific prohibition of
law, such as strike by employees performing The Court found that the Union failed to comply with
governmental functions; or the 24-hour notice requirement to the Board before it
(2) [when it] violates a specific requirement of law[, conducted the alleged strike vote meeting. This
such as Article 263 of the Labor Code on the rendered the strike ILLEGAL. The illegality of the
requisites of a valid strike]; or strike is further bolstered by the fact that the alleged
(3) [when it] is declared for an unlawful purpose, such meeting never took place (meaning, it doesn’t have
as inducing the employer to commit an unfair labor the blessing of majority of the union membership).
practice against nonunion employees; or Union failed to comply with Sec. 10, Rule XXII which
(4) [when it] employs unlawful means in the pursuit provides that:… a decision to declare a strike must be
of its objective, such as a widespread terrorism of approved by a majority of the total union
nonstrikers [for example, prohibited acts under Art. membership in a BU concerned obtained by secret
264(e) of the Labor Code]; or ballot in meetings or referenda called for the
(5) [when it] is declared in violation of an existing purpose…The regional branch of the Board may, at
injunction[, such as injunction, prohibition, or order its own initiative or upon the request of any affected
issued by the DOLE Secretary and the NLRC under party, supervise the conduct of the secret balloting.
Art. 263 of the Labor Code]; or In every case, the union or employer shall furnish the
(6) [when it] is contrary to an existing agreement, regional branch a notice of meetings…at least 24-
such as a nostrike clause or conclusive arbitration hours before such meetings, as well as the results of
clause. the voting at least 7-days before the intended strike

Regarding the Union officers and members’ liabilities In National Federation of Labor v. NLRC, the Court
for their participation in the illegal picket and strike, enumerated the notices required by Art. 263 of the
Art. 264(a), paragraph 3 of the Labor Code provides Labor Code and the Rules:
that “[a]ny union officer who knowingly participates 1. A notice of strike, w/ the required contents,
in an illegal strike and any worker or union officer should be filed with the DOLE, specifically the
who knowingly participates in the commission of Regional Branch of the Board, copy furnished
illegal acts during a strike may be declared to have the employer;
lost his employment status x x x.” The law makes a 2. A cooling-off period must be observed
distinction between union officers and mere union between the filing of notice and the actual
members. Union officers may be validly terminated strike 30-days in case of bargaining deadlock;
from employment for their participation in an illegal and 15-days in case of unfair labor practice.
strike, while union members have to participate in 3. Before a strike is actually commenced, a
and commit illegal acts for them to lose their strike vote should be taken by secret
employment status. Thus, it is necessary for the balloting, with a 24-hour prior notice to the
company to adduce proof of the participation of the Board
striking employees in the commission of illegal acts 4. The result of the strike vote should be
during the strikes. reported to the Board at least 7-days before
the intended strike or lockout, subject to the
cooling-off period

49
The requirement of giving notice of the conduct of a subject to limitations imposed by law. Article 263 (g)
strike vote to the Board at least 24-hours before the of the Labor Code constitutes one such limitation
meeting is designed to (a) inform the Board of the provided by law. To reiterate, Article 263 (g) of the
intent of the union to conduct a strike vote; (b) give Labor Code constitutes an exception to the
the Board ample time to decide on whether or not management prerogative of hiring, firing, transfer,
there is a need to supervise the conduct of strike vote demotion and promotion of employees. And to the
to prevent any acts of violence and/or irregularities; extent that Article 263 (g) calls for the admission of
and (c) should the Board decide on its own initiative all workers under the same terms and conditions
or upon request of an interested party including the prevailing before the strike, the respondent is
employer, to supervise the strike vote, to give it restricted from exercising its generally unbounded
ample time to prepare deployment of required right to transfer or reassign its employees. The
personnel. Until and unless the Board is notified at respondent is mandated, under the said order, to
least 24-hours of the unions decision to conduct a issue embarkation orders to the employees to enable
strike vote, and the date, place, and time thereof, the them to report to their ship assignments in
Board cannot determine for itself whether to compliance with the Order of the Secretary of Labor.
supervise a strike vote meeting or not. Failure of a
union to comply with the requirements of giving
notice to the Board at least 24-hours prior to the
Manila Diamond Hotel Employees Union
holding of a strike vote meeting will render the v. CA, Secretary, December 16, 2004
subsequent strike ILLEGAL. Union filed a petition for CE, which was dismissed.
After a few months, the Union sent a letter to the
Trans-Asia Shipping Lines-Unlicensed Hotel informing it of its desire to negotiate for a CBA.
The Hotel informed the Union that it can’t recognize
Crews Employees Union v. CA, July 7, the latter as the employee’s bargaining agent since
2004 its petition for certification election had been
Unions of R&F and supervisory employees filed dismissed. The Union announced that it was taking a
separate notices of strike with the NCMB against strike vote, they filed a notice of strike for the Hotel’s
respondent for ULP. To avert work stoppage, Sec. alleged refusal to bargain. The NCMB summoned both
Laguesma intervened and issued the order certifying parties and held a series of dialogues.
the labor dispute to the NLRC for compulsory
arbitration and enjoining the strike. Despite the order, The Union, however, staged a strike against the
petitioners went on strike, paralyzing the operations Hotel. The Hotel claims that the strike was illegal and
of the employer. Sec. Laguesma was constrained to it had to dismiss some employees for their
issue an order directing all striking workers to return participation in the allegedly illegal concerted activity.
to work within 12 hours and for the company to The Union, on the other hand, accused the Hotel of
accept them back under the same terms and illegally dismissing the workers. Then Secretary
conditions prevailing before the strike. Trajano assumed jurisdiction over the labor dispute.
He issued an Order directing the striking employees
The powers granted to the Secretary of Labor under to return to work. However, the Hotel refused to
Article 263 (g) of the Labor Code have been accept them, modifying the order issued by Secretary
characterized as an exercise of the police power of Trajano. Instead of an actual return to work, Acting
the State, with the aim of promoting public good. Secretary Espaol directed that the strikers be
When the Secretary exercises these powers, he is reinstated only in the payroll.
granted “great breadth of discretion” in order to find
a solution to a labor dispute. The most obvious of The Court ruled that there is no showing that the
these powers is the automatic enjoining of an facts called for payroll reinstatement as an
impending strike or lockout or the lifting thereof if one alternative remedy. A strained relationship between
has already taken place. Assumption of jurisdiction the striking employees and management is no reason
over a labor dispute, or as in this case the for payroll reinstatement in lieu of actual
certification of the same to the NLRC for compulsory reinstatement.
arbitration, always co-exists with an order for workers
to return to work immediately and for employers to As a general rule, the State encourages an
readmit all workers under the same terms and environment wherein employers and employees
conditions prevailing before the strike or lockout. themselves must deal with their problems in a
manner that mutually suits them best. Article 263,
Case law recognizes the employer’s right to transfer paragraph (g) of the Labor Code, which allows the
or assign employees from one area of operation to Secretary of Labor to assume jurisdiction over a labor
another. This right, however, is not absolute but dispute involving an industry indispensable to the
50
national interest, provides an exception: (g) When, in but is essentially involved in, the labor dispute itself.
his opinion, there exists a labor dispute causing or The powers granted to the Secretary under Article
likely to cause a strike or lockout in an industry 263(g) of the Labor Code have been characterized as
indispensable to the national interest, the Secretary an exercise of the police power of the State, with the
of Labor and Employment may assume jurisdiction aim of promoting public good. When the Secretary
over the dispute and decide it or certify the same to exercises these powers, he is granted great breadth
the Commission for compulsory arbitration. of discretion in order to find a solution to a labor
dispute. The most obvious of these powers is the
Such assumption or certification shall have the effect automatic enjoining of an impending strike or lockout
of automatically enjoining the intended or impending or its lifting if one has already taken place. In this
strike or lockout as specified in the assumption or case, the Secretary assumed jurisdiction over the
certification order. If one has already taken place at dispute because it falls in an industry indispensable
the time of assumption or certification, all striking or to the national interest. It is of no moment that PEU
locked out employees shall immediately return to never acquiesced to the submission for resolution of
work and the employer shall immediately resume the issue on the legality of the strike. PEU cannot
operations and readmit all workers under the same prevent resolution of the legality of the strike by
terms and conditions prevailing before the strike or merely refusing to submit the issue for resolution. It is
lockout. also immaterial that this issue, as PEU asserts, was
not properly submitted for resolution of the Secretary.
The Court pointed out that the law uses the precise The authority of the Secretary to assume jurisdiction
phrase of “under the same terms and conditions,” over a labor dispute causing or likely to cause a strike
revealing that it contemplates only actual or lockout in an industry indispensable to national
reinstatement. This is in keeping with the rationale interest includes and extends to all questions and
that any work stoppage or slowdown in that particular controversies arising from such labor dispute. The
industry can be inimical to the national economy. power is plenary and discretionary in nature to enable
him to effectively and efficiently dispose of the
Philcom Employees Union v. Phil. Global dispute.

Communications, July 17, 2006


While CBA negotiations were ongoing, PEU filed a
Nissan Motors v. Secretary, June 21,
Notice of Strike due to perceived unfair labor practice 2006
and bargaining deadlock. While the union and the Notice of Strike was filed by the Union due to
company officers and representatives were meeting, collective bargaining deadlock. DOLE assumed
the remaining union officers and members staged a jurisdiction and issued an order directing any party to
strike at the company premises, barricading the cease and desist for committing any act that might
entrances and egresses thereof and setting up a exacerbate the situation. Oder had been violated so
stationary picket at the main entrance of the building. DOLE thereafter ordered dismissal of union officers
Then Acting Labor Secretary Trajano issued an Order and recall dismissal of EEs.
assuming jurisdiction over the dispute, enjoining any While the employer is authorized to declare a union
strike or lockout, directing the parties to cease from officer who participated in an illegal strike as having
committing any act that may exacerbate the lost his employment, his/its option is not as wide with
situation, directing the striking workers to return to respect to union members or workers for the law
work within twenty-four (24) hours from receipt of the itself draws a line and makes a distinction between
Secretary’s Order and for management to resume union officers and members/ordinary workers. An
normal operations, as well as accept the workers ordinary striking worker or union member cannot, as
back under the same terms and conditions prior to a rule, be terminated for mere participation in an
the strike. illegal strike; there must be proof that he committed
illegal acts during the strike. And lest it be forgotten,
The issue is WON the Secretary properly took the law invests the Secretary of Labor and
cognizance of the issue on the alleged illegal strike Employment the prerogative of tempering the
even though it was not properly submitted to the consequence of the defiance to the assumption order.
Secretary for resolution. (YES) The Secretary may thus merely suspend rather than
dismiss the employee involved.
The Secretary properly took cognizance of the issue
on the legality of the strike. As correctly pointed out, FEU-NRMF v. FEU-NRMFEA-AFW, October
since the very reason of the Secretary’s assumption
of jurisdiction was PEUs declaration of the strike, any
16, 2006
issue regarding the strike is not merely incidental to, Union filed notice of strike on the ground of deadlock.
A strike vote was conducted, and after the expiration
51
of the thirty day cooling of period and seven day day, union staged a strike. SC held that strike was
strike ban, union staged a strike. Before the strike, illegal. One of the procedural requirements for the
union offered skeletal force of nursing and health strike to be valid is the observance of the seven-day
personnel who will man the hospital’s operation for strike ban. Here, the strike was done on the same day
duration of strike but the hospital refused to accept. when the notice has been filed. The Union violated
the seven-day strike ban. This requirement should be
FEUNRMF filed a Petition for the Assumption of observed to give the DOLE an opportunity to verify
Jurisdiction (AJO) or for Certification of Labor Dispute whether the projected strike really carries the
with the NLRC. Secretary of Labor granted the approval of the majority of the union members. Also,
petition and an Order assuming jurisdiction over the there was no union busting to warrant the non
labor dispute was issued, thereby prohibiting any observance of the cooling off period. To constitute
strike. The copy of the AJO was not served to the union busting, there must be a dismissal and the
respondent because there no union officer was existence of union must be threatened by such
around. Instead the copy was posted in several dismissal. Here, notice of strike merely assailed mass
conspicuous places within the premises of the promotion. Promotion is different from dismissal.
hospital. Striking employees continued to strike
claiming that they did not know about the AJO order. Club Filipino v. Bautista, July 13, 2009
Union made several demands for negotiation but the
Under the NLRC Revised Rules of Procedure, service company refused to do so. Union filed notice of strike
of copies of orders should be made by the process on the ground of bargaining deadlock. Two weeks
server either personally or through registered mail. after, the company formally responded to the
Merely posting copies of the AJO does not satisfy the demands of the union. In cases of bargaining
rigid requirement for proper service. Needless to say, deadlocks, the notice shall, as far as practicable,
the manner of service made by the process server further state the unresolved issues in the bargaining
was invalid and irregular. Respondent union could not negotiations and be accompanied by the written
therefore be adjudged to have defied the said Order proposals of the union, the counter-proposals of the
since it was not properly apprised thereof. The employer and the proof of a request for conference to
affidavits presented by the petitioner FEU-NRMF and settle differences. In cases of unfair labor practices,
relied upon by the Labor Arbiter and the NLRC, in the notice shall, as far as practicable, state the acts
arriving at the conclusion that the respondent union complained of, and efforts taken to resolve the
committed illegal acts during the strike, could not be dispute amicably. Any notice which does not conform
given probative value by this Court as the adverse with the requirements of this and the foregoing
party was not given a chance to cross-examine the section shall be deemed as not having been filed and
affiants. Accordingly, the strike conducted by the the party concerned shall be so informed by the
respondent union was valid under the circumstances. regional branch of the Board.

For a strike to be valid, the following requisites must Here, the union could not have attached the counter
concur: (1) the thirty-day notice or the fifteen-day proposal of the company in the notice of strike as
notice, in case of unfair labor practices; (2) the two- there was no such counter proposal. One cannot give
thirds (2/3) required vote to strike done by secret what he does not have. Indeed, compliance with the
ballot; and (3) the submission of the strike vote to the requirement was impossible because no counter-
DOLE at least seven days prior to the strike. In proposal existed at the time the union filed a notice
addition, in case of strikes in hospitals, it shall be the of strike. Hence, strike staged was legal.
duty of the striking employees to provide and
maintain an effective and skeletal workforce of
medical and other health personnel in order to insure
Soriano Aviation v. Employees
the proper and adequate protection of the life and Association of A. Soriano Aviation, August
health of its patients. These procedural requirements, 14, 2009
along with the mandatory cooling off and strike ban CBA contains “no strike, no lockout” clause. During
periods had been fully observed by the respondent the holiday, members of the union refused to render
union. OT work. Company treated it as a violation of the CBA
so it gave 30-day suspension to the said EE’s. Union
Pilipino Telephone Corporation v. PILTEA, went on a strike. Strike is invalid when the employed
means was illegal. The union members repeated
June 22, 2007 name-calling, harassment and threats of bodily harm
Union filed a notice of strike on the ground of union directed against company officers and non-striking
busting and mass promotion of its officers and employees and more significantly, the putting up of
members during the CBA negotiations. On the same placards, banners and streamers with vulgar
52
statements imputing criminal negligence to the infractions committed by the dismissed Union
company come within the puri ew of illegal acts. members.
Hence the strike was illegal.
PHIMCO Industries, Inc. v. PILA, August
Jackbilt Industries v. Jackbilt Employees 11, 2010
Union, March 20, 2009 Parties end up with a collective bargaining deadlock.
Due to crisis, Jackbilt decided to temporarily stop its PILA then staged a strike preventing-through force,
business compelling its EEs to go on leave for 6 intimidation and coercion- the ingress and egress of
months. So the union went on strike. NLRC ordered non-striking EE’s into and from company premises.
the union to refrain from preventing access to Strike is illegal. Despite the validity and compliance
Jackbilt’s property but the union, as found by the of procedural requirements, strike could still be held
investigating labor arbiter, violated the order because illegal if the means employed is illegal. The means
on various occasions, it stopped and inspected become illegal when they come within this
private vehicles from entering and exiting the prohibition: “no person engaged in picketing shall
company’s premises. Hence, NLRC issued writ of commit any act of violence, coercion, or intimidation
preliminary injunction against the union. Union were or obstruct the free ingress to or egress from
dismissed so they filed for ULP but company said that premises…”
the dismissal was valid because the union conducted
an illegal strike. Solidbank Corporation v. Gamier,
Company is correct. The use of unlawful means in the
November 15, 2010
course of a strike renders such strike illegal. Union declared a deadlock and filed a notice of
Therefore, strike was ipso facto illegal. The filing of a strike. DOLE ordered the union to refrain from
petition to declare the strike illegal was thus committing any acts that might exacerbate the
unnecessary. Here, NLRC already found that union situation. But still, the union protested by holding a
committed illegal acts by preventing the free entry rally in from of the office of secretary of labor. EE’s
into and exit from Jackbilt’s compound. did not work for 3 days.

SC ruled that the rally violated the order of DOLE.


Alcantara & Sons v. CA, GR G.R. No. Strike is any temporary stoppage of work by
155109, September 29, 2010 concerted action of EEs as a result of labor dispute.
The CBA has no strike and no lockout provisions. The term “strike” shall comprise not only concerted
Parties entered into a deadlock so Union filed a notice work stoppages, but also slowdowns, mass leaves,
of strike. Union conducted a strike vote and majority sitdowns, attempts to damage, destroy or sabotage
favored it. The Union reported the strike vote to the plant equipment and facilities and similar activities.
DOLE. After the observance of the mandatory cooling- Thus, the fact that the conventional term “strike” was
off period, they went on strike. LA: Union’s strike is not used by the striking employees to describe their
illegal; Union officers should be deemed to have common course of action is inconsequential, since
forfeited their employment; and Union members, the substance of the situation, and not its
finding no proof that they actually committed illegal appearance, will be deemed to be controlling.
acts during the strike, should be reinstated.

The SC held that Union committed an illegal strike. A


Escario v. NLRC, September 27, 2010
strike may be regarded as invalid although the labor The petitioners were regular employees of PINA and
union has complied with the strict requirements for were part of the union. All the officers and members
staging one as provided in Article 263 of the Labor of the union walked out of PINA's premises to go to
Code when the same is held contrary to an existing the barangay office to support Canete, an officer of
agreement, such as a no strike clause or conclusive the union charged with oral defamation by PINA's
arbitration clause. Here, the CBA between the parties officers. Due to the walk-out, PINA preventively
contained a "no strike, no lockout" provision 2)Union suspended all officers of the Union and thereafter
members should also be terminated because of their terminated them. The union filed a notice of strike,
illegal acts of threatening, coercing and intimidating claiming that PINA was guilty of union busting
non-strikers, obstructing the free ingress and egress through the constructive dismissal of its officers. PINA
from the company premises and resisted and defied also filed a complaint for ULP and abandonment of
the implementation of the writ of preliminary work against the union . LA ruled that strike was
injunction. But they are given financial assistance illegal, and was affirmed by NLRC and CA.
because the records fail to disclose any past

53
There are two causes for the dismissal of an complying with the mandatory legal requirements of
employee, namely unlawful lockout, and participation a strike notice and strike vote.
in an illegal strike. The third paragraph of Art 264(a)
authorizes the award of full backwages only when the
termination of employment is a consequence of an
Fadriquelan v. Monterey Foods, June 8,
unlawful lockout. On the consequences of an illegal 2011
strike, the provision distinguishes between a union The CBA between the company and the union expired
officer and a union member participating in an illegal so when the same resulted in a deadlock, the union
strike. A union officer who knowingly participates in filed a notice of strike with the NCMB. Said company
an illegal strike is deemed to have lost his filed with the DOLE a petition for assumption of
employment status, but a union member who is jurisdiction over the dispute in view of the dire effects
merely instigated or induced to participate in the on the meat industry. Thereafter, the union filed a
illegal strike is more benignly treated. With respect to second notice of strike before the NCMB on the
backwages, the principle of a day’s wage for a fair ground that the company committed ULP. It filed a
day’s labor remains as the basic factor in determining third notice of strike on the claim that company
the award. If there is no work performed by the engaged in union busting and illegal dismissal of
employee there can be no wage or pay unless, of union officers. No strike shall be declared after the
course, the laborer was able, willing and ready to Secretary of Labor has assumed jurisdiction over a
work but was illegally locked out, suspended or labor dispute. A strike conducted after such
dismissed or otherwise illegally prevented from assumption is illegal and any union officer who
working. Under the principle of a fair day’s wage for a knowingly participates in the same may be declared
fair day’s labor, the petitioners were not entitled to as having lost his employment. it. The ordinary
the wages during the period of the strike (even if the worker cannot be terminated for merely participating
strike might be legal), because they performed no in the strike. There must be proof that he committed
work during the strike. illegal acts during its conduct. On the other hand, a
union officer can be terminated upon mere proof that
Bagong Pagkakaisa ng Manggagawa sa he knowingly participated in the illegal strike.

Triumph v. Secretary, July 5, 2010 In this case, identity and participations of Eguna,
The union and Triumph had an existing which expired Malaluan, Alonso, Dimaano, Mayuga, Rizaldo, Suico,
so they entered into re-negotiations but the same Escamillas, and Bautroin the slowdowns were
resulted in a deadlock. A notice of strike was filed by properly established. These officers simply refused to
the union and the company filed a notice of lockout work or they abandoned their work to join union
for ULP due to the union's work slow-down. The assemblies. In termination cases, the dismissed
Secretary of Labor issued a return to work order, employee is not required to prove his innocence of
wherein some employees attempted to return but the charges against him. The burden of proof rests
were prevented by the striking workers. The upon the employer to show that the employees
company informed NCMB that it was willing to accept dismissal was for just cause. The employers failure to
all returning employees so the striking employees do so means that the dismissal was not justified.
returned to work but 20 union officers were still not Here, the company failed to show that all 17 union
allowed entry. They were served a notice of officers deserved to be dismissed.
termination.

Under the law, the Labor Secretary's assumption of


Magdala Multipurpose & Livelihood v.
jurisdiction over the dispute or its certification to the KMLMS, October 19, 2011
National Labor Relations Commission for compulsory KMLMS filed a notice of strike on March 5, 2002 but
arbitration shall have the effect of automatically only acquired its legal personality when its
enjoining the intended or impending strike or lockout registration as an independent labor organization was
and all striking or locked out employees shall granted on April 9, 2002. On May 6, 2002, it staged a
immediately return to work and the employer shall strike where everal prohibited acts were committed
immediately resume operations and readmit all by its members. Magdala filed a petition to declare
workers under the same terms and conditions before the strike illegal and that the officers and members of
the strike or lockout. Thus, the Union’s defiance of the the union who participated be declared to have lost
order of the Secretary of Labor made the strike their employment status. There is no question that
illegal. They were also liable for leading and the strike was illegal because when the union had
instigating slowdown (during the CBA negotiations), a filed a notice of strike and conducted a strike-vote, it
form of strike undertaken by the union without had not yet acquired legal personality and thus could
not legally represent the union and its members.

54
Thus, the mandatory notice of strike and the conduct in labor cases warrants the restoration of the status
of the strike-vote report were ineffective for having quo ante and bringing the parties back to the
been filed and conducted before KMLMS acquired respective positions before the illegal strike and
legal personality and the failure of the union to illegal lockout. In the case at bar, since both AER and
comply with the same renders the strike illegal. the union are at fault or in pari delicto, they should be
Furthermore, the the striking workers committed acts restored to their respective positions prior to the
of (1) interference by obstructing the free ingress to illegal strike and illegal lockout. Nonetheless, if
or egress from petitioners’ compound and (2) reinstatement is no longer feasible, the concerned
coercion and intimidation. union. For union officers, employees should be given separation pay up to the
knowingly participating in an illegal strike is a valid date set for the return of the complaining employees
ground for termination of their employment. But for in lieu of reinstatement.
union members who participated in a strike, their
employment may be terminated only if they Naranjo v. Biomedica Health Care,
committed prohibited and illegal acts during the
strike and there is substantial evidence or proof of
September 19, 2012
their participation, 5 employees of Biomedica Health were absent for
personal reasons for one day but when they returned
to work the next day, they were not allowed to enter
Automotive Engine Rebuilders v. the premises. Biomedica issued notices to the
Progresibong Unyon, July 13, 2011; petitioners, accusing them of conducting an illegal
January 16, 2013 strike and directed them to explain why they should
The union filed a PCE before the DOLE and the not be dismissed. The petitioners failed to submit
company submitted all its employees for drug testing. their written explanation thus Biomedica served
Those who refused were threatened with dismissal. notices of termination. SC ruled that petitioners were
The results of the drug test came out and the 7 not afforded procedural due process as Biomedica
employees were found positive for illegal drugs. AER failed to quote said provisions of the company policy
issued a memorandum suspending these employees as to adequately inform them of the nature of the
from work. Out of the 7 employees, only 2 were charges against them. Moreover, the period of 24
allowed to return. As they were securing their medical hours alloted to petitioners to answer the notice were
certificates, they were shocked to receive a letter insufficient as employees should be given a
from AER charging them with insubordination and reasonable opportunity to file a response to the
absence without leave. The union tried to prevent the notice. No hearing was also set. In addition,
transfer of machines which prompted AER to issue a petitioners were also denied substantive due process
memorandum accusing those involved of gross as Biomedica failed to even establish that petitioners
insubordination, and work stoppage. They were indeed violated company rules, failing to even
denied entry into company premises so the affected present a copy of the rules and to prove that
workers staged a picket and filed a complaint for ULP. petitioners were made aware of such regulations.
The workers should be reinstated including those who Petitioners did not stage a mass leave nor did they go
tested positive for illegal drugs despite both parties on strike. Only 5 were absent and for personal
being in pari delicto. AERs fault is obvious from the reasons so there was no intent to go on strike. Thus,
fact that a day after the union filed a PCE before the an ordinary striking worker cannot be terminated for
DOLE, it hit back by requiring all its employees to mere participation in an illegal strike. There must be
undergo a compulsory drug test. Although AER proof that he committed illegal acts during a
argues that the drug test was applied to all its strike."
employees, it was silent as to whether the drug test
was a regular company policy and practice in their 35 VCMC v. Yballe, January 15, 2014
years in the business. Moreover, AER failed to show The President of the Union expressed the union's
proof that the drug test conducted on its employees desire to renew the existing CBA but VCMC told her to
was performed by an authorized drug testing center. first secure the endorsement of the legal counsel of
Like management, the union and the affected the Federation which is the EBR of the hospital's
workers were also at fault for resorting to a concerted employees. Legal counsel of the Federation informed
work slowdown and walking out of their jobs of the hospital that the proposal submitted by Nava was
protest for their illegal suspension. never referred to the Federation. Due to the conflict
between the Federation and the Union, the hospital
Lastly, there was no proof that the workers temporarily suspended collection of union fees. Union
deliberately abandoned their jobs. Abandonment is membership was also suspended on the ground of
totally inconsistent with the immediate filing of a serious violations of the Constitution and By-Laws.
complaint for illegal dismissal. In pari delicto doctrine Thus, several union members launched a series of
55
mass actions. DOLE issued certifications that there is PMI Faculty and Employees Union v. PMI
nothing in the records that Union is a registered LLO.
The Union filed a notice of strike but was denied for
Colleges, June 19, 2016
want of personality but they still conducted a strike The union filed a notice of strike with the NCMB
vote and more strikes. officers. A worker merely against the school for gross violations of several CBA
participating in an illegal strike may not be provisions. It also filed a second notice of strike for
terminated from employment. It is only when he the same violation. On the last day of the cooling off
commits illegal acts during a strike that he may be and strike vote period, they were not allowed to enter
declared to have lost employment status. In contrast, company premises so the union staged a premature
a union officer may be terminated from employment strike. School filed a petition to declare strike illegal
for knowingly participating in an illegal strike or for failure of the union to comply with the valid
participates in the commission of illegal acts during a requirements of a strike. NLRC is incorrect in
strike. Yballe et al. are not entitled to back wages. If admitting the belatedly filed CD which in itself
there is no work performed by the employee there already raises doubts as to its credibility. The School’s
can be no wage or pay unless, of course, the laborer explanation for its belated filing was that it took a
was able, willing and ready to work but was illegally long time to take the depositions of the personnel of
locked out, suspended or dismissed or otherwise the studio(Studio is in Tagbilaran) who took the
illegally prevented from working; but for this footage. This explanation is unacceptable as they
exception to apply, it is required that the strike be have member of the school’s management(School is
legal, which is not the case here. in Bohol) who is a lawyer who can take the
Deposition. In sum, the CD is not credible should not
have been given much consideration.
Tabangao Shell Refinery Employees
Association v. Pilipinas Shell, April 7, The Hongkong Shanghai Banking
2014 Corporation Employees Union v. NLRC,
Union and the company started re-negotiations for a
new CBA wherein the union proposed a 20% annual
January 11, 2016
across the board basic salary increase. After several HSBC announced its implementation of a job
meetings, they failed to reach an amicable evaluation program. The Union demanded the
settlement. During the cooling period, the union suspension of such because it was an unfair labor
members unanimously voted for the holding of a practice. The Union’s concerted activities lasted for
strike. Thus, the company filed a Petition for 11 months. The Union Officers and Members walked
Assumption of Jurisdiction with the Sec. of Labor and out and blocked the entry and exit. HSBC filed for a
Employment. The union, then, filed a pet. for petition for habeas corpus on behalf of its officials
certiorari, alleging that the issue is ULP in the form of and employees whom they airlifted to enable them to
bad faith in bargaining and not the CBA deadlock. In leave the bank premises. It also filed a complaint to
the meantime, the union filed a complaint for ULP declare the strike illegal and thereafter terminated
before the NLRC. SOLE held that there was already a the individual petitioners. Strike was illegal for failure
deadlock although the ground for the 1 st Notice of of the Union to file the notice of strike with the DOLE;
Strike was ULP for bargaining in bad faith. The to observe the cooling-off period; and to submit the
existence of bad faith and deadlock are both results of the strike vote to the NCMB pursuant to
Questions of Fact and evidentiary. “The Secretary of Article 263 of the Labor Code. Article 264(e) of the
the DOLE has been explicitly granted by Article Labor Code expressly enjoined the striking workers
263(g) of the Labor Code the authority to assume engaged in picketing from committing any act of
jurisdiction over a labor dispute causing or likely to violence, coercion or intimidation, or from obstructing
cause a strike or lockout in an industry indispensable the free ingress into or egress from the employer’s
to the national interest, and decide the same premises for lawful purposes, or from obstructing
accordingly. The power of the Secretary of Labor and public thoroughfares.
Employment to assume jurisdiction over this dispute
includes and extends to all questions and While Article 264 authorizes the termination of the
controversies arising from the said dispute, such as, union officers and EEs, it does not remove from the
but not limited to the union’s allegation of bad faith EEs their right to due process. Regardless of their
bargaining. It also includes and extends to the actions during the strike, the EEs remain entitled to
various unresolved provisions of the new CBA such as an opportunity to explain their conduct and why they
compensation. should not be penalized. The failure by HSBC to
strictly observe the twin-notice requirements resulted
in illegal dismissal. HSBC should be held liable for two
types of illegal dismissal- the first type was made

56
without both substantive and procedural due process,
employee of his duties;
while the other was based on a valid cause but lacked (c) Fraud or willful breach by the employee
compliance with procedural due process. To the first of the trust reposed in him by his employer
type belonged the dismissal of the 18 EEs, while the or duly authorized representative;
second type included the rest of the petitioners. (d) Commission of a crime or offense by the
employee against the person of his
employer or any immediate member of his
family or his duly authorized
Security of Tenure representatives; and
(e) Other causes analogous to the
Read D.O. 147-15 foregoing

Article 292(b), Labor Code.


(b) Subject to the constitutional right of workers to
Article 298, Labor Code. Closure of
security of tenure and their right to be protected
establishment and reduction of personnel.- The
against dismissal except for a just and authorized
employer may also terminate the employment of
cause and without prejudice to the requirement of
any employee due to the installation of labor-
notice under Article 283 of this Code, the employer
saving devices, redundancy, retrenchment to
shall furnish the worker whose employment is
prevent losses or the closing or cessation of
sought to be terminated a written notice containing
operation of the establishment or undertaking
a statement of the causes for termination and shall
unless the closing is for the purpose of
afford the latter ample opportunity to be heard and
circumventing the provisions of this Title, by
to defend himself with the assistance of his
serving a written notice on the workers and the
representative if he so desires in accordance with
Ministry of Labor and Employment at least one (1)
company rules and regulations promulgated
month before the intended date thereof. In case of
pursuant to guidelines set by the Department of
termination due to the installation of labor-saving
Labor and Employment. Any decision taken by the
devices or redundancy, the worker affected thereby
employer shall be without prejudice to the right of
shall be entitled to a separation pay equivalent to
the worker to contest the validity or legality of his
at least his one (1) month pay or to at least one (1)
dismissal by filing a complaint with the regional
month pay for every year of service, whichever is
branch of the National Labor Relations Commission.
higher. In case of retrenchment to prevent losses
The burden of proving that the termination was for
and in cases of closures or cessation of operations
a valid or authorized cause shall rest on the
of establishment or undertaking not due to serious
employer. The Secretary of the Department of
business losses or financial reverses, the
Labor and Employment may suspend the effects of
separation pay shall be equivalent to one (1)
the termination pending resolution of the dispute in
month pay or at least one-half (1/2) month pay for
the event of a prima facie finding by the
every year of service, whichever is higher. A
appropriate official of the Department of Labor and
fraction of at least six (6) months shall be
Employment before whom such dispute is pending
considered one (1) whole year.
that the termination may cause a serious labor
dispute or is in implementation of a mass lay-off.

Article 299, Labor Code. Disease as ground for


termination. - An employer may terminate the
Article 294, Labor Code. Security of tenure. - In
services of an employee who has been found to be
cases of regular employment, the employer shall
suffering from any disease and whose continued
not terminate the services of an employee except
employment is prohibited by law or is prejudicial to
for a just cause or when authorized by this Title. An
his health as well as to the health of his co-
employee who is unjustly dismissed from work shall
employees: Provided, That he is paid separation
be entitled to reinstatement without loss of
pay equivalent to at least one (1) month salary or
seniority rights and other privileges and to his full
to one-half (1/2) month salary for every year of
backwages, inclusive of allowances, and to his
service, whichever is greater, a fraction of at least
other benefits or their monetary equivalent
six (6) months being considered as one (1) whole
computed from the time his compensation was
year.
withheld from him up to the time of his actual
reinstatement.

Article 300, Labor Code. Termination by


employee. - (a) An employee may terminate
Article 297, Labor Code. Termination by
without just cause the employee-employer
employer. - An employer may terminate an
relationship by serving a written notice on the
employment for any of the following causes:
employer at least one (1) month in advance. The
(a) Serious misconduct or willful
employer upon whom no such notice was served
disobedience by the employee of the lawful
may hold the employee liable for damages.
orders of his employer or representative in
connection with his work;
(b) An employee may put an end to the relationship
(b) Gross and habitual neglect by the

57
without serving any notice on the employer for any
of the following just causes: Violation of this provision is hereby declared
1. Serious insult by the employer or his unlawful and subject to the penal provisions under
representative on the honor and person of the Article 288 of this Code.
employee;
2. Inhuman and unbearable treatment accorded
the employee by the employer or his
representative; Just Causes
3. Commission of a crime or offense by the
employer or his representative against the person Salas v Aboitiz One, June 27, 2008
of the employee or any of the immediate members Salas was tasked with monitoring and maintaining
of his family; and
4. Other causes analogous to any of the foregoing. the availability and supply of Quickbox needed by
Aboitiz in its day-to- day operations. However, Salas
had run out of large quickbox, hampering the
Article 301, Labor Code. When employment not business operations. Aboitiz then wrote Salas a
deemed terminated. - The bona-fide suspension of memorandum requiring him to explain in writing why
the operation of a business or undertaking for a he should not be disciplinarily dealt with for his (i)
period not exceeding six (6) months, or the failure to monitor the stock level of Large Quickbox
fulfillment by the employee of a military or civic which led to inventory stock out; and (ii) failure to
duty shall not terminate employment. In all such
report to his immediate superior the Large Quickbox
cases, the employer shall reinstate the employee
to his former position without loss of seniority problem when the stock level was already critical. An
rights if he indicates his desire to resume his work administrative hearing was conducted. Aboitiz sent
not later than one (1) month from the resumption him a decision notice, terminating him for loss of
of operations of his employer or from his relief from trust and confidence. Salas then requested for a
the military or civic duty. reconsideration of the decision through a letter,
asking if he could instead avail of the early retirement
plan, having worked for Aboitiz for ten years already
Article 302, Labor Code. Retirement. - Any but this was denied. Claiming termination without
employee may be retired upon reaching the
cause, Salas filed
retirement age established in the collective
bargaining agreement or other applicable with the Labor Arbiter a complaint against Aboitiz for
employment contract. illegal dismissal. Aboitiz responded that there was
valid termination, asserting that Salas was dismissed
In case of retirement, the employee shall be entitled to for just cause and with due process, Salas having
receive such retirement benefits as he may have earned willfully breached his duty when he ran out of Large
under existing laws and any collective bargaining Quickbox, justifying the termination of his
agreement and other agreements: Provided, however,
That an employee’s retirement benefits under any employment.
collective bargaining and other agreements shall not be
less than those provided therein. Gross negligence connotes want or absence of or
failure to exercise slight care or diligence, or the
In the absence of a retirement plan or agreement entire absence of care. To warrant removal from
providing for retirement benefits of employees in service, the negligence should not merely be gross,
the establishment, an employee upon reaching the
but also habitual. Although it was Salas' duty to
age of sixty (60) years or more, but not beyond
sixty-five (65) years which is hereby declared the monitor and maintain the availability and supply of
compulsory retirement age, who has served at Quickbox, records show that Salas had made a
least five (5) years in the said establishment, may requisition as early as May 21, 2003, even making
retire and shall be entitled to retirement pay several follow- ups. If there is anything that Salas can
equivalent to at least one-half (1/2) month salary be faulted for, it is his failure to promptly inform his
for every year of service, a fraction of at least six immediate
(6) months being considered as one whole year.
supervisor of the non-delivery of the requisitioned
Unless the parties provide for broader inclusions, items. Nevertheless, such failure did not amount to
the term one-half (1/2) month salary shall mean gross neglect of duty or to willful breach of trust,
fifteen (15) days plus one-twelfth (1/12) of the 13th which would justify his dismissal from service.
month pay and the cash equivalent of not more Moreover, there appears nothing to suggest that
than five (5) days of service incentive leaves. Salas’ position was a highly or even primarily
confidential position, so that he can be removed for
Retail, service and agricultural establishments or
operations employing not more than ten (10) loss of trust and confidence by the employer.
employees or workers are exempted from the Aboitiz’s reliance on the past offenses of Salas for his
coverage of this provision. eventual dismissal is likewise unavailing. The correct

58
rule has always been that such previous offenses filed an illegal dismissal case against SMC. The SC
may be used as valid justification for dismissal from held that SMC failed to prove the falsification charge
work only if the infractions are related to the against Ibias. The testimonies presented against Ibias
subsequent offense upon which the basis of were uncorroborated and self-serving. As to
termination is decreed. unauthorized absences, Ibias confessed that some of
his absences were without permission. The dismissal
RB Michael Press v Galit, February 13, of Ibias was well within the purview of petitioner’s
management prerogative. SMC was acting within its
2008 right as employer when it dismissed Ibias. An
Galit was employed by RB Michael Press as an offset employer has the prerogative to prescribe reasonable
machine operator. He was ordered to render OT rules and regulation necessary for the proper conduct
service to comply with a job order deadline but of its business, to provide certain disciplinary
refused. The following day, he reported to work but measures, and to assure that the same are complied
the company told him not to work and return in the with.
afternoon for the hearing. He was thereafter
terminated so he filed a complaint for illegal dismissal
and money claims. For willful disobedience to be a
LBC Express v. Mateo, June 9, 2009
valid cause for dismissal, these two elements must Mateo was tasked to deliver and pick up packages
concur: (1) the employee’s assailed conduct must from LBC to its customers and was assigned a
have been willful, that is, characterized by a wrongful Kawasaki motorcycle. He parked said vehicle and
and perverse attitude; and (2) the order violated switched off the engine but took the key with him, but
must have been reasonable, lawful, made known to did not lock the steering wheel. When he returned,
the employee, and must pertain to the duties which the vehicle was gone. The management issued him a
he had been engaged to discharge.In the present notice of termination which led to im filing a
case, there is no question that petitioners’ order for complaint for illegal dismissal. Mateo was
respondent to render overtime service to meet a undisputedly negligent when he left the motorcycle
production deadline complies with the second along Burke Street in Escolta, Manila without locking
requisite. The fact that respondent refused to provide it despite clear, specific instructions to do so. He
overtime work despite his knowledge that there is a deliberately did not heed the employers very
production deadline that needs to be met, and that important precautionary measure to ensure the
without him, the offset machine operator, no further safety of company property. The memorandum
printing can be had, shows his wrongful and perverse directing Mateo to be present for
mental attitude; thus, there is willfulness. investigation clearly provided the reasons or grounds
for Mateo's investigation. As stated there, the
Under the twin notice requirement, the employees grounds were the alleged carnapping of the
must be given two (2) notices before his employment motorcycle and the alleged pilferage of a package.
could be terminated: (1) a first notice to apprise the Nothing could be clearer. What the law merely
employees of their fault, and (2) a second notice to requires is that the employee be informed of the
communicate to the employees that their particular acts or omissions for which his dismissal is
employment is being terminated. Not to be taken sought. that. Mateo was thereafter given the
lightly of course is the hearing or opportunity for the opportunity to explain his side and was handed the
employee to defend himself personally or by counsel requisite second notice (of termination). Thus,
of his choice. In view of the lack of due process, procedural due process was complied with.
termination of respondent was railroaded in serious
breach of his right to due process. And as a Genuino v. NLRC, December 4, 2007
consequence of the violation of his statutory right to Genuino was employed by Citibank as Treasury Sales
due process and following Agabon, petitioners are Division Head. She was sent a letter that she was
liable jointly and solidarily to pay nominal damages. under preventive suspension and after investigation,
her employment was terminated on the ground of
San Miguel Corporation v NLRC, April serious misconduct, commission of a crime against
the bank and willful breach of the trust reposed upon
16, 2008 her. SC held that she was dismissed for just cause but
According to the company's policy on employee without the observance of due process. The first
conduct, it punishes falsification of company records written notice to be served on the employees should
with discharge for the first offense if the offender contain the specific causes or grounds for termination
himself or somebody else benefits from the against them, and a directive that the employees are
falsification. During investigation, Ibias testified that given the opportunity to submit their written
his absences were sick leaves and denied the explanation within a reasonable period. Reasonable
falsification charge but still, he was dismissed. Ibias
59
opportunity should be construed as a period of at required permission before she taught in other
least five (5) calendar days from receipt of the notice schools. Thus, it failed to prove that misconduct was
to give the employees an opportunity to study the induced by wrongful intent. Also, one of the
accusation against them. After serving the first provisions therein categorically stated
notice, the employers should schedule and conduct a that should a violation of any of the terms and
hearing or conference. After determining that conditions thereof be committed, the penalty that will
termination of employment is justified, the employers be imposed would either be suspension or dismissal
shall serve the employees a written notice of from employment. Thus, contrary to its position from
termination. Here, the letters sent by Citibank did not the beginning, SSC-R clearly had the discretion to
identify the particular acts / omissions allegedly impose a lighter penalty of suspension and was not at
committed by Genuino. The first notice informing the all compelled to dismiss Moreno under the
employee of the charges should neither be pro-forma circumstances, just because the Faculty Manual said
nor vague. It should set out clearly what the so.
employee is being held liable for.
Janssen Pharmaceutica v. Silayro,
Bughaw v. Treasure Island, March 28, February 26, 2008
2008 Silayro was terminated for granting unauthorized
An employee of Treasure Island was caught in premium goods to and from customers and
flagrante delicto while in possession of shabu and dishonesty in accomplishing the DCR. In termination
implicated Bughaw in said crime. Three letters were cases, the burden of proof rests with the employer to
sent terminating his employment for using illegal show that the dismissal is for just and valid cause.
drugs within company premises. Bughaw filed for Failure to do so would necessarily mean that the
illegal dismissal. The requirements for the lawful dismissal was not justified and therefore was illegal.
dismissal of an employee are twofold, the substantive Dishonesty is a serious charge, which the employer
and the procedural aspects. Not only must the must adequately prove, especially when it is the basis
dismissal be for a just or authorized cause, the for termination. In this case, petitioner had not been
rudimentary requirements of due process — notice able to identify an act of dishonesty,
and hearing— must, likewise, be observed before an misappropriation, or any illicit act, which the
employee may be dismissed. The charge of drug respondent may have committed in connection with
abuse inside the company's premises and during the erroneously reported product samples. No willful
working hours against petitioner constitutes serious violation was committed but only a failure to exercise
misconduct, which is one of the just causes for a standard of care. The superficial compliance with
termination. Furthermore, the burden on respondent two notices and a hearing in this case cannot be
to present clear and unmistakable proof that considered valid where these notices were issued and
petitioner was duly served a copy of the notice of the hearing made before an offense was even
termination but he refused receipt. Where the committed.
dismissal is for just cause, as in the instant case, the
lack of statutory due process should not nullify the Suico v. NLRC, January 30, 2007
dismissal or render it illegal or ineffectual. However, In all three cases, notices were sent to the
the employer should indemnify the employee for the complainants in order for them so submit a written
violation of his right to procedural due process. and notarized explanation for their actions and why
they should not be terminated. However, they did not
Moreno v. San Sebastian College, March submit any written explanation but only gave a letter
28, 2008 informing the company that they are electing the
Moreno was a member of the permanent college their right to a formal hearing. Nevertheless, because
faculty who was terminated for her unauthorized the complainants did not submit a written
external teaching engagements. In order to explanation, the company considered this as a waiver
constitute serious misconduct which will warrant the of their right to a hearing and dismissed them for
dismissal of an employee under paragraph (a) of serious misconduct.
Article 282 of the Labor Code, it is not sufficient that
the act or conduct complained of has violated some PLDT violated the requirements of due process under
established rules or policies. It is equally important the Labor Code when it dismissed said employees
and required that the act or conduct must have been without heeding their request for the conduct of a
performed with wrongful intent. SSC-R merely formal hearing as provided for under PLDT Systems
anchored Moreno’s alleged bad faith on the fact that Practice and prior to submission of their respective
she had full knowledge of the policy that was violated answers to the charges against them. The first
and that it was relatively easy for her to secure the notices sent to Suico, et al. set out in detail the
60
nature and circumstances of the violations imputed to constitute willful breach of the trust reposed in him
them, required them to explain their side and by the employer and his a ground for termination.
expressly warned them of the possibility of their
dismissal should their explanation be found wanting. SAMASAH-NUWHRAIN v. VA Magsalin,
The last notices informed Suico, et al. of the decision
to terminate their employment and cited the
June 6, 2011
evidence upon which the decision Angelito Caragdag, a waiter and a director of the
was based. These two notices would have sufficed union, was dismissed due to: (1) refused to be
had it not been for the existence of Systems Practice frisked; (2) barged in while his immediate supervisor
No. 94016. Under Systems Practice No. 94016, PLDT was about to counsel 2 staff members; (3) He left his
granted its employee the alternative of either filing a work assignment during official hours w/o permission.
written answer to the charges or requesting for Voluntary Arbitrator found his dismissal legal but
opportunity to be heard and defend himself with the granted him financial assistance of 100K. The SC held
assistance of his counsel or union representative, if that the grant of severance compensation shall be
he so desires. Suico, et al. exercised their option allowed only when the cause of the dismissal is other
under Systems Practice No. 94016 by requesting that than serious misconduct or for causes which reflect
a formal hearing be conducted. PLDT is bound to adversely on the employee’s moral character.
comply with the Systems Practice.
Lopez v. Alturas Group of Companies,
Perez & Doria v. PT&T, April 7, 2009 April 11, 2011
Acting on an alleged unsigned letter, the company Lopez, a truck driver, wa dismissed for smuggling
conducted an investigation on Perez and Doria scrap iron using the company’s truck. Company filed
(shipping clerk and supervisor respectively) regarding a case for qualified theft. On the other hand, Lopez
anomalous transactions in the Shipping section when sued for illegal dismissal. Two Facets of Dismissal: (1)
they jacked up the value of the freight costs and the legality of the act of dismissal (substantive due
duplicates of the shipping docs allegedly showed process); (2) legality of the manner of dismissal
traces of tampering. Petitioners were placed in (procedural due process). As to substantive,
preventive suspension for 30 days, extended another petitioner was entrusted with the company’s goods
30 days before they were dismissed. and property, and its handling and protection. Hence,
even if he did not occupy a managerial position, he
The SC held that they were dismissed without just can be said to e holding a position of responsibility. As
cause and w/o due process. Petitioners failed to to procedural, he was given opportunity to explain his
establish loss of trust and confidence because they side when he was informed of the charges against
relied on bare allegations. Willful breach by the him and required to submit written explanation which
employee of the trust reposed in him by his employer he complied. That there was no hearing is of no
or duly authorized representative is a just cause for moment.
termination. However, confidence should not be
simulated. It should not be used as a subterfuge for Concepcion v. Minex Import
causes which are improper, illegal, or unjustified. The
burden of proof rests on the employer to establish
Corporation, January 24, 2012
that the dismissal was for a just cause in view of the Concepcion was the supervisor of Minex’s kiosk. One
security of Tenure under the Constitution and the weekend their total sales reached P50K so petitioner
Labor Code. wrapped the amount in a bag and deposited it in the
drawer. The next day, money was gone. She reported
the incident to her Manager who came with two
Bacolod-Talisay Realty v. Dela Cruz, April policemen who arrested her. The RTC found probable
30, 2009 cause in the THEFT case filed against her. The
Dela Cruz was dismissed on charges of payroll conviction of an employee in a criminal case is not
padding, selling canepoints w/o the consent of indispensable to warrant his dismissal by his
management and misappropriating proceeds. employer. If there is sufficient evidence to show that
Investigation revealed that there were farm workers the employee has been guilty of a breach of trust, or
whose names were entered into the payroll even that his employer has ample reason to distrust him, it
though they have not rendered service and while cannot justly deny to the employer the authority to
Dela Cruz said he would return the money, he did not. dismiss such employee.
The SC held that Bacolod Talisay was able to establish
with substantial evidence that just cause existed for
termination of employment. The acts committed

61
Prudential Guarantee & Assurance was not wearing his ID. Dongon reassured the guard
that he will secure a special permission from the
Labor Union v. NLRC, June 13, 2012 management but instead, he lent his ID to Villaruz to
The company conducted an on-the-spot security be able to obtain clearance for the release of the
check in the IT department beginning with Vallota’s goods they will deliver to clients. The SC held that
(Junior Programmer and Union BOD) computer. They willfulness must be attended by a wrongful and
found a folder named “MAA”. Sensing that Vallota perverse mental attitude rendering the employee’s
was being singled out, the Union insisted that all the act inconsistent with proper subordination. It is
computers be checked. She was eventually implied that in every act of willful disobedience, the
dismissed. The SC held that to be a valid ground for erring employee obtains undue advantage
dismissal, loss of trust and confidence must be based detrimental to the business interest of the employer.
on willful breach of trust and founded on clearly In this case, petitioner’s motive in lending his
established facts. A breach is willful if it is done company ID to Villaruz was to benefit the company.
intentionally, knowingly and purposely without Thus, his dismissal was plainly unwarranted.
justifiable excuse, as distinguished from an act done
carelessly, thoughtlessly, heedlessly or inadvertently.
It must rest on substantial grounds and not on the
Alilem Credit Cooperative v. Bandiola,
employer’s arbitrariness, whims, caprices, or February 25, 2013
suspicion; otherwise, the employee would remain Petitioner’s BOD received a letter from a certain
eternally at the mercy of the employer. Further, in Napoleon reporting the alleged immoral conduct and
order to constitute a just cause, the act complained unbecoming behavior of respondent for his illicit
of must be work-related and show that the employee relationship with Thelma, Napoleon’s sister. They
is unfit to continue working for the employer. both denied the allegation. Petitioner’s investigation
revealed that they were having an extra-marital
Cosmos Bottling Co. v. Fermin, June 20, affair. Engaging in illicit marital affairs is a ground for
termination of employment not only under the law
2012 but even under the Personnel Policy of the petitioner.
Wilson Fermin was accused of stealing his fellow
employee’s phone which was found in his locker.
Thereafter, Braga, the owner of the phone, withdrew
Cavite Apparel v. Marquez, February 6,
his complaint and believed that it was a prank but the 2013
company still dismissed him. Theft committed against Michelle was dismissed for HABITUAL ABSENTEEISM
a co-employee is considered as a case analogous to for being AWOL on 3 different occasions despite
SERIOUS MISCONDUCT for which the penalty of showing of medical certificates. But the SC held that
dismissal from service may be meted out to the her absences are not habitual. Neglect of duty, to be
erring employee. a ground for dismissal, must be both gross and
habitual. GROSS NEGLIGENCE implies ant of care in
Sampaguita Auto Transport v. NLRC & the performance of one’s duties. HABITUAL NEGLECT
imparts repeated failure to perform one’s duties for a
Sagad, January 30, 2013 period of time, depending on the circumstances.
Sagad was hired as a bus driver. An evaluator was Here, 4 absences in her 6 years of service cannot be
deployed in guise of a passenger to observe driver’s considered gross and habitual neglect of duty,
work performance and results lead to him being a especially so since the absences were spread out
reckless driver. Conductors were also asked to over a 6-month period.
comment on Sagad’s work and it was revealed that
he cheats by way of an unreported early bus trip, and
was involved in a hit-n-run accident.
Esguerra v. Valle Verde, June 13, 2012
Esguerra was hired as Cost Control Supervisor. The
The irregularities or infractions committed by Sagad management found out that proceeds which had
in connection with his work constitutes serious been remitted to the accounting department for an
misconduct or at the very least, conduct analogous to event were lacking. Also, there were unauthorized
serious misconduct. He also committed breach of charges of food on one of the participants. The SC
trust and confidence of his employer. held that as Cost Control Supervisor, she had the
duty to remit to the accounting department the cash
sales proceeds from every transaction she was
Dongon v. Rapid Movers, August 28, assigned to. This is not a routine task any employee
2013 can perform Thus, Esguerra occupies a position of
Tanduay’s security guard called the attention of trust and confidence. Any breach of the trust imposed
Dongon (truck helper leadman) for Villaruz (driver) upon her can be a valid ground for dismissal.

62
analogous to “gross neglect of duty”, a just cause of
Leus v. St. Scholastica’s College- dismissal under Art. 297 of the LC.

Westgrove, January 28, 2015


Cheryll Santos Leus was hired as Assistant to the
PJ Lhuillier, Inc. v. Camacho, February
Director of the Lay Apostolate and Community 22, 2017
Outreach Directorate. Leus and her boyfriend Camacho brought along an unauthorized person, a
conceived a child out of wedlock. She was asked to non-employee during the QTP operation (pull-out of
resign but she refused. Leus and her boyfriend got foreclosed items). Camacho explained that he
married. She was dismissed for serious misconduct. requested his mother’s personal driver to drive him to
The fact of Leus’ pregnancy out of wedlock, without Pangasinan.He was dismissed for it was a clear
more, is not enough to characterize the petitioner’s violation of an established company policy designed
conduct as disgraceful or immoral. The totality of the to safeguard the pawnshop against robberies and
circumstances surrounding the conduct alleged to be untoward incidents.
disgraceful or immoral must be assessed against the
prevailing norms of conduct. It is not the totality of The SC upheld his dismissal. He was primarily
the circumstances surrounding the conduct per se responsible for administering and controlling the
that determine whether the same is disgraceful or operations of branches in his assigned area, ensuring
immoral, but the conduct that is generally accepted cost efficiency, manpower productivity. He was also
by society as respectable or moral. responsible for monitoring the overall security and
integrity in the area, including branch personnel
Saudi Arabian Airlines v. Rebesencio, safety. It can be deduced that Camacho held a
managerial position and enjoyed full trust and
January 14, 2015 confidence of his superiors. His act was without
Respondents (flight attendants) were separated from justification. The fact that an employer did not suffer
service on for being pregnant despite having pecuniary damage will not obliterate the
processed their maternity leaves. Saudia asked them respondent’s betrayal of trust and confidence
to resign but they refused. They received a letter reposed in him by his employer.
which had the word “RESIGNATION”. Constructive
dismissal is the cessation of work because continued
employment is rendered impossible, unreasonable or
Alaska Milk Corporation v. Ponce, July
unlikely, as an offer involving a demotion in rank or a 26, 2017
diminution in pay and other benefits. It is tantamount Ponce (Director for Engineering Services) was
to involuntary resignation due to harsh, hostile, and dismissed for his abrasive behavior towards his
unfavorable conditions set by the employer. The colleagues, his failure to provide updates on several
gauge is whether a reasonable person in the engineering works, and for soliciting official receipts
employee’s position would feel compelled to give up from colleagues in exchange for 5% rebate of the
his employment under the prevailing circumstances. value of receipts given to him. The SC held that
soliciting ORs in exchange of a 5% cash rebate is an
Capin-Cadiz v. Brent Hospital and act inimical to the company’s interests because
Ponce will be reimbursed for expenses he did not
Colleges, February 24, 2016 incur. They consider such act a fraudulent
Petitioner was subject to an indefinite suspension due representation sufficient to erode its trust and
to her alleged immoral act of engaging in premarital confidence.
sexual relations and getting pregnant in the process.
Also, as a condition for her reinstatement, she must
marry her boyfriend. She was illegally dismissed. Allan John Uy Reyes v. Global Beer
[Same doctrine as Leus v. St. Scho] Below Zero, Inc., October 4, 2017
Petitioner was late for work because his son was sick.
Puncia v. Toyota Shaw, June 28, 2016 VP for Operations Vinson Co say won’t listen to his
Puncia was tasked to sell 7 vehicles monthly as quota explanation and told him not to report to work
but for 2 months he was only able to sell 1. A hearing anymore. Petitioner did not report to work on the
was conducted but he failed to attend such despite following days and waited for further instructions.
due notice. Toyota dismissed him on the ground of Vinson asked petitioner to resign but he refused. He
insubordination for his failure to attend the scheduled filed for illegal dismissal. Respondent claims that
hearing and justify his absence. Puncia’s repeated petitioner was not dismissed and that he simply
failure to perform his duties (reaching his quota) falls stopped reporting to work.
under the concept of gross inefficiency. In this
regard,case law instructs that “gross inefficiency” is
63
Abandonment requires deliberate, unjustified refusal Manatad v. PT&T, March 7, 2008
of the employee to resume his employment, without Employee was dismissed based on retrenchment.
intention of returning. For abandonment to exist, two Court held that it was a VALID retrenchment. Firstly,
factors must be present: (1) failure to report for work the losses expected should be substantial and not
or absence w/o valid or justifiable reason; (2) a clear merely de minimis in extent. Secondly, the
intention to sever employer-employee relationship substantial loss apprehended must be reasonably
with the second element as the more determinative imminent, as such imminence can be perceived
factor being manifested by some overt acts. In this objectively and in good faith by the employer. thirdly,
case, no such abandonment was proven. be reasonably necessary and likely to effectively
prevent the expected losses. Lastly, alleged losses if
already realized, and the expected imminent losses
Authorized Causes sought to be forestalled, must be proved by sufficient
and convincing evidence. In the case at bar,
Andrada v. NLRC, December 28, 2007 respondent experienced serious financial crises as
Employees were terminated based on retrenchment. shown in the financial statements audited by
Court held that there was NO valid retrenchment. independent auditors, SGV & Co. and Alba Ledesma &
Retrenchment is an exercise of management’s Co. It is unlikely therefore that respondent was just
prerogative to terminate the employment of its feigning business losses in order to ease out
employees en masse, to either minimize or prevent employees. In fact, even granting arguendo that
losses, or when the company is about to close or respondent was not experiencing losses, it is still
cease operations for causes not due to business authorized by Article 283 of the Labor Code to cease
losses. It is an authorized cause. The requirements its business operations. Explicit in the said provision
are that: is that closure or cessation of business operations is
(1) It is undertaken to prevent losses, which are allowed even if the business is not undergoing
not merely de minimis, but substantial, economic losses.
serious, actual, and real, or if only expected,
are reasonably imminent as perceived
Linton Commercial v. Hellera, October
objectively and in good faith by the employer;
(2) The employer serves written notice both to 10, 2007
the employees and the DOLE at least one Workers filed a Complaint for illegal reduction of
month prior to the intended date of workdays with the NLRC. They pointed out that Linton
retrenchment; and implemented the reduction of work hours without
(3) The employer pays the retrenched employees observing Article 283 of the Labor Code, which
separation pay equivalent to one month pay required submission of notice thereof to DOLE one
or at least month pay for every year of month prior to the implementation of reduction of
service, whichever is higher. The Court later personnel. Linton had failed to establish enough
added the requirements that the employer factual basis to justify the necessity of a reduced
must use fair and reasonable criteria in workweek and present adequate, credible and
ascertaining who would be dismissed and x x persuasive evidence that it was indeed suffering, or
x retained among the employees and that the would imminently suffer, from drastic business losses.
retrenchment must be undertaken in good for retrenchment to be justified, any claim of actual
faith. Except for the written notice to the or potential business losses must satisfy the following
affected employees and the DOLE, non- standards:
compliance with any of these requirements (1) the losses incurred are substantial and not
render[s] the retrenchment illegal. de minimis;
(2) the losses are actual or reasonably
In the present case, Company failed to show its imminent;
financial condition prior to and at the time it enforced (3) retrenchment is reasonably necessary and
its retrenchment program. Another authorized cause is likely to be effective in preventing expected
is redundancy. It exists when the number of losses; and
employees is in excess of what is reasonably (4) the alleged losses, if already incurred, or
necessary to operate the business. Here, Company the expected imminent losses sought to be
did not even bother to illustrate or explain in detail forestalled, are proven by sufficient and
how and why it considered petitioners positions convincing evidence.
superfluous or unnecessary.

64
AMA Computer College v. Garcia, April the decision to terminate must not be based on
salary.
14, 2008
Employees were dismissed based on redundancy. There is no clear proof that respondent’s services are
Court held that there was ILLEGAL DISMISSAL. in excess of the Company’s reasonable demands and
Redundancy exists when the service capability of the requirements; and that there is no other alternative
workforce is in excess of what is reasonably needed available to the Company except to dismiss
to meet the demands of the business. Requisites are: respondent. The superfluity of [respondents] position
1) Good faith if ER in abolishing the redundant has not been established. There has been no previous
position; and over-hiring of employees. On the contrary, the
2) Fair and reasonable criteria in ascertaining Company had already terminated nine (9) employees.
what positions are to be declared redundant There is no proof of decreased volume of business.
and accordingly abolished. Court explained that a position is superfluous as an
There must be adequate proof of such redundancy; it outcome of diff. Factors: over-hiring of workers,
is not enough to allege that company has become decrease in volume of business, dropping of a
overmanned. Also, there was no proof that particular product line/service activity.
reasonable criteria was applied. Some criteria are:
- Less preferred status (e.g. temporary EE)
- Efficiency
PT & T v. NLRC, April 15, 2005
- Seniority PT&T realized that it needed to undertake measures
against losses to prevent the company from going
bankrupt, particularly by reducing its workforce from
GSWU-NAFLU-KMU v. NLRC, October 17, 2,500 to 900 employees. It decided to implement a
2006 temporary retrenchment of some employees
Employees were terminated due to serious business dubbed as Temporary Staff Reduction Program
losses. EEs filed a suit for illegal dismissal, ULP & (TSRP) lasting for not more than five and a half (5)
money claims. They argued, among others, that the months. EEs now question this.
notice requirement was not complied with. Court held
that The mere posting on the company bulletin Court held that there was NO valid retrenchment
board does not, meet the requirement under program. First, PT&T sufficiently complied with its
Article 283 of serving a written notice on the burden to prove that it incurred substantial losses to
workers. The purpose of the written notice is to warrant the exercise of the extreme measure of
inform the employees of the specific date of retrenchment to prevent the company from totally
termination or closure of business operations, and going under. However, while an employer may have a
must be served upon them at least one month before valid ground for implementing a retrenchment
the date of effectivity to give them sufficient time to program, it is not excused from complying with the
make the necessary arrangements. To meet the required written notice served both to the employee
foregoing purpose, service of the written notice concerned and the DOLE at least one month prior to
must be made individually upon each and every the intended date of retrenchment. The purpose of
employee of the company. this requirement is not only to give employees some
time to prepare for the eventual loss of their jobs and
Also, Court held that EEs are entitled to separation their corresponding income, look for other
pay when there was retrenchment to prevent losses employment and ease the impact of the loss of their
or in cases of closure, BUT NOT due to losses or jobs but also to give the DOLE the opportunity to
financial reverses. ascertain the verity of the alleged cause of
termination. Compliance with the one-month notice
Dickinson Philippines v. NLRC, rule is mandatory regardless of whether the
retrenchment is temporary or permanent. This was
November 15, 2005 not complied with.
EE was terminated based on redundancy. Court held
that he was ILLEGALLY DISMISSED. The principal
reason why respondent’s position was declared
Oriental Petroleum v. Fuentes, October
redundant is the fact that he was the highest paid 14, 2005
employee. The Company’s main purpose in EEs were terminated due to retrenchment.
terminating [respondent] was to cut down expenses Retrenchment was allegedly based on serious
and it did so by dismissing him in one fell swoop, financial losses of the company. Court held that it was
camouflaging its malice by using the ground of invalid for failure to comply with substantive and
redundancy. Thus was violated the Company rule that procedural requirements. Retrenchment being a
measure of last resort, they should have been able to
65
demonstrate that it expected no abatement of its efficiency, seniority, physical fitness, age, and
losses in the coming years; that petitioner was not financial hardship for certain workers.
able to prove that retrenchment was resorted to only
after less drastic means have been tried and found SC said that PAL failed to comply with element 1, 4
wanting. Also, as regards the rule that reasonable and 5. Because First, the fact that PAL underwent
criteria be used in effecting retrenchment, such as corporate rehabilitation does not automatically justify
but not limited to: (a) less preferred status (e.g., the retrenchment of its cabin crew personnel. Second,
temporary employee); (b) efficiency; and (c) the hiring of new employees and subsequent rehiring
seniority, Oriental failed to demonstrate its of "retrenched" employees constitute bad faith.
transparency and good faith in the implementation of Lastly, by discarding the cabin crew personnel’s
its decision to retrench respondents. Moreover, with previous years of service and taking into
more reason should the retrenchment in this case be consideration only 1 year’s worth of job performance
held invalid, considering that petitioner utterly failed for evaluation, PAL did away the concept of seniority,
to show that it had any standard at all in selecting the loyalty and past efficiency, and treated all cabin
employees to be retrenched. attendants as if they were on equal footing, with no
one more senior than the other.
FASAP v. PAL, July 22, 2008 and October
2, 2009 General Milling Corp. v. Viajar, January
PAL applied Section 112 of the PAL-FASAP CBA in 30, 2013
retrenching cabin crew personnel: that retrenchment GMC terminated EEs based on redundancy. Court held
shall be based on the individual employee’s efficiency that there was no sufficient basis. There is no
rating and seniority. PAL determined the cabin crew showing that GMC made an evaluation of the existing
personnel efficiency ratings through an evaluation of positions and their effect to the company. Neither did
the individual cabin crew member’s overall GMC exert efforts to present tangible proof that it was
performance for the year 1997 alone, it did not experiencing business slow down or over hiring. .On
consider their performance during the previous years. the other hand, the respondent presented proof that
the petitioner had been hiring new employees while it
The burden falls upon the employer to prove was firing the old ones, negating the claim of
economic or business losses with sufficient redundancy.
supporting evidence. Its failure to prove these
reverses or losses necessarily means that the Am-Phil Food v. Padilla, October 1, 2014
employee’s dismissal was not justified. Any claim of
EE was retrenched due to serious business losses. He
actual or potential business losses must satisfy
argued that there are 6 contractual employees while
certain established standards before any reduction of
he was a regular employee with good evaluation
personnel becomes legal. These are:
record. Moreover, Am-Phil was still hiring new
(1) That retrenchment is reasonably
employees. Court held that there was NO valid
necessary and likely to prevent business
retrenchment for noncompliance with substantive
losses which are substantial, serious, actual
and procedural requirements.
or real;
● Am-Phil’s audited financial statements were
(2) That the employer served written notice
deemed unworthy of consideration for they
both to the employees and to the DOLE at
were not presented when the evidence in
least 1 month prior to the intended date of
chief were considered by the LA.
retrenchment;
● Did not serve a written notice w/ DOLE 1
(3) That the employer pays the retrenched
month before intended date of retrenchment.
employees separation pay equivalent to 1
● While it is true that Am-Phil gave Padilla
month pay or at least ½ month pay for every
separation pay, compliance with none but
year of service, whichever is higher;
one (1) of the many requisites for a valid
(4) That the employer exercises its
retrenchment does not absolve Am-Phil of
prerogative to retrench employees in good
liability.
faith for the advancement of its interest and
not to defeat or circumvent the employees’
right to security of tenure; and, SPI Technologies v. Mapua, April 7, 2014
(5) That the employer used fair and Mapua was terminated due to redundancy. Court held
reasonable criteria in ascertaining who would that she was illegally dismissed. SPI did not even
be dismissed and who would be retained bother to differentiate the two positions allegedly
among the employees, such as status, redundant. Furthermore, on the assumption that the
functions of a Marketing Communications Manager
are different from that of a Corporate Development
66
Manager, it was not even discussed why Mapua was Uniwide Sales v. NLRC, February 29,
not considered for the position. Also, it is quite
peculiar that two kinds of notices were served to
2008
Mapua, thereby causing confusion as regards the Kawada, was terminated on the grounds of violation
effectivity of Mapua’s termination. Hence, SPI must of company rules, abandonment of work, and loss of
shoulder the consequence of causing the confusion trust and confidence. She received successive
brought by the variations of termination letters given memoranda accusing her of different infractions. The
to Mapua. Court held that Kawada was NOT constructively
dismissed. Constructive dismissal is the cessation of
work because continued employment is rendered
PNCC Skyway Corporation v. Secretary
impossible, unreasonable or unlikely; when there is
of Labor and Employment, February 6, demotion in rank, or diminution in pay or both; or
2018 when a clear discrimination, insensibility, or disdain
PSC served termination letters to its employees 3 by an ER becomes unbearable to the EE.
days before the full transfer of its operation that will
lead it its closure of operation. It also served a notice The test of constructive dismissal is whether a
of termination to DOLE. Court held that PSC did not reasonable person in the EE’s position would have felt
comply with the procedural requirements, i.e. it did compelled to give up his position under the
not serve the written notice to the employees and to circumstances. It is an act amounting to dismissal but
the DOLE at least 1 month before the intended made to appear as if it were not. Kawada’s bare
date of cessation of business operations. allegations of constructive dismissal without
Purpose of this requirement is to give employees time corroborating evidence cannot be given credence.
to prepare for the eventual loss of their jobs, as well
as DOLE, the opportunity to ascertain the veracity of Norkis Trading v. Genilo, February 11,
alleged cause of termination. 2008
Genilo was appointed as Credit and Collection
Manggagawa ng Komunikasyon sa Manager of Magna Financial Services Group, Norki’s
Pilipinas v. Philippine Long Distance sister company. A special audit team found out that
Genilo forwarded the monthly collection reports of
Telephone Co., April 19, 2017 the NICs under his supervision without checking the
PLDT terminated employees based on redundancy,
veracity of the same. He was placed under 15 days
thus, the Union filed a notice of strike. Court held that
suspension without pay and allowances. Then, he was
termination was valid. Redundancy is backed up by
appointed as Marketing Assistant. He failed a case for
substantial evidence that the services of employees
illegal suspension and constructive dismissal, inter
are in excess of what is required by the company.
alia. Court held that he was constructively
PLDT presented data on the decreasing volume of
dismissed. The transfer an EE is prerogative of
received calls by the Operator Services Center for
management, provided that:
years 1996-2002. There was a 72% reduction of
1) There is no demotion in rank of diminution of
demand calls due to growth of wireless
salary, benefits and other privileges; and
communication.
2) The action is not motivated by discrimination,
made in bad faith, or effected as a form of
punishment or demotion without sufficient
Constructive Dismissal/ cause.
Preventive Suspension In this case, there was a reduction in his duties and
responsibilities which amounted to a demotion
Maricalum v. Decorion, April 12 2006
tantamount to constructive dismissal.
Decorion was a regular employee of Maricalum. He
failed to attend a meeting called by the Supervisor,
thus, he was placed under preventive suspension for Fungo v. Lourdes School, July 27, 2007
insubordination. He was placed under disciplinary Fungo questioned the dismissal of her husband from
suspension for 6 months which would include the the same school by using the documents from the
period of his preventive suspension. The Court held filing cabinet that was entrusted to her by the School
that his preventive suspension was INVALID for Treasurer. Thus, the School Treasurer compelled
exceeding the 30-day allowable period of suspension. Fungo to tender her resignation within 30 minutes,
Thus, this amounts to constructive dismissal. otherwise, she will not receive her separation pay.
The Court held that Fungo was constructively
dismissed. Her act of retrieving the document did not

67
constitute a breach of trust and confidence since she Court held that there existed constructive dismissal.
did not show the documents any other person except There is constructive dismissal when there is
to the School Treasurer himself. Fungo was made to cessation of works because continued employment is
resign against her will with threat. Clearly, her rendered impossible or unlikely as an offer involving a
consent was vitiated. demotion or diminution in pay. It is impossible for him
to continue working for an employer who does not
The University of the Immaculate pay his, salaries. The law grants 2 remedies to an
illegally dismissed employee; reinstatement with
Conception v. NLRC, January 26, 2011 back wages or separation pay with backwages. As
Axalan (the EE) attended a seminar, then, she was there are already strained relations between the
asked to explain why she should not be dismissed for parties, then separation pay with back wages is
AWOL. She answered that she held online classes proper.
while she attended the seminar so she could not be
considered absent. Then, Axalan attended another
seminar in Baguio. The Dean wrote to her again
Intec Cebu, Inc. v. CA, June 22, 2016
informing her that 2nd attendance was subjected to a Respondents were hired a production workers of
2nd charge of AWOL. She was suspended for 6 Intec. In 2005, their working days were reduced from
months for each charge. Court held that there is NO 6 days to 24 days. Respondents eventually found out
constructive dismissal as Axalan promptly resumed that contractual workers are hired instead to perform
teaching after the expiration of the suspension their tasks. They were subsequently terminated thus
period. Thus, Axalan cannot claim that she was left filed for a complaint for illegal dismissal. The
with no choice but to quit, a crucial element in a Supreme Court ruled that there exists constructive
finding constructive dismissal. dismissal. There was no reason to implement cost
cutting measure in the form of reduction of
employees. At the time it was implemented, the
Robinsons Galleria/Robinsons company was still earning based on its numbers for
Supermarket Corp. v. Sanchez, January 2006. The hiring of new workers necessarily entailed
19, 2011 costs. In his case, there was no proof submitted that
Ranchez was a probationary employee of Robinsons the newly hired employees were performing work
Galleria for a period of 5 months. 2 weeks after she different from the regular workers.
was hired, she reported to her supervisor a loss of
cash amounting to P20 299. The operations manager Sumifru (Philippines) Corporation v.
of the supermarket ordered that Ranchez be stripped- Baya, April 17, 2017
search but it yielded nothing. Manuel reported the Baya was employed by AMSFC as a supervisor where
matter to the police and she was constrained to he joined the union of supervisors AMSKARBEMCO. He
spend 2 weeks in jail. She later filed a complaint for was reassigned to DFC, the sister company of AMSFC.
illegal dismissal thereafter, petitioner sent a notice of During the transfer, Baya remained an active
termination. The SC held that Ranchez was illegally member of the union while being a member of DFC’s
dismissed. A probationary enjoys security of tenure. supervisory union. 220 ha of the 513 ha banana
Aside from the just and authorized causes of plantation of AMSFC were covered by the
termination, an additional ground is added for comprehensive agrarian reform law. In order to
dismissal of probationary employees i.e. failure to ventur a possible business, the beneficiaries were
qualify as a regular employee in accordance with made to choose which group they wanted to belong,
reasonable standards made known by the employer. AMSKARBEMCO or SAFFPAI (pro-company
In this case, Ranchez was not afforded due process. beneficiaries association). The result yielded that
The haphazard manner in the investigation left they were more inclined to join the first union. AMSFC
responded to cry foul. The company already pre- later learned that the first union choose another
judge her guilt without proper investigation and corporation to enter into a business with. Thus, when
instantly reported her to the police. baya returned to AMSFC, he was given a rank and file
position. The SC held that in constructive dismissal
Dreamland Hotel v. Johnson, March 12, cases, the employer had the burden of proving that
2014 the transfer and demotion of an employee are for a
Dreamland Hotel was looking for a resort manager. valid and legitimate grounds such as genuine
Johnson applied and was eventually hired. He gave a business necessity. Both DFC and AMSFC were both
loan of Php 100, 000 for the completion of the hotel. aware that there was already a lack of supervisory
At the start of his employment, the hotel was not yet position, yet they still ordered Baya’s return.
done. He eventually asked for his salary but was not
given and he was forced to resign. The Supreme
68
Union Security Clause provision; and (3) there is sufficient evidence to
support the union’s decision to expel the employee
from the union. In this case, all requisites were met
Alabang Country Club v. NLRC, February by the company.
14, 2008
ACC and the union entered into a CBA which provided General Milling Corp. v. Casio, March 10,
a union shop clause. A new set of officers were
elected. During the audit, the new officers discovered 2010
that there were irregularities in the recorded entries, Casio et al were regular employees of petitioner.
unaccounted expenses and disbursements and Officers and members of the union issued a resolution
uncollected loans. After explaining their side, the old expelling them from the union because of acts
set of officers were dismissed from the union because inimical to public interest. The union threatened the
of malversation of union funds. The union demanded company that if they do not expel the respondent,
from the corporation to terminate the old officers they will file a ULP case against the company. Thus
because of the union shop clause, thus they were the company dismissed them. The Supreme Court
dismissed. The Supreme Court held that the held that the dismissal was not valid. Under the Code,
termination was valid. Under the Code, an employee an employee may be validly terminated based on: (1)
may be validly terminated based on: (1) just causes; just causes; (2) authorized causes; (3) disease; and
(2) authorized causes; (3) disease; and (4) (4) resignation. Another form of termination is the
resignation. Another form of termination is the enforcement of the union security clause. To enforce
enforcement of the union security clause. To enforce the clause, 3 things are needed to be proven: (1) the
the clause, 3 things are needed to be proven: (1) the union security clause is applicable; (2) the union is
union security clause is applicable; (2) the union is requesting for the enforcement of the union security
requesting for the enforcement of the union security provision; and (3) there is sufficient evidence to
provision; and (3) there is sufficient evidence to support the union’s decision to expel the employee
support the union’s decision to expel the employee from the union. In this case however, there was no
from the union. In this case, all are present as the sufficient evidence to support the termination of the
language of the CBA which declared that members respondents.
must maintain their membership in good standing as
condition sine qua non or their continued
employment is unequivocal. Disease
Crayons Processing v. Pula, July 30,
Inguillo v. First Philippine Scales, June 5,
2007
2009
Pula was employed by petitioner as a preparation
FSPI and the union entered into a CBA for 5 years.
machine operator. Eventually, he suffered a heart
During the lifetime of the CBA, petitioners joined
attack and was advised to take a leave of absence for
another union, NLM which is an affiliate of NLM-
3 months. Following the angiogram, he was
Katipunan. NLM Katipunan filed an intra-union dispute
considered fit to work but 13 days later, he suffered
against the union and the company. NLM members
dizziness and was advised to take a LOA for 1 month.
were ordered to return Php 90, 000 of union dues
When he returned, he was forced to resign as he was
erroneously collected from the members. The order
not given any reassignment and was given financial
was not followed and later the executive board of the
aid instead. The SC held that the termination was not
union addressed a petition to the company seeking
valid. For a dismissal based on disease to prosper, 2
termination of petitioners for disloyalty. Later, the
requisites must concur: (1) the employee must be
petitioners were terminated so they filed for illegal
suffering from a disease which cannot be cured within
dismissal. The Supreme Court held that the
6 months and his continued employment is prohibited
termination because of the union security clause.
by law or prejudicial to his health or the health of his
Under the Code, an employee may be validly
co-employees; and (2) a certification to that effect
terminated based on: (1) just causes; (2) authorized
must be issued by a competent public health
causes; (3) disease; and (4) resignation. Another form
authority. According to the SC, the certification is
of termination is the enforcement of the union
required as in the case of its absence, the
security clause. Under their CBA, it I clear that the
characterization or even diagnosis of the disease
petitioners must remain a member in good standing
would primarily be shaped according to the interests
for continued employment. To enforce the clause, 3
of the parties rather than the studied analysis of the
things are needed to be proven: (1) the union
appropriate medical professionals.
security clause is applicable; (2) the union is
requesting for the enforcement of the union security

69
Villaruel v. Yeo Han Guan, June 1, 2011 Deoferio v. Intel Technology, June 18,
Villaruel was an employee respondent. During the 2014
course of his employment, he got sick and was Intel employed Deoferio and was assigned in the US.
hospitalized for 2 months. When he reported for work, He was later repatriated to the Philippines after he
he was no longer permitted to go back to work was diagnosed for major depression with psychosis.
because of his sickness. He was given separation pay He underwent a series of medical and psychiatric
amounting to Php 15, 000. Villaruel argued that it is treatment after his confinement. At the PGH, Dr. Lee
not enough and that the separation pay should be concluded that the psychotic symptoms are not
computed from his first day of employment which is curable within a period of 6 months and will
on June 1963. The Supreme Court said that negatively affect his work and social relation with his
respondent did not actually terminate Villaruel: first, co-workers. Thus Intel chose to terminate him and he
the only cause of action in petitioner’s original later filed for illegal dismissal. The SC ruled that the
complaint is that he was offered a very low termination was valid. Dr. Lee’s report substantially
separation pay; second, there was no allegation of proves that Deoferio was suffering from
illegal dismissal; and third, there was no prayer for schizophrenia, which is not curable within 6 months
reinstatement. It was petitioner who initiated the even with proper treatment. Moreover, this was
severance of his employment relations with substantiated by the unusual and bizarre acts that
respondent as he never intended to return to his Deoferio committed while at Intel’s employ.
employment with respondent on the found that his
health is falling. Villaruel in effect resign. Resignation
is defined as a voluntary at of an employee who finds
himself in a situation where he believes that personal
Temporary Suspension of
reasons cannot be sacrificed in favor of the exigency Operations/ Floating Status
of the service and he has no other choice but to
disassociate himself from his employment. As he Manila Mining Corp. Employees
resigned, he is not entitled to separation pay.
Association v. Manila Mining Corp.,
However, the SC granted financial assistance as a
measure of social and compassionate justice and as September 29, 2010
an equitable concession. MMC constructed several tailings dams to treat and
store its waste materials. After the expiration of the
Padillo v. Rural Bank of Nabunturan, tailings permit, the DENR-EMB did not issue a
permanent permit due to the inability of MMC to
January 21, 2013 secure an Environmental Clearance Certificate (ECC).
Padillo was employed by respondents as its SA It was compelled to temporarily shut down its
Bookkeeper. He suffered a mild stroke which operations and lay-off more than 400 employees.
consequently impaired his ability to effectively pursue MMC also called for the suspension of the negotiation
his work. He wrote a letter addressed to respondent of the CBA with the union. According to the SC, the
expressing desire to avail of an early retirement lay-off was valid. MMC’s faultless failure to secure a
package, however this request remain unheeded. Not permit cause the shutdown as it did not succeed in
having claimed his retirement benefits, Padillo filed obtaining the consent of all residents in the
for recovery of unpaid benefits. According to the SC, community. Thus, it is apparent that the suspension
as it is petitioner who voluntary retired, the provisions of the operations was not due to its fault nor was it
on termination on the ground of disease is not necessitated by financial reasons. MMC’s suspension
applicable thus his claim for separation pay must be of the negotiation cannot also be equated to a refusal
denied. The applicable provisions therefore are those to bargain.
stated under the Labor Code concerning retirement.
In the absence of any applicable agreement, an
Nippon Housing v. Leynes, August 3,
employee must: (1) retire when he is at least 60
years of age; and (2) serve at least 5 years in the 2011
company to entitle him to a retirement benefit of at Nippon hired Leynes for the position of property
least ½ month salary for every year of service with a manager. Leynes had a misunderstanding with Engr.
fraction of 6 months being considered as a whole Cantuba regarding the extension of the latter’s
year. These requirements are cumulative in nature. working hours. Nippon allowed Cantuba to report to
Padillo in this case is only 55 years old so he cannot work despite misunderstandings. Because of her
claim for retirement benefits, without prejudice to the disappointment, she submitted a letter asking for an
proceeds under the Philam Life Plan. Nonetheless, emergency leave of absence because she needs to
financial aid was granted by the SC. coordinate with her lawyer regarding her resignation
letter. Later, she sent another letter expressing her
70
intention to return. However, a substitute was already not give him a vested right to the position as would
hired thus she was under floating status. The SC held deprive the company of its prerogative to change the
that the alleged termination was actually valid. The assignment of, or transfer the security guard to, a
hiring of a substitute was brought about by Leynes’ station where his services would be most beneficial to
own rash announcement of her intention to resign. the client. Respondent’s lack of assignment for more
She only had herself to blame for precipitately setting than 6 months cannot be attributed to Exocet. On the
in motion the events which led to a hiring of her contrary, records show that one month after
replacement. Respondent was relieved, Exocet had already offered
a position in the general security service because
SKM Art Corp. v. Bauca, November 27, there were no available clients requiring positions for
VIP security.
2013
SKM informed the 23 respondents that it will suspend
operations for 6 months because of a fire incident.
Spectrum Security Services, Inc. v.
Respondents said that there was a discrimination in Grave, June 7, 2017
choosing the workers to be laid off. According to the Spectrum employed and posted respondents security
SC, the suspension was valid. Under Art 286 of the guards at the premises of Ibiden Co. Later, the
Labor Code. The bona fide suspension of the security guards were issued notices “to return to
operations of a business or undertaking for a period unit” directing them to report to its head office and to
not exceeding 6 months shall not terminate update their documents for re-assignment because of
employment. Employer shall reinstate the employee an action plan spectrum sought to implement.
to his former position without loss of seniority rights if However, the guards did not heed to such order.
he indicates his desire to resume his work not later According to the guards, the action plan amounted to
than 1 month from the resumption of operations. a constructive dismissal and it was the agency’s
However, SKM failed to recall the employees after 6 retaliation for their bringing complaints to recover
months thus considered the employment already unpaid holiday pay and 13th month pay. According to
terminated and therefore respondents were illegally the SC, the guards have actually abandoned their
dismissed. works and had severed their employment relationship
themselves. After the lay-off period of 6 months, the
Exocet Security v. Serrano, September employers should either recall the affected security
guards to work or consider them permanently
29, 2014 retrenched pursuant to the requirements of the law;
Exocet assigned respondent as a close-in security otherwise, the employers would be held to have
personnel for one of JG summits corporate officers. dismissed them, and would be liable for such
After 8 years, respondent was reassigned as close-in dismissals.Despite having been notified of the need
security for Lance Gokongwei. He was later relieved for them to appear before the petitioner's head office
by JG Summit and was not given any reassignment by to update their documents for purposes of reposting,
Exocet for more than 6 months. He thus filed for the respondents refused to receive the notices, and
illegal dismissal. According to the SC, respondent was did not sign the same, without first knowing the
not constructively dismissed. While there is no contents of the memo.
provision in the Code which governs “floating status”
or temporary “off-detail” of security guards, this
situation was considered by the SC as form of
temporary retrenchment or lay-off. The concept has
Illegal Strike
been defined as that period of time when security Jackbilt Industries v. Jackbilt Employees
guards are in between assignments or when they are
made to wait after being relieved from a previous Union, March 20, 2009
post until they are transferred to a new one. It takes Article 264(e) of the Labor Code prohibits any person
place when the security agency’s clients decide not engaged in picketing from obstructing the free
to renew their contracts with the agency, resulting in ingress to and egress from the employer’s premises.
a situation where the available posts under its Since the Union was found to have prevented the free
existing contracts are less than the number of guards entry into and exit of vehicles from Jackbilt’s
in its roster. The placement of the employee on a compound, Union officers and employees clearly
floating status should not last for more than six committed illegal acts in the course through a strike.
months. After six months, the employee should be The use of unlawful means in the course of a strike
recalled for work, or for a new assignment; otherwise, renders such strike illegal. Therefore, strike was ipso
he is deemed terminated. facto illegal. The filing of a petition to declare the
strike illegal was thus unnecessary. Article 264 of the
The security guard’s right to security of tenure does Labor Code further provides that an employer may
71
terminate employees found to have committed illegal
acts in the course of a strike. Petitioner clearly had
the legal right to terminate respondents officers and
PHIMCO Industries v. PHIMCO Industries
employees. Labor Association, August 11, 2010
PHIMCO filed a petition to declare the strike of its
Escario v. NLRC, September 27, 2010 union illegal, alleging that the strikers prevented
As a general rule, backwages are granted to entrance and exit to and from the company
indemnify a dismissed employee for his loss of compound via a virtual human blockade (moving
earnings during the whole period that he is out of his picket) and real physical obstructions (like benches or
job. Considering that an illegally dismissed employee makeshift structures). The Supreme Court ruled that
is not deemed to have left his employment, he is the strike was illegal. Here, the union fully satisfied
entitled to all the rights and privileges that accrue to the procedural requirements [ (a) a notice of strike be
filed with the DOLE 30 days before the intended date
him from the employment. However, back wages are
thereof OR 15 days, in case of ULP; (b) a strike vote be
not granted to employees participating in an illegal approved by a majority of the total union membership in the
strike as it simply accords with the reality that they bargaining unit concerned, obtained via a secret ballot; and
do not render work for the employer during the (c) a notice be given to the DOLE of the results of said
period of the illegal strike. The principle of a fair day's voting at least 7 days before the actual strike ], but the
wage for a fair day’s labor remains as the basic factor strike herein was still illegal because it was
in determining the award thereof. If there is no work accompanied by illegal means employed, such as
performed by the employee there can be no wage or when the strikers herein blocked the free ingress and
pay unless, of course, the laborer was able, willing engross from PHIMCO premises via actual violence
and ready to work but was illegally locked out, and patent intimidation while picketing. The said
suspended or dismissed or otherwise illegally illegal acts were established by pictures submitted by
prevented from working. Thus, under the principle of PHIMCO clearly showed that though the picket was
a fair day's wage for a fair day's labor, the petitioners moving, it was not peaceful because did effectively
were not entitled to the wages during the period of obstruct the entry and exit points of the company
the strike (even if the strike might be legal), because premises on various occasions. Thus, under Art.
they performed no work during the strike. 279(e), the picketing carried on w/ violence, coercion
or intimidation, such as in this case, is unlawful.
Abaria v. NLRC, December 7, 2011
(relate to Bascon v. CA, February 5, VCMC v. Yballe, January 15, 2014
2004) The president of a union (local affiliate of the
The local chapter president wrote to MCCHI Federation which was the EBR of VCMC) (note that
expressing desire to renew CBA. Atty. Alforque, this union is not a registered labor union) wrote to
representative of NFL, found out about this and VCMC, expressing their desire to renew the existing
according to him, this was never referred to NFL, the CBA and their proposals to amend the same. VCMC
mother federation. Thus, the collection of the union refused to bargain with them, explaining that the EBR
fees was later temporarily suspended by the of their rank and file employees was the Federation,
company. Nava, the local chapter president, not them. In the end, this resulted to the Union's
protested this and claims to negotiate with them officers and its members to launch a series of mass
directly instead of NFL. Thus, NFL sought to suspend actions (wearing black and red armbands/headbands;
Nava with several others. The group later launched a marching around the hospital premises; putting up
series of mass actions such as wearing of black and placards, posters, streamers). As a result of this,
red armbands. Later, they were dismissed. According VCMC dismissed them. Included amongst the
to the SC, the hospital cannot be guilty of ULP as dismissed union members are herein respondents,
alleged by Nava’s group as the local chapter was not Yballe, et al.
a legitimate labor organization as it was never
registered and never certified by majority of the rank The Court ruled that the strike was illegal for having
and file hospital employees to represent them in the been conducted by a union which does not have the
CBA. Thus, being not registered, it cannot exercise legal personality to conduct one. Not being a
rights given to a legitimate labor organization. registered labor union, their Notice of Strike was
Therefore, the strike is illegal as it cannot represent deemed not filed by the NCMB for want of legal
the rights of its members having no legal personality. personality. But, the Court held that, despite the
Moreover, they have committed illegal acts by illegal strike, Yballe, et al., as mere union members
blocking the ingress and egress of the hospital and who participated in the same, were still illegally
infliction of violence and intimidation to non- dismissed. In the Decision dated Dec. 7, 2011 (Abaria
participants. case), the Court declared as invalid the dismissal of
72
Hospital employees who participated in the illegal constitutionally guaranteed right. Thus, any
strike conducted by Union which is not a legitimate deprivation of this right must be attended by due
labor organization. Since there was no showing that process of law. This means that any disciplinary
the union members committed any illegal act during action which affects employment must pass due
the strike (their participation limited only to wearing process scrutiny in both its substantive and
armbands), they may not be deemed to have lost procedural aspects. Suspension from work is prima
their employment status by their mere participation facie a deprivation of this right (of security of tenure).
in the illegal strike. On the other hand, the union Thus, termination and suspension from work must be
leaders who conducted the illegal strike despite reasonable to meet the constitutional requirement of
knowledge that Union is not a duly registered labor due process of law. It will be reasonable if it is based
union were declared to have been validly terminated on just or authorized causes enumerated in the Labor
by Hospital. Code. On the other hand, in order to comply with
procedural due process, the twin-notice rule must be
complied with. In this case, PAL denied Montinola
Suspension substantive due process. Just cause has to be
supported by substantial evidence. PAL was unable to
Caong v. Regualos, January 26, 2011 substantiate its claim.
Caong was employed as jeepney drivers under a
boundary agreement (drivers were required to pay a Montinola is entitled to her claim of damages. Illegally
specified amount to the jeepney owner, the excess of suspended employees, similar to illegally dismissed
which will be kept by the driver). There were employees, are entitled to moral damages when their
deficiencies in their boundary payments, so they suspension was attended by bad faith or fraud,
were barred from driving the jeepneys pending their oppressive to labor, or done in a manner contrary to
payment of the arrears. As such, Caong filed a case morals, good customs, or public policy.
for illegal dismissal against the jeepney owner. The
Court held that he was not illegally dismissed. All the
lower courts uniformly declared that Caong was not Retirement
dismissed from employment but merely suspended
pending payment of his arrears. Suspension was not Cercado v. Uniprom, October 13, 2010
to sever the ER-EE relationship and it only dragged on Uniprom implemented a company-wide early
because Caong refused to pay the arrears. retirement program for a number of its employees,
Suspension cannot be categorized as dismissal, one of which was Cercado. Cercado rejected
considering that there was no intent on the part of Uniprom's offered early retirement package.
Regualos to sever the ER-EE relationship between him However, Uniprom, under the power given by their
and Caong. In fact, it was made clear that Caong retirement plan, still decided that Cercado was to
could put an end to the suspension if he only paid his retire, and effected the same. Cercado then filed a
recent arrears. complaint for illegal dismissed against Uniprom.

Also, due process is simply the opportunity to be The Court ruled that Cercado did not validly retire.
heard. Since this is not a case of termination of Retirement is the result of a bilateral act of the
employment (just suspension), the twin-notice rule is parties, a voluntary agreement between the
not necessary. The meeting conducted by Regualos employer and the employee whereby the latter, after
served as sufficient notice to Caong. reaching a certain age, agrees to sever his or her
employment with the former. Article 302 of the Labor
Montinola v. PAL, September 8, 2014 Code, as amended by R.A. No. 7641, pegs the age for
Montinola, a flight attendant employed by PAL, was compulsory retirement at 65 years, while the
suspended for 1 year by PAL on the ground that she minimum age for optional retirement is set at 60
committed violations of PAL's company policy. years. An employer is, however, free to impose a
Allegedly, these violations stem from a report they retirement age earlier than the foregoing mandates
received from the customs officials of Honolulu, which as a valid exercise of MP. It is axiomatic that a
states that she and her fellow crew members pilfered retirement plan giving the employer the option to
a number of goods from the aircraft of PAL. Montinola retire its employees below the ages provided by law
now questions the validity of her suspension and asks must be assented to and accepted by the latter,
to be granted moral and exemplary damages in otherwise, its adhesive imposition will amount to a
connection to the same. The Court ruled that deprivation of property without due process of law.
Montinola was illegally suspended. Security of tenure Acceptance by the employees of an early retirement
of workers is a statutorily protected, and age option must be explicit, voluntary, free, and
uncompelled. While an employer may unilaterally
73
retire an employee earlier than the legally control of the employer, such as when the
permissible ages under the Labor Code, this employer — who is in severe financial strait,
prerogative must be exercised pursuant to a mutually has suffered serious business losses, and has
instituted early retirement plan. In other words, only ceased operations — implements
the implementation and execution of the option may retrenchment, or abolishes the position due
be unilateral, but not the adoption and institution of to the installation of labor-saving devices;
the retirement plan containing such option. For the (2) when the illegally dismissed employee has
option to be valid, the retirement plan containing it contracted a disease and his reinstatement
must be voluntarily assented to by the employees or will endanger the safety of his co-employees;
at least by a majority of them through a bargaining or,
representative. (3) where a strained relationship exists
between the employer and the dismissed
Cercado did not accept or consent to the plan. CA employee
only deemed her to do so. However, implied
knowledge, regardless of duration, cannot equate to None of the above are availing in this case. As for the
the voluntary acceptance required by law in granting claim of retrenchment, although it is a recognized
an early retirement age option to an employer. The management prerogative, it is still subject to faithful
law demands more than a passive acquiescence on compliance with the substantive and procedural
the part of employees, considering that an requirements laid down by law and jurisprudence.
employer’s early retirement age option involves a Here, these requirements were not complied with, or,
concession of the former’s constitutional right to if it was complied with, there was no proof of such
security of tenure. compliance.

Consequences of Dismissal Sagum v. CA, May 26, 2005


Sagum was illegally dismissed, however the CA
Composite Enterprises v. Caparoso, denied her reinstatement. Instead, it ordered for the
employer to just pay her separation pay, citing
August 8, 2007 "strained relations". The Court ruled that the CA erred
Caparoso filed a complaint for illegal dismissal in denying Sagum's reinstatement based on alleged
against Composite. LA ruled in favor of Caparoso, strained relations. The existence of strained relations
finding that they were illegally dismissed. The NLRC is a factual finding and should be initially raised,
reversed this. Composite, now, questions the argued and proven before the Labor Arbiter. Nowhere
decisions/orders of the lower tribunals, specifically on was the issue of strained relations raised in ER’s
whether the order for their reinstatement can be pleadings before the LA and the NLRC. As a rule, no
changed to an order for payment of separation pay strained relations should arise from a valid and legal
instead, on the ground that Caparoso's position is no act of asserting one's right; otherwise, an employee
longer available as the company is undergoing who asserts his right could be easily separated from
retrenchment. the service by merely paying his separation pay on
the pretext that his relationship with his employer
The Court ruled that the order for the reinstatement had already become strained.
of Caparoso should be followed. Reinstatement is the
restoration to a state or condition from which one has
been removed or separated. The intent of the law in
Agabon v. NLRC, November 17, 2004
making a reinstatement order immediately executory Agabon, et al. were dismissed by their employer for
is much like a return-to-work order (i.e ., to restore abandonment of work; and so, they filed a case for
the status quo in the workplace in the meantime that illegal dismissal. The Court ruled that Agabon, et al.
the issues raised and the proofs presented by the were validly dismissed for a just cause (abandonment
contending parties have not yet been finally of work). Abandonment is the deliberate and
resolved). It is a legal provision which is fair to both unjustified refusal of an employee to resume his
labor and management because while execution of employment. It is a form of neglect of duty, hence, a
the order cannot be stayed by the posting of a bond just cause for termination of employment by the
by the employer, the workers also cannot demand employer. For a valid finding of abandonment, these
their physical reinstatement if the employer opts to two factors should be present: (1) the failure to report
reinstate them only in the payroll. Payment of for work or absence without valid or justifiable
separation pay as a substitute for reinstatement is reason; and (2) a clear intention to sever ER-EE
allowed only under exceptional circumstances, viz : relationship, with the second as the more
(1) when reasons exist which are not determinative factor. Agabon, et al. were frequently
attributable to the fault or are beyond the absent since they were subcontracted for another
74
company. Subcontracting for another company
violation against ER violation
clearly showed the intention to sever the ER-EE
relationship. (technically, due to his Initiated by ER's exercise of
acts) EE initiated the his management
dismissal process prerogative
However, the ER in this case did not comply with
procedural due process (twin notice rule); and so, Payment of separation pay Payment of separation pay
they are should pay nominal damages (PhP 30K). The is NOT required is required
omnibus rules contemplate four possible situations
when an employee is dismissed:
Based on the above, SC held that:
(1) the dismissal is for a just cause (Art. 297),
(1) if the dismissal is based on a just cause (Art. 297)
for an authorized cause (Art. 298), or for
but the ER failed to comply with the notice
health reasons (Art. 299), and due process
requirement, the sanction to be imposed upon him
was observed;
should be TEMPERED because the dismissal process
(2) the dismissal is without just or authorized
was, in effect, initiated by an act imputable to the
cause but due process was observed;
employee; and
(3) the dismissal is without just or authorized
(2) if the dismissal is based on an authorized cause
cause and there was no due process; and
(Art. 298) but the employer failed to comply with the
(4) the dismissal is for just or authorized
notice requirement, the sanction should be STIFFER
cause but due process was not observed.
because the dismissal, process was initiated by the
employer's exercise of his management prerogative.
This case is under 4th situation. Under the 4th
situation, the dismissal should be upheld. While the
Also, Court held here that the CA erred when it
procedural infirmity cannot be cured, it should not
ordered Jaka to pay separation pay. In previous
invalidate the dismissal. However, the employer
jurisprudence, the SC held that the general rule is
should be held liable for noncompliance with the
that separation pay is required to be given to EEs in
procedural requirements of due process. As such,
all cases of business closure/cessation of operations
citing Wenphil v. NLRC, the court ruled that where the
by ER. The exception to this is when the closure of
employer had a valid reason to dismiss an employee
business / cessation of operations is due to serious
but did not follow the due process requirement, the
business losses or financial reverses; duly proved, in
dismissal may be upheld but the employer will be
which case, the right of affected EEs to separation
penalized to pay an indemnity (nominal damages) to
pay is lost for obvious reasons. The exception should
the employee [Belated Due Process Rule].
be applied in this case.

Jaka Food Processing v. Pacot, March 28, Industrial Timber v. Ababon, March 30,
2005 2006
EEs were dismissed by Jaka because Jaka was in dire
ITC ceased its business operations; however, they
financial straits. However, the dismissal was effected
failed to comply with procedural due process (no
without due process (no notice). The Court ruled that
notice). The Court ruled that the employees were
the employees were validly dismissed for the reason
validly dismissed for an authorized cause (cessation
of retrenchment (under an authorized cause). The
of business operations). In this case, there was a valid
Statement of Income and Deficit of Jaka to prove its
authorized cause considering the closure or cessation
alleged losses was prepared by an independent
of ITC’s business which was done in good faith and
auditor, SGV & Co. It convincingly showed that the
due to circumstances beyond ITC’s control.
corporation was in dire financial straits, which the EEs
failed to dispute.
Although the closure was done in good faith and for
valid reasons, SC finds that ITC did not comply with
However, since Jaka failed to comply with procedural
the notice requirement. ITC gave final notice to its
due process, they are required to pay nominal
workers of its closure on the same day closure was to
damages.The amount of nominal damages to be paid
take effect (instead of the 1 month notice required by
in this case is PhP50K. The amount here (50K) is
law). While the Court ruled that the sanction should
higher than the Abagon case (30K) because this case
be stiffer in a dismissal based on authorized cause
involves dismissal for an authorized cause, while
(Art. 298) (as the dismissal process was initiated by
Abagon involves dismissal for a just cause
the ER’s management prerogative) where the
JUST CAUSE (297) AUTHORIZED CAUSE employer failed to comply with the notice
(298)
requirement than a dismissal based on just cause
EE is guilty of some EE did not commit any (Art. 282) with the same procedural infirmity;
however, in instances where the execution of a
75
decision becomes impossible, unjust, or too thus hold that the ER is not obliged to give separation
burdensome, modification of the decision becomes benefits to the minority employees pursuant to Art.
necessary in order to harmonize the disposition with 297 as interpreted in the case of Galaxie. As such, SPI
the prevailing circumstances. In the determination of should not be directed to give financial assistance
the amount of nominal damages which is addressed amounting to PhP15K to each of the minority
to the sound discretion of the court, several factors employees based on the Formal Offer of Settlement.
are taken into account: If at all, such formal offer should be deemed only as a
(1) the authorized cause invoked (whether it calculated move on the ER's part to further minimize
was a retrenchment or a closure or cessation the expenses that it will be bound to incur should
of operations or otherwise); litigation drag on, and not as an indication that it was
(2) the number of employees to be awarded; still financially sustainable. However, since the Union
(3) the capacity of the employers to satisfy chose not to accept, said offer did not ripen into an
the awards, taking into account their enforceable obligation on the part of SPI from which
prevailing financial status as borne by the financial assistance could have been realized by the
records; minority employees.
(4) the employer's grant of other termination
benefits in favor of the employees; and Also, the Court ruled that the ER was unable to
(5) whether there was a bona fide attempt to comply with the notice requirement, when it merely
comply with the notice requirements as posted various copies of its notice of closure in
opposed to giving no notice at all. conspicuous places within the business premises. As
earlier explained, the ER was required to serve
Considering the circumstances in this case, SC written notices of termination to its employees, which
reduced the nominal damages from 50K to 10K. it, however, failed to do. So, while the ER had a valid
ground to terminate its employees (closure of
Sangwoo Phil. v. Sangwoo Phils. business), its failure to comply with the proper
procedure for termination renders it liable to pay the
Employees Union, Dec. 9, 2013 employee nominal damages.
ER closed its business due to serious economic losses
and financial reversion, posting notices regarding said
Equitable Banking v. Sadac, June 8,
closure in conspicuous places of their office. ER also
offered to separation benefits in the amount of 1/2 of 2006
the EE's monthly pay for every year of service. Most Issue here is WON salary increases should be part of
of the employees accepted this offer; however, some an EE's backwages. The Court ruled that it should
minority of the EEs refused the offer. The question not. Art. 294 (security of tenure) mandates that an
here is WON the minority EEs are entitled separation EE’s full back wages shall be inclusive of allowances
pay and WON the notice requirement of Art. 298 was and other benefits OR their monetary equivalent. A
complied with. salary increase CANNOT be interpreted as an
allowance or a benefit. Salary increases are not akin
The Court ruled that the minority EEs are not entitled to allowances or benefits, and cannot be confused
to separation benefits. Closure of business, as an with either. The term allowances is sometimes used
authorized cause for termination of employment, synonymously with emoluments, as indirect or
aims to prevent further financial drain upon an contingent remuneration, which may or may not be
employer who cannot pay anymore his employees earned, but which is sometimes in the nature of
since business has already stopped. In such a case, compensation, and sometimes in the nature of
the employer is generally required to give separation reimbursement. Allowances and benefits are granted
benefits to its employees, unless the closure is due to to the EE separate from, and in addition to the wage
serious business losses. The case of Galaxie was or salary. In contrast, salary increases are amounts
cited, wherein the Court held that Art. 297 does not which are added to the EE's salary as an increment
obligate an employer to pay separation benefits when thereto for varied reasons deemed appropriate by the
the closure is due to serious losses. To require an employer. Salary increases are not separate grants by
employer to be generous when it is no longer in a themselves but once granted, they are deemed part
position to do so, in our view, would be unduly of the EEs salary. To extend the coverage of an
oppressive, unjust, and unfair to the employer. Ours allowance or a benefit to include salary increases
is a system of laws, and the law in protecting the would be to strain both the imagination of the Court
rights of the working man, authorizes neither the and the language of law.
oppression nor the self-destruction of the employer.
Perforce, without any cogent reason to deviate from The base figure to be used in the computation of
the findings on the validity of SPI's closure, the Court backwages due to an EE should include the (1) basic

76
salary, and (2) regular allowances that he had been reinstatement. Though the grant of reinstatement
receiving (such as the emergency living allowances commonly carries with it an award of backwages, the
and the 13th month pay mandated under the law) at inappropriateness or non-availability of one does not
the time of his dismissal. carry with it the inappropriateness or non-availability
of the other.
Carlos v. CA, August 28, 2007
The main issue raised here is when the period for The award of backwages is not conditioned on the
computation of backwages and separation pay is EE’s ability or inability to, in the interim, earn any
supposed to end? This question was squarely income. While it may be true that the EE was
addressed in Gaco v. NLRC where it was held that in detained in Criminal Case No. 5137, the Prosecutor
such circumstance, the computation shall be up to found no probable cause for the detention of the EE
the time of finality of this Court's decision. and resolved to dismiss the case. The private
Apparently, the justification is that along with the respondent has not yet been convicted by final
finality of this Court's decision, the issue of illegal judgment in Criminal Case No. 5251. Indeed, he is
dismissal is finally laid to rest. Parenthetically, in this presumed innocent until his guilt is proved beyond
case, the award for separation pay equivalent to one- reasonable doubt.
month pay for every year of service shall be
computed from the time the EEs were illegally Chronicle Securities v. NLRC, November
separated from their employment up to the finality of 25, 2004
this Court's Decision. EE was illegally dismissed. The question here is WON
the computation for backwages and separation pay
Tomas Claudio Memorial College v. CA, was proper (computed from dismissal until finality of
February 16, 2004 judgment for illegal dismissal)? The Court held that
EE was dismissed by the College, since the EE was the computation was improper, considering that the
arrested by the police for violating the Dangerous ER (a broadsheet publisher) closed their publication
Drugs Act. Due to his dismissal, the EE filed a business years before the finality of the judgment.
complaint against the College. In the end, it was held ER's closure of their newspaper business was made
that he was validly dismissed. The issue now is WON on legal and valid grounds. It was never resorted to
a legally dismissed EE is entitled to the payment of as a means to deprive the EE of the opportunity to be
backwages during the period he was detained by the reinstated to his former position. To allow the
police? The College argues that it is absurd for them computation of the backwages due the EE to be
to pay backwages to the EE while the latter was in based on a period beyond their closure would be an
jail, as the EE would thereby be enriching himself at injustice to the ER. Court's power to exact retribution
the expense of the College. The Court held that the from erring employers for cases of illegal dismissal
EE should be granted backwages, even if he was should not go beyond what is recognized as just and
validly dismissed. The payment of backwages is fair under the circumstances.
generally granted on the ground of equity. It is a form
of relief that restores the income that was lost by Intercontinental Broadcasting v.
reason of the unlawful dismissal; the grant thereof is Benedicto, July 20, 2006
intended to restore the earnings that would have EE (marketing manager) was illegally dismissed by
accrued to the dismissed employee during the period the ER. Now, the ER questions the computation for
of dismissal until it is determined that the termination backwages and commission granted to the EE, since
of employment is for a just cause. It is not private they were computed from the time of his dismissal
compensation or damages but is awarded in until the finality of the decision on illegal dismissal
furtherance and effectuation of the public objective of (when EE already reached 68 years old, three years
the Labor Code. Nor is it a redress of a private right beyond the mandatory retirement age). The Court
but rather in the nature of a command to the ruled that the computation for backwages and
employer to make public reparation for dismissing an commission was incorrect. EE was entitled to
employee either due to the former's unlawful act or backwages only up to the time he reached 65 years
bad faith. old, the compulsory retirement age under the law.
When EE was illegally dismissed on Oct. 11, 1994, he
The normal consequences of a finding that an was already 64 years old. He turned 65 years old on
employee has been illegally dismissed are: (1) that Dec. 1, 1994 at which age he was deemed to have
the employee becomes entitled to reinstatement to retired. Since backwages are granted on grounds of
his former position without loss of seniority rights; equity for earnings lost by an employee due to his
and (2) the payment of backwages corresponding to illegal dismissal, EE was entitled to backwages only
the period from his illegal dismissal up to actual for the period he could have worked had he not been
77
illegally dismissed (from time of dismissal until he to disease), as well as for illegal dismissals in which
reached the compulsory retirement age). reinstatement is no longer feasible. On the other
hand, an employee dismissed for any of the just
EE was also entitled to commission only until the time causes enumerated under Article 299 is not, as a
he reached 65 years old. commissions are given to rule, entitled to separation pay. As an exception,
employees only if the ER receives income. EEs, as a allowing the grant of separation pay or some other
reward, receive a percentage of the earnings of the financial assistance to an employee dismissed for just
ER, which they, through their efforts, helped produce. causes is based on equity and social justice. The
Commissions are also given in the form of incentives Court, in the case of San Miguel v. Lao, made a the
or encouragement so that EEs will be inspired to put test on when separation pay may be awarded.
a little more industry into their tasks. Commissions Accordingly, it may be awarded provided that the
can also be considered as direct remunerations for dismissal does not fall under either of two
services rendered. All these different concepts of circumstances:
commissions are incongruent with the claim that an (1) there was serious misconduct, or
EE can continue to receive them indefinitely after (2) the dismissal reflected on the employees
reaching his mandatory retirement age. EE's right to moral character. In this case, the dismissal in
the commissions was coterminous with his the present case was due to loss of trust and
employment; and, in this case, this ended when he confidence, not serious misconduct.
reached the compulsory retirement age. When EE violated the policy of the Bank, there was no
indication that his actions were perpetrated for his
Velasco v. NLRC, June 26, 2006 self-interest or for an unlawful purpose. On the
EE was illegally dismissed. However, in this case, contrary, and as the facts indicate his actions were
NLRC only awarded the EE separation pay, in lieu of motivated by a desire to accommodate a valued
reinstatement, without backwages. The issue now client of the bank.
raised is WON separation pay, in lieu of
reinstatement, is proper and WON NLRC erred in not Bago v. NLRC, April 4, 2007
granting EE backwages. The Court ruled that, in this LA ruled that the EE was illegally dismissed. This led
case, separation pay in lieu of reinstatement is to the payroll reinstatement of the EE. The ruling of
proper. The accepted doctrine is that separation pay the LA was overturned by the NLRC, which found that
may avail in lieu of reinstatement if reinstatement is the EE was validly dismissed for loss of trust and
no longer practical or in the best interest of the confidence. As a result, the payroll reinstatement of
parties. Likewise, separation pay in lieu of the EE was discontinued. The main issue in this case
reinstatement may be awarded if the employee is WON herein unilaterally discontinuation of the EE's
decides not to be reinstated. In this case, EE opted to payroll reinstatement was valid. The Court ruled that
seek separation pay in lieu of reinstatement. the EE's claim that ER "unilaterally withheld her
payroll reinstatement" after the NLRC reversed on the
The Court ruled that the EE is correct in saying that LA's decision has no merit. Article 229, para. 6 of the
he is entitled to backwages, notwithstanding the Labor Code provides that the decision of the NLRC on
award of separation pay. It was grievous error appeals from decisions of the LA "shall become final
amounting to GAD on the part of the NLRC to have and executory after 10 calendar days from receipt
considered an award of separation pay as equivalent thereof by the parties." That the Court of Appeals
to the aggregate relief constituted by reinstatement may take cognizance of and resolve a petition for
plus payment of backwages under Art. 294. The grant certiorari for the nullification of the decisions of the
of separation pay was a proper substitute only for NLRC on jurisdictional and due process considerations
reinstatement; it could not be an adequate substitute does not affect the statutory finality of the NLRC
for both reinstatement and backwages. Decision. Also, in this case, there is no showing that
the CA issued a TRO to enjoin the execution of the
PCIB v. Abad, February 28, 2005 NLRC decision, as affirmed by its Resolution. The CA
EE was validly dismissed for loss of trust and or SC, also, did not overturn the Resolution of the
confidence (just cause) by the Bank. However, lower NLRC, so the NLRC's Resolution here became final
tribunals still granted him separation pay. The Bank is and executory, which was a valid basis for the
now questioning the grant of separation pay despite discontinuance of the payroll reinstatement.
the valid dismissal based on a just cause. The Court
ruled that the grant of separation pay is proper in this Panuncillo v. CAP Phils., February 9,
case, despite valid dismissal. The award of separation 2007
pay is required for dismissals due to causes specified Issue was whether petitioner is entitled to back
under Articles 298 (authorized cause) and 299 (due wages from the date her compensation was withheld
78
pursuant to NLRC decision (reinstating her), until it (2) to reinstate them in the payroll, and that failing to
was reversed. The Court held no. An order of exercise either of the 2 options will require the
reinstatement is immediately executory if it was from employer to pay the employee’s salaries.
the LA. NLRC decisions however are different as it
needs to issue a writ of execution based on Art. 224. However, after the LA’s decision is reversed by a
If the requirements of Art. 224 were to govern, then higher tribunal, the EE may be barred from collecting
the executors nature of a reinstatement order or the accrued wages, if it is shown that the delay in
award contemplated by Art. 223 will be unduly enforcing the reinstatement pending appeal was
circumscribed and rendered ineffectual. Thus, without the fault on the part of the ER. The test here
Panuncillo here is not entitled to collect any is twofold: (1) there must be actual delay or the fact
backwages from the time NLRC decision became final that the order of reinstatement pending appeal was
and executors up to the time it was reversed. In sum, not executed period to its reversal; and (2) the delay
while under the 6th paragraph of Art. 223 of the LC, must not be due to the ER’s unjustified act or
the decision of the NLRC becomes final and executors omission. If the delay is due to the employer’s
after the lapse of 10 calendar days from receipt unjustified refusal, the ER may still be required to pay
thereof by parties, the adverse party is not precluded the salaries notwithstanding the reversal of the LA’s
from assailing it via Petition for Certiorari under Rule decision. HOWEVER, in this case, PAL was not made
65 before the CA, and then to the SC via a petition for to pay since it was still under receivership, thus its
review under Rule 45. If during the pendency of the obligation to pay did not attach. In sum, the
review, no order is issued by the courts enjoining the obligation to pay employees salaries upon the failure
execution of a decision of the LA or NLRC which is of the employer to exercise the alternative options
favorable to the employee, the LA or NLRC must under Art. 223 is not a hard and fast rule, considering
exercise extreme prudence and observe judicial the inherent constraints of corporate rehabilitation.
courtesy when circumstances so warrant.
PAL v. Paz, November 26, 2014
The order of reinstatement is incompatible with the EE conducted a strike to which DOLE issued a return
finding that the dismissal is for a valid cause, or to work order. The return of the EE was delayed thus
finding of guilt. Where the totality of the evidence they were barred entry by the company in their
was sufficient to warrant the dismissal of the workplace. They sued for illegal dismissal with the LA
employee, the law warrants their dismissal without which ruled in their favor. The LA ordered
making any distinction between a first offender and a reinstatement and payment of full backwages from
habitual delinquent. Art. 223 of the LC is clear that date of strike to actual reinstatement. NLRC however
the employee, who is ordered reinstated must be reversed. Notwithstanding this, EE’s moved for the
accepted back to work under the same terms and issuance of a writ of execution with the LA for the
conditions prevailing prior to their dismissal or reinstatement salaries. Issue is whether the EE’s are
separation. The decision of the LA reinstating a entitled to this reinstatement salaries. The Court held
dismissed or separated employee, insofar as the no, due to the corporate rehabilitation that PAL is
reinstatement aspect is concerned, shall immediately undergoing.
be executors, even pending appeal. An order to
reinstate an employee, even to a position one rank The rule is that the EE is entitled to reinstatement
lower which was formerly held by petitioner was thus salaries notwithstanding the reversal of the LA
erroneous. decision granting him said relief. However, as
enunciated in the Garcia v. PAL case, after the LA’s
Garcia v. Philippine Airlines, January 20, decision is reversed by a higher tribunal, the EE may
2009 be barred from collecting accrued wages if it is shown
EE were dismissed for violating company’s code of that the delay in enforcing the reinstatement pending
discipline. Issue was whether they may collect their appeal was without fault on the part of the ER. [See
wages during the period between the LA’s also twofold test in previous case]. A scrutiny of this
reinstatement pending appeal and the NLRC’s case however shows that the delay in reinstating the
decision overturning it--NO. SC affirms that even if EE was not due to PAL’s unjustified refusal to abide by
the order of reinstatement of the LA is reversed on the reinstatement order and payment of salaries, but
appeal, it is obligatory on the part of the employer to because of the constraints of corporate rehabilitation.
reinstate and pay the wages of the dismissed Thus, all claims of payment against PAL are
employee during the period of appeal until reversal suspended.
by the higher court. This settles that the LA’s order of
reinstatement is immediately executory and the
employer has to either (1) re-admit them to work or

79
Islriz v.Capada, January 31, 2011 bar. The dismissal in this case was found to be valid
EE were illegally dismissed, LA ordered and as such the EE’s are not entitled to full
reinstatement. NLRC set aside LA decision upon backwages.
appeal but also ordered reinstatement but without
backwages--this decision became final and executory. Palteng v. UCPB, February 27, 2009
Issue is whether EE may collect their wages during EE was found to have committed several offenses
the period between the LA order of reinstatement under the company’s discipline code. She was made
pending appeal and the NLRC resolution overturning to explain, and claimed that she made an honest
the LA. Court ruled in the affirmative. Even if the mistake. She was later dismissed with forfeiture of all
order of reinstatement of the LA is reversed on benefits. She then filed a complaint for illegal
appeal, it is obligatory on the part of the ER to dismissal, seeking reinstatement with full backwages,
reinstate and pay the wages of the dismissed EE or in the alternative, payment of separation pay with
during the period of appeal until reversal by the full backwages. The Court ruled that she was not
higher court or tribunal. It is also settled that LA’s entitled to backwages but only to separation pay.
order of reinstatement is immediately executory and Court stated that an employee who is illegally
the employer has to either re-admit them to work dismissed from work is entitled to reinstatement
under same terms and conditions prevailing prior to without loss of seniority rights as well as full
their dismissal, or to reinstate them in the payroll. backwages, inclusive of allowances and to other
Failure to exercise either option in the alternative will benefits or their monetary equivalent computed from
esquire the ER to pay the EE’s salaries. the time compensation was withheld up to actual
reinstatement. HOWEVER, in the event that
After reversal of LA’s decision, EE is barred from reinstatement is no longer possible, EE may be given
collecting accrued wages, if it is shown that the delay separation pay instead. Reinstatement and payment
in enforcing the reinstatement pending appeal was of backwages are distinct and separate reliefs. The
without fault on the part of the ER. [See twofold test award of one does not bar the other. Backwages may
in previous case]. If the delay is due to the be awarded without reinstatement, and
employer’s unjustified refusal, the employer may still reinstatement may be ordered without awarding
be required to pay the salaries notwithstanding the backwages.
reversal of the LA’s decision. Applying the two-fold
test in the case at bar, ER failed to reinstate or effect Alcantara & Sons v. CA, September 29,
payroll reinstatement of EE until the decision was
overturned--as such there was actual delay of
2010
reinstatement. There was also a finding that the The petitioner company and the Union entered into a
delay is due to the unjustified act or omission of the CBA that bound them to hold no strike and no lockout
ER. The Garcia v. PAL doctrine cannot apply in the in the course of its life. There was a deadlock
case at bar, as Islriz here is not undergoing pertaining to the economic provisions prompting the
rehabilitation or any analogous situation which would union to file a notice of strike. Some union members
justify the non-exercise of the two options filed a claim for illegal dismissal when the company
(reinstatement or payroll reinstatement). tried to enjoin them from striking.. Issues are whether
the terminated members are entitled to backwages
on account of company’s refusal to reinstate them
Lansangan v. Amkor Technology (pending appeal by the parties from LA decision); and
Philippines, January 30, 2009 whether the terminated members are entitled to
EE’s were terminated for extremely serious offenses accrued backwages and separation pay. The Court
as defined in their code of discipline, prompting the held yes to both. In the first issue, Court held that
former to file illegal dismissal complaints. LA although the LA failed to act on the terminated
dismissed but ordered reinstatement. NLRC deleted members’ motion for reinstatement pending appeal,
reinstatement. CA affirmed finding of guilt of EE, but the company had the duty under Art. 223 to
ordered to pay EE back wages without qualification immediately reinstate the affected EE’s. As to the
and deduction from the date of LA decision up to second, the Court awarded the accrued backwages
NLRC decision. The Court affirms the CA decision in and separation pay in furtherance of the laws policy
limiting the payment of backwages up until the NLRC of compassionate justice and equity--despite the
decision only. Art. 223 and the Roquero case finds no members being dismissed validly. (In sum, Court is
application in the case at bar, as they both pertain to saying that since their grievance is understandable +
a situation where the dismissed or separated they don’t have any past infractions, the award is
employee is given an interim relief while the case for proper)
illegal dismissal is pending. It does not apply when
there is no finding of illegal dismissal as in the case at
80
Aboc v. Metrobank, December 13, 2010 reinstatement, if viable, or separation pay, if
EE participated in the lending activity of his boss. reinstatement is no longer viable; and (2) backwages.
Bank dismissed EE, prompting the latter to file a These two reliefs are separate and distinct from each
complaint for illegal dismissal. LA ruled in EE’s favor-- other and are awarded conjunctively. However, due to
Bank reinstated him as per LA’s order. NLRC reversed the strained relations of the parties, the payment of
LA decision, holding that EE was guilt of serious separation pay was considered proper. Also, being a
misconduct and breach of trust and loss of probationary EE, she is entitled to separation pay and
confidence. NLRC also ordered bank to pay EE back wages. The backwages should be reckoned from
reinstatement wages. Issue is whether CA erred in the time of her constructive dismissal until the date
affirming NLRC? Court held that CA, NLRC is correct. of the termination of employment.
There was just causes for EE’s dismissal (serious
misconduct and breach of trust and loss of The computation should not cover the entire period
confidence). As to the monetary award for from the time her compensation was withheld up to
reinstatement, the Court held that it was warranted the time of her actual reinstatement. This is because
as provided under Art. 223 [LA decision being EE was a probationary EE, and the lapse of her
immediately executory as to reinstatement/payroll probationary employment without her appointment
reinstatement] in the case at bar, bank had already as a regular effectively severed the EE-ER
reinstated EE in its payroll. Since bank chose payroll relationship between the parties.
reinstatement for EE, the Court held that EE then
became a reinstated regular EE. Being a regular EE, Pfizer v. Velasco, March 9, 2011
he was restored to his previous position without loss EE was handed a suspension due to a series of
of seniority rights and other privileges. violations of company rules. She filed a complaint for
illegal suspension. EE was later terminated from her
Prince Transport v. Garcia, January 12, employment to which she filed a complaint for illegal
dismissal. LA and NLRC both ordered reinstatement
2011 with backwages. CA reversed but ordered company to
ER wanted to block the formation of a Union by the pay wages from the fate of LA decision up to CA
EE’s so he transferred union members to a sub- decision. Issue is whether company is required to pay
company. Said sub-company deteriorated due to the wages from the said time period. Court held yes,
fault of the ER. Thus EEs filed a complaint for illegal citing Art. 223 stating that “an EE entitled to
dismissal and unfair labor practice. Issue is whether reinstatement shall either be admitted back or payroll
the order to reinstate EE was valid considering that reinstatement. It also entitles the EE to receive his
the issue of reinstatement was never brought before accrued backwages from the moment of order of
the CA, and EE never questioned the award of reinstatement up to the date when the same was
separation pay. Court held in the affirmative stating reversed by a higher court.
that the complaints were clear that EEs were seeking
reinstatement. Rule 7, Sec. 2 of ROC provides that a “Reinstatement” is understood to mean restoration to
pleading shall specify the relief sought but may add a state or condition from which one has been
other reliefs as may be deemed just and equitable. removed or separated. Company in the case at bar
Thus, a court can grant the relief warranted by the did not reinstate EE nor reinstated her in its payroll,
allegation and the proof even if it is not specifically as ordered by LA. Foreseeably, an ER may circumvent
sought. the immediately enforceable reinstatement order of
the LA by crafting return-to-work directives that are
Robinsons Galleria/Robinsons ambiguous or meant to be rejected by the EE, and
Supermarket Corp. v. Ranchez, January then disclaim liability for backwages due to non-
reinstatement by capitalizing on the EE’s refusal to
19, 2011 work. In sum, the option of the ER to effect actual or
EE was a probationary EE who was accused of payroll reinstatement must be exercised in GF. In
stealing from the supermarket. She then filed a case sum, the Court also reiterated that reinstatement
for illegal dismissal after being terminated. Issue is pending appeal necessitates that it must be
whether she was illegally terminated to which Court immediately self-executory without need of a writ of
ruled in the affirmative. As a probationary EE, she execution.
enjoys security of tenure and may only be terminated
for a just and authorized cause, while also according
the EE due process. These were not accorded to her,
Luna v. Allado Construction, May 30,
thus she was illegally dismissed. Under Art. 279 of 2011
the LC, an EE who was unjustly or illegally, or EE filed a complaint for illegal dismissal when he was
constructively dismissed is entitled to: (1) either not given any work after refusing to accept
81
reassignment. LA dismissed complaint, deemed that Nacar V. Gallery Frames, August 13,
EE resigned but ER still needs to pay money by way
of financial assistance NLRC reversed, ruling that EE
2013
was illegally dismissed and should be paid separation EE was awarded backwages and separation pay
pay. CA sustained LA ruling. Issue is whether EE was which became final and executory with the SC. EE
illegally dismissed to which the Court held no--BUT, later asked for recomputation as he alleged that his
the financial assistance is only proper. As a general backwages should be computed from the time of his
rule, financial assistance is allowed only in instances illegal dismissal until the finality of the SC decision
where the EE is validly dismissed for causes other (May 2002) with interest. The LA denied the motion
than serious misconduct or those reflecting on his saying that the reckoning point should be from the
moral character. Also, the Court has ruled before that time EE was illegally dismissed (January 1997) until
where there is no dismissal to speak of, an award of the decision of the LA (October 1998). LA reasoned
financial assistance is not in order. But, as an that said date should be the reckoning point because
exception, financial assistance may be extended as a EE did not appeal hence as to EE, the decision
measure of social justice and exceptional became final and executory. Court held that this as
circumstances, and as an equitable concession. In the incorrect--the computation should be from the LA
case at bar, there appears to be no infraction decision up to the SC decision (May 2002). There are
committed by EE in all his years of service, thus the two parts to a decision when it comes to illegal
financial assistance is only proper. dismissal: (1) ruling that EE was illegally dismissed--
which is immediately final even if ER appeals--but will
be reversed if ER wins on appeal; and (2) ruling on
Villaruel v. Yeo Han Guan, June 1, 2011 the award of backwages and/or separation pay. For
EE was not allowed to return to work due to his backwages, it will be computed from the date of
illness. According to ER, EE was asked to report for illegal dismissal until the date of the decision of the
work but he did not show up. LA ruled in favor of EE LA. But if ER appeals, then the date shall be extended
and ordered payment of separation benefits but in until the date when the appellate court’s decision
the CA, the latter deleted the award of separation shall become final.
pay. Issue is whether EE was dismissed and whether
he is entitled to separation pay. Court ruled that EE
was not dismissed but voluntarily resigned. EE was
Integrated Microelectronics v. Pionilla,
the one who initiated the severance of his August 28, 2013
employment relations when he said in his pleadings EE was found to have violated company rules by
that he never intended to return to work due to his letting another person use his ID. He admitted to his
failing health. “Resignation” is defined as the violation and was dismissed. He filed a complaint for
voluntary act of an EE who finds himself in a situation illegal dismissed with the LA who ruled in his favor.
where he believes that personal reasons cannot be NLRC reversed but CA affirmed LA decision. ISSUE
sacrificed in favor of the exigency of the service, and before the SC was whether or not the award of
he has no other choice but to disassociate himself reinstatement and full backwages to EE is excessive
from his employment. and unfair. Court ruled in the affirmative. As a general
rule, an illegally dismissed EE is entitled to
As to the issue of separation pay, the rule is that an reinstatement (or separation pay if reinstatement is
EE who voluntarily resigns from employment is not not viable) and payment of full backwages. In certain
entitled to separation pay, except when it is cases, however, the Court has carved out an
stipulated in the employment contract or CBA, or it is exception to the foregoing rule and thereby ordered
sanctioned by established ER practice or policy. By the reinstatement of the employee without
way of exception, the Court has allowed grants of backwages on account of the following: (a) the fact
separation pay to stand as a measure of social that dismissal of the EE would be too harsh of a
justice. There is no provision in the LC which grants penalty and (b) that the ER was in good faith in
separation pay to voluntarily resigning EEs. Since EE terminating the EE. IN THE CASE AT BAR, Court
was not terminated from his work, and is deemed to observed that dismissal was too harsh of a penalty to
have resigned, he is not entitled to separation pay be imposed against EE and that ER was in good faith
under the LC. However, the Court granted him when it dismissed EE. Since these circumstances
financial assistance for having served ER over 30 trigger the application of the exception to the rule on
years, and having no infractions. back wages, the Court ruled that backwages must be
deleted, notwithstanding the illegality of his
dismissal.

82
United Tourist Promotion v. Kemplin, separation pay in lieu of reinstatement is allowed.
When that happens (when separation pay is allowed
February 5, 2014 in lieu of reinstatement), the finality of the illegal
Kemplin served as President for petitioner company. dismissal decision becomes the reckoning point
He was terminated by the company for the expiration instead of the reinstatement that the law decrees. In
of his contract and for his inhuman treatment of his allowing separation pay, the final decision effectively
subordinates. Kemplin filed a case for illegal dismissal declares that the employment relationship ended so
which the LA ruled in his favor. ISSUE before the that separation pay and back wages are to be
Court is whether or not KEMplin was illegally computed up to that point.
dismissed and whether he should be reinstated.
COURT HELD that Kemplin was illegally dismissed The recomputation is not violative of the principle of
because due process was not accorded to him plus immutability of a final and executory judgment
there was no violation that would warrant his because it is a natural consequence of an illegal
dismissal. As to the reinstatement issue, the COURT dismissal decision. Recomputation of the
RULED that it is not proper, applying the doctrine of consequences of illegal dismissal upon execution of
strained relations. Under the doctrine of strained the decision does not constitute as an alteration or
relations, payment of separation pay is considered an amendment of the final decision being implemented.
acceptable alternative to reinstatement when the The illegal dismissal ruling stands; only the
latter option is no longer desirable or viable. Doctrine computation of monetary consequences of the
of strained relations also applies with EE decides not dismissal is affected and this is not a violation of the
to be reinstated and demands separation pay. principle of immutability of final judgments.

International School v. ISAE, February 5,


Wenphil Corporation v. Abing, April 7,
2014
EE was terminated for allegedly failing to improve on
2014
her performance as an instructor. She alleged that LA ruled that EEs were illegally dismissed by
she was being asked to teach a class that she was petitioner--ordered reinstatement with payment of
not trained to teach. EE was later dismissed. She then back wages from date of termination until date of
filed a complaint for illegal dismissal. LA, NLRC, and actual reinstatement. Parties agreed to payroll
CA all declared that EE was illegally dismissed. ISSUE reinstatement while on appeal. NLRC ordered
is whether EE was really illegally dismissed and payment of separation pay instead of reinstatement.
whether she is entitled to separation pay. COURT CA reversed NLRC which was affirmed by SC whose
HELD that she was not illegally dismissed and found decision became final and executory. EEs filed for
her to be negligent with her duties and guilty of gross recomputation, alleging that there was a ruling of
inefficiency(i.e. recurring problems in her lesson plan illegal dismissal by the LA thus under the law, they
that was not resolved). The COURT HELD as to were entitled to backwages from LA decision until the
separation pay that it was proper to grant it to EE in NLRC reversed. ISSUE before the Court is whether EEs
view of the length of her service with the school and are entitled to reinstatement and backwages. COURT
having no record of infraction. HELD in the affirmative--because orders of
reinstatement are immediately executory, the ER has
the obligation to reinstate and pay the wages of the
University of Pangasinan v. Fernandez, dismissed EE during the period of appeal until
November 12, 2014 reversal of higher court. In case of payroll
EEs filed illegal dismissal case against petitioner reinstatement, EE is not obligated to return or
school which they won through an SC decision reimburse the salary he received because it will run
(2005). This SC ruling became final and executory counter to the immediate excretory nature of the
thus they sought for its execution. EEs moved for order of reinstatement. Court also stated that
recomputation from the date of the LA decision up to reinstatement and backwages can be awarded
the 2005 SC decision. Petitioner school questioned simultaneously as reliefs available to an illegally
amount arrived at by the recomputation. ISSUE is if dismissed employee.
the computation of backwages and benefits should
be reckoned from the LA decision up to the 2005 SC As to the reckoning period, the period for computing
decision. COURT RULED in the affirmative. That the the backwages due to the EEs during period of appeal
amount of award increased is a consequence of ERs should end on the date that a higher court reversed
continued recourse against the illegal dismissal the labor arbitration ruling of illegal dismissal.
decision. Art. 279 provides for the consequences of
illegal dismissal in no uncertain terms, qualified only
by jurisprudence in its interpretation of when
83
PNCC Skyway Corporation v. Secretary should be the legal basis for the computation of the
backwages? The Court ruled that backwages and
of Labor and Employment, April 19, separation pay should be computed from the time of
2016 dismissal until the finality of the decision ordering the
PSC was a subsidiary of PNCC to operate the Skyway. separation pay. The rule is, if the LA's decision, which
PNCC then entered into a Toll Operation Agreement, granted separation pay in lieu of reinstatement, is
whereby the operation and management of the appealed by any party, the employer-employee
Skyway would be transferred from PSC to SOMCO. A relationship subsists and until such time when
transition period of 5 1/2 months was provided decision becomes final and executory, the employee
commencing on the date of signing of the ASTOA is entitled to all the monetary awards awarded by the
until December 31, 2007, during which period, PSC LA.
continued to operate the Skyway. Accordingly, PSC
issued termination letters to its employees and filed a United Coconut Chemicals, Inc. v.
notice of closure, stating that it shall cease to operate
and maintain the Skyway, and that the services of the
Almores, July 12, 2017
employees would be consequently terminated UCCI hired the respondent as its Senior Utilities
effective January 31, 2008. PSC stated in the notices Inspector, and eventually became a member of the
of termination to the employees (as well as in the Union until his expulsion. He then filed a complaint
notice to the DOLE) that the dismissal of the for illegal dismissal in the NLRC. ISSUE: how should
employees would take effect on January 31, 2008, backwages be computed? The determination of the
when it admitted that it actually ceased to operate salary base for the computation of backwages
and maintain the Skyway upon its turnover to SOMCO requires simply an application of judicial precedents
on December 31, 2007. As such, PSC fixed the defining the term "backwages." An unqualified award
termination date at January 31, 2008 only to make it of backwages means that the employee is paid at the
appear that it was complying with the one-month wage rate at the time of his dismissal. Furthermore,
notice requirement. The Court Ruled that the mere the award of salary differentials is not allowed, the
fact that PSC turned over the operation and established rule being that upon reinstatement,
management of the Skyway to SOMCO and ceased illegally dismissed employees are to be paid their
business operations on December 31, 2007, should backwages without deduction and qualification as to
not be taken to mean that the FSC employees were any wage increases or other benefits that may have
ipso facto terminated on the same date. The been received by their co-workers who were not
employees were notified that despite the cessation of dismissed or did not go on strike. The base figure to
its operations on December 31, 2007 – which, as a be used in reckoning full backwages is the salary rate
consequence thereof, would result in the of the employee at the time of his dismissal. The
needlessness of their services – the effective date of amount does not include the increases or benefits
their termination from employment would be on granted during the period of his dismissal because
January 31, 2008. As a rule, notice of the eventual time stood still for him at the precise moment of his
closure of establishment is a “personal right of the termination, and move forward only upon his
employee to be personally informed of his [or her] reinstatement. Hence, the respondent should only
proposed dismissal as well as the reasons therefor.” receive backwages that included the amounts being
The reason for this requirement is to “give the received by him at the time of his illegal dismissal but
employee some time in prepare for the eventual loss not the benefits granted to his co-employees after his
of his [or her] job.” dismissal.

C.I.C.M. Mission Seminaries v. Perez, Appeal Bond


January 18, 2017
This is an offshoot from an illegal dismissal case. The McBurnie v. Ganzon, EGI-Managers, Inc.,
LA recognized Perez’s right to receive payment of October 17, 2013
backwages and separation pay in lieu of
reinstatement. The decision became final and Sara Lee v. Macatlang, June 4, 2014
executory. Then Perez filed for a Writ of Execution. In Balite v. SS Ventures, February 4, 2015
10 July 2014, the LA ruled that the cash bond was
insufficient. CICM appealed to the NLRC. In the
meantime, LA issued a writ of execution that would
garnish the amount from CICM’s bank account. CICM Dispute Settlement
moved for the quashal of such writ, and it was
granted, but the amount still stands. ISSUE: What Article 128, Labor Code. Visitorial and

84
enforcement power. - (a) The Secretary of Labor Article, and no inferior court or entity shall issue
and Employment or his duly authorized temporary or permanent injunction or restraining
representatives, including labor regulation officers, order or otherwise assume jurisdiction over any
shall have access to employer's records and case involving the enforcement orders issued in
premises at any time of the day or night whenever accordance with this Article.
work is being undertaken therein, and the right to
copy therefrom, to question any employee and (e) Any government employee found guilty of
investigate any fact, condition or matter which may violation of, or abuse of authority, under this Article
be necessary to determine violations or which may shall, after appropriate administrative
aid in the enforcement of this Code and of any investigation, be subject to summary dismissal
labor law, wage order or rules and regulations from the service.
issued pursuant thereto.
(f) The Secretary of Labor and Employment may, by
(b) Notwithstanding the provisions of Articles 129 appropriate regulations, require employers to keep
and 217 of this Code to the contrary, and in cases and maintain such employment records as may be
where the relationship of employer-employee still necessary in aid of his visitorial and enforcement
exists, the Secretary of Labor and Employment or powers under this Code.
his duly authorized representatives shall have the
power to issue compliance orders to give effect to
the labor standards provisions of this Code and Article 129, Labor Code. Recovery of wages,
other labor legislation based on the findings of simple money claims and other benefits. - Upon
labor employment and enforcement officers or complaint of any interested party, the Regional
industrial safety engineers made in the course of Director of the Department of Labor and
inspection. The Secretary or his duly authorized Employment or any of the duly authorized hearing
representatives shall issue writs of execution to the officers of the Department is empowered, through
appropriate authority for the enforcement of their summary proceeding and after due notice, to hear
orders, except in cases where the employer and decide any matter involving the recovery of
contests the findings of the labor employment and wages and other monetary claims and benefits,
enforcement officer and raises issues supported by including legal interest, owing to an employee or
documentary proofs which were not considered in person employed in domestic or household service
the course of inspection. (As amended by Republic or househelper under this Code, arising from
Act No. 7730, June 2, 1994). employer-employee relations: Provided, That such
complaint does not include a claim for
An order issued by the duly authorized reinstatement: Provided further, That the
representative of the Secretary of Labor and aggregate money claims of each employee or
Employment under this Article may be appealed to househelper does not exceed Five thousand pesos
the latter. In case said order involves a monetary (P5,000.00). The Regional Director or hearing
award, an appeal by the employer may be officer shall decide or resolve the complaint within
perfected only upon the posting of a cash or surety thirty (30) calendar days from the date of the filing
bond issued by a reputable bonding company duly of the same. Any sum thus recovered on behalf of
accredited by the Secretary of Labor and any employee or househelper pursuant to this
Employment in the amount equivalent to the Article shall be held in a special deposit account by,
monetary award in the order appealed from. (As and shall be paid on order of, the Secretary of
amended by Republic Act No. 7730, June 2, 1994). Labor and Employment or the Regional Director
directly to the employee or househelper concerned.
(c) The Secretary of Labor and Employment may Any such sum not paid to the employee or
likewise order stoppage of work or suspension of househelper because he cannot be located after
operations of any unit or department of an diligent and reasonable effort to locate him within a
establishment when non-compliance with the law period of three (3) years, shall be held as a special
or implementing rules and regulations poses grave fund of the Department of Labor and Employment
and imminent danger to the health and safety of to be used exclusively for the amelioration and
workers in the workplace. Within twenty-four hours, benefit of workers.
a hearing shall be conducted to determine whether
an order for the stoppage of work or suspension of Any decision or resolution of the Regional Director
operations shall be lifted or not. In case the or hearing officer pursuant to this provision may be
violation is attributable to the fault of the employer, appealed on the same grounds provided in Article
he shall pay the employees concerned their 223 of this Code, within five (5) calendar days from
salaries or wages during the period of such receipt of a copy of said decision or resolution, to
stoppage of work or suspension of operation. the National Labor Relations Commission which
shall resolve the appeal within ten (10) calendar
(d) It shall be unlawful for any person or entity to days from the submission of the last pleading
obstruct, impede, delay or otherwise render required or allowed under its rules.
ineffective the orders of the Secretary of Labor and
Employment or his duly authorized representatives The Secretary of Labor and Employment or his duly
issued pursuant to the authority granted under this authorized representative may supervise the

85
payment of unpaid wages and other monetary member for the writing of the opinion. It shall be
claims and benefits, including legal interest, found mandatory for the division to meet for purposes of
owing to any employee or househelper under this the consultation ordained therein. A certification to
Code. this effect signed by the Presiding Commissioner of
the division shall be issued, and a copy thereof
attached to the record of the case and served upon
Article 220, Labor Code. National Labor the parties.
Relations Commission. - There shall be a National
Labor Relations Commission which shall be The Chairman shall be the Presiding Commissioner
attached to the Department of Labor and of the first division, and the seven (7) other
Employment solely for program and policy members from the public sector shall be the
coordination only, composed of a Chairman and Presiding Commissioners of the second, third,
twenty-three (23) Members. fourth, fifth, sixth, seventh and eight divisions,
respectively. In case of the effective absence or
Eight (8) members each shall be chosen only from incapacity of the Chairman, the Presiding
among the nominees of the workers and employers Commissioner of the second division shall be the
organizations, respectively. The Chairman and the Acting Chairman.
seven (7) remaining members shall come from the
public sector, with the latter to be chosen The Chairman, aided by the Executive Clerk of the
preferably from among the incumbent Labor Commission, shall have administrative supervision
Arbiters. over the Commission and its regional branches and
all its personnel, including the Labor Arbiters.
Upon assumption into office, the members
nominated by the workers and employers The Commission, when sitting en banc, shall be
organizations shall divest themselves of any assisted by the same Executive Clerk, and, when
affiliation with or interest in the federation or acting thru its Divisions, by said Executive Clerk for
association to which they belong. its first division and seven (7) other Deputy
Executive Clerks for the second, third, fourth fifth,
The Commission may sit en banc or in eight (8) sixth, seventh and eighth divisions, respectively, in
divisions, each composed of three (3) members. the performance of such similar or equivalent
The Commission shall sit en banc only for purposes functions and duties as are discharged by the Clerk
of promulgating rules and regulations governing of Court and Deputy Clerks of Court of the Court of
the hearing and disposition of cases before any of Appeals.
its divisions and regional branches and formulating
policies affecting its administration and operations. The Commission and its eight (8) divisions shall be
The Commission shall exercise its adjudicatory and assisted by the Commission Attorneys in its
all other powers, functions, and duties through its appellate and adjudicatory functions whose term
divisions. Of the eight (8) divisions, the first, shall be coterminous with the Commissioners with
second, third, fourth, fifth and sixth divisions shall whom they are assigned. The Commission
handle cases coming from the National Capital Attorneys shall be members of the Philippine Bar
Region and other parts of Luzon; and the seventh with at least one (1) year experience or exposure in
and eighth divisions, cases from the Visayas and the field of labor-management relations. They shall
Mindanao, respectively: Provided, That the receive annual salaries and shall be entitled to the
Commission sitting en banc may, on temporary or same allowances and benefits as those falling
emergency basis, allow cases within the jurisdiction under Salary Grade twenty-six (SG 26). There shall
of any division to be heard and decided by any be as many Commission Attorneys as may be
other division whose docket allows the additional necessary for the effective and efficient operations
workload and such transfer will not expose litigants of the Commission but in no case more than three
to unnecessary additional expenses. The divisions (3) assigned to the Office of the Chairman and each
of the Commission shall have exclusive appellate Commissioner.
jurisdiction over cases within their respective
territorial jurisdiction. No Labor Arbiter shall be assigned to perform the
functions of the Commission Attorney nor detailed
The concurrence of two (2) Commissioners of a to the office of any Commissioner. (As amended by
division shall be necessary for the pronouncement Section 1, Republic Act No. 9347 [July 27, 2006]
of a judgment or resolution. Whenever the required and as previously amended by Republic Act No.
membership in a division is not complete and the 7700 and Section 5, Republic Act No. 6715).
concurrence of two (2) Commissioners to arrive at
a judgment or resolution cannot be obtained, the
Chairman shall designate such number of Article 221, Labor Code. Headquarters, Branches
additional Commissioners from the other divisions and Provincial Extension Units. - The Commission
as may be necessary. and its first, second, third, fourth, fifth and sixth
The conclusions of a division on any case divisions shall have their main offices in
submitted to it for decision shall be reached in Metropolitan Manila, and the seventh and eighth
consultation before the case is assigned to a divisions in the cities of Cebu and Cagayan de Oro,

86
respectively. The Commission shall establish as Article 223, Labor Code. Salaries, benefits and
many regional branches as there are regional other emoluments. – The Chairman and
offices of the Department of Labor and members of the Commission shall have the same
Employment, sub-regional branches or provincial rank, receive an annual salary equivalent to, and
extension units. There shall be as many Labor be entitled to the same allowances, retirement and
Arbiters as may be necessary for the effective and benefits as, those of the Presiding Justice and
efficient operation of the Commission. (As Associate Justices of the Court of Appeals,
amended by Section 2, Republic Act No. 9347 [July respectively. Labor Arbiters shall have the same
27, 2006] and previously amended by Section 6, rank, receive an annual salary equivalent to and be
Republic Act No. 6715 [March 21, 1989]). entitled to the same allowances, retirement and
other benefits and privileges as those of the judges
of the regional trial courts. In no case, however,
Article 222, Labor Code. Appointment and shall the provision of this Article result in the
Qualifications. - The Chairman and other diminution of the existing salaries, allowances and
Commissioners shall be members of the Philippine benefits of the aforementioned officials.
Bar and must have been engaged in the practice of
law in the Philippines for at least fifteen (15) years,
with at least five (5) years experience or exposure Article 224, Labor Code. Article. 217. Jurisdiction
in the field of labor-management relations, and of the Labor Arbiters and the Commission. - (a)
shall preferably be residents of the region where Except as otherwise provided under this Code, the
they shall hold office. The Labor Arbiters shall Labor Arbiters shall have original and exclusive
likewise be members of the Philippine Bar and must jurisdiction to hear and decide, within thirty (30)
have been engaged in the practice of law in the calendar days after the submission of the case by
Philippines for at least ten (10) years, with at least the parties for decision without extension, even in
five (5) years experience or exposure in the field of the absence of stenographic notes, the following
labor-management relations. cases involving all workers, whether agricultural or
non-agricultural:
The Chairman and the other Commissioners and 1. Unfair labor practice cases;
the Labor Arbiters shall hold office during good 2. Termination disputes;
behavior until they reach the age of sixty-five (65) 3. If accompanied with a claim for
years, unless sooner removed for cause as reinstatement, those cases that workers
provided by law or become incapacitated to may file involving wages, rates of pay,
discharge the duties of their office; Provided, hours of work and other terms and
however, That the President of the Republic of the conditions of employment;
Philippines may extend the services of the 4. Claims for actual, moral, exemplary and
Commissioners and Labor Arbiters up to the other forms of damages arising from the
maximum age of seventy (70) years upon the employer-employee relations;
recommendation of the Commission en banc. 5. Cases arising from any violation of
Article 264 of this Code, including
The Chairman, the Division Presiding questions involving the legality of strikes
Commissioners and other Commissioners shall all and lockouts; and
be appointed by the President. Appointment to any 6. Except claims for Employees
vacancy in a specific division shall come only from Compensation, Social Security, Medicare
the nominees of the sector which nominated the and maternity benefits, all other claims
predecessor. The Labor Arbiters shall also be arising from employer-employee relations,
appointed by the President, upon recommendation including those of persons in domestic or
of the Commission en banc to a specific arbitration household service, involving an amount
branch, preferably in the region where they are exceeding five thousand pesos (P5,000.00)
residents, and shall be subject to the Civil Service regardless of whether accompanied with a
Law, rules and regulations: Provided, that the Labor claim for reinstatement.
Arbiters who are presently holding office in the
region where they are residents shall be deemed (b) The Commission shall have exclusive appellate
appointed thereat. jurisdiction over all cases decided by Labor
Arbiters.
The Chairman and the Commission, shall appoint
the staff and employees of the Commission, and its (c) Cases arising from the interpretation or
regional branches as the needs of the service may implementation of collective bargaining
require, subject to the Civil Service Law, rules and agreements and those arising from the
regulations, and upgrade their current salaries, interpretation or enforcement of company
benefits and other emoluments in accordance with personnel policies shall be disposed of by the Labor
law. (As amended by Section 3, Republic Act No. Arbiter by referring the same to the grievance
9347 [July 27, 2006] and as previously amended by machinery and voluntary arbitration as may be
Section 7, Republic Act No. 6715 [March 21, 1989]). provided in said agreements.

87
Article 225, Labor Code. Powers of the execution of the judgment shall be suspended
Commission. - The Commission shall have the pending the resolution of the appeal upon the filing
power and authority: by such person of a bond on condition that he will
(a) To promulgate rules and regulations governing abide by and perform the judgment of the
the hearing and disposition of cases before it and Commission should the appeal be decided against
its regional branches, as well as those pertaining to him. Judgment of the Commission on direct
its internal functions and such rules and regulations contempt is immediately executory and
as may be necessary to carry out the purposes of unappealable. Indirect contempt shall be dealt with
this Code; (As amended by Section 10, Republic Act by the Commission or Labor Arbiter in the manner
No. 6715, March 21, 1989). prescribed under Rule 71 of the Revised Rules of
Court; and (As amended by Section 10, Republic
(b) To administer oaths, summon the parties to a Act No. 6715, March 21, 1989).
controversy, issue subpoenas requiring the
attendance and testimony of witnesses or the (e) To enjoin or restrain any actual or threatened
production of such books, papers, contracts, commission of any or all prohibited or unlawful acts
records, statement of accounts, agreements, and or to require the performance of a particular act in
others as may be material to a just determination any labor dispute which, if not restrained or
of the matter under investigation, and to testify in performed forthwith, may cause grave or
any investigation or hearing conducted in irreparable damage to any party or render
pursuance of this Code; ineffectual any decision in favor of such party:
Provided, That no temporary or permanent
(c) To conduct investigation for the determination injunction in any case involving or growing out of a
of a question, matter or controversy within its labor dispute as defined in this Code shall be
jurisdiction, proceed to hear and determine the issued except after hearing the testimony of
disputes in the absence of any party thereto who witnesses, with opportunity for cross-examination,
has been summoned or served with notice to in support of the allegations of a complaint made
appear, conduct its proceedings or any part thereof under oath, and testimony in opposition thereto, if
in public or in private, adjourn its hearings to any offered, and only after a finding of fact by the
time and place, refer technical matters or accounts Commission, to the effect:
to an expert and to accept his report as evidence
after hearing of the parties upon due notice, direct (1) That prohibited or unlawful acts have been
parties to be joined in or excluded from the threatened and will be committed and will be
proceedings, correct, amend, or waive any error, continued unless restrained, but no injunction or
defect or irregularity whether in substance or in temporary restraining order shall be issued on
form, give all such directions as it may deem account of any threat, prohibited or unlawful act,
necessary or expedient in the determination of the except against the person or persons, association
dispute before it, and dismiss any matter or refrain or organization making the threat or committing
from further hearing or from determining the the prohibited or unlawful act or actually
dispute or part thereof, where it is trivial or where authorizing or ratifying the same after actual
further proceedings by the Commission are not knowledge thereof;
necessary or desirable; and (2) That substantial and irreparable injury to
complainant's property will follow;
(d) To hold any person in contempt directly or (3) That as to each item of relief to be granted,
indirectly and impose appropriate penalties greater injury will be inflicted upon complainant by
therefor in accordance with law. the denial of relief than will be inflicted upon
defendants by the granting of relief;
A person guilty of misbehavior in the presence of or (4) That complainant has no adequate remedy at
so near the Chairman or any member of the law; and
Commission or any Labor Arbiter as to obstruct or (5) That the public officers charged with the duty to
interrupt the proceedings before the same, protect complainant's property are unable or
including disrespect toward said officials, offensive unwilling to furnish adequate protection.
personalities toward others, or refusal to be sworn,
or to answer as a witness or to subscribe an Such hearing shall be held after due and personal notice
affidavit or deposition when lawfully required to do thereof has been served, in such manner as the
so, may be summarily adjudged in direct contempt Commission shall direct, to all known persons against
by said officials and punished by fine not exceeding whom relief is sought, and also to the Chief Executive and
five hundred pesos (P500) or imprisonment not other public officials of the province or city within which the
exceeding five (5) days, or both, if it be the unlawful acts have been threatened or committed, charged
Commission, or a member thereof, or by a fine not with the duty to protect complainant’s property:
exceeding one hundred pesos (P100) or Provided, however, that if a complainant shall also allege
imprisonment not exceeding one (1) day, or both, if that, unless a temporary restraining order shall be issued
it be a Labor Arbiter. without notice, a substantial and irreparable injury to
complainant’s property will be unavoidable, such a
The person adjudged in direct contempt by a Labor temporary restraining order may be issued upon testimony
Arbiter may appeal to the Commission and the under oath, sufficient, if sustained, to justify the

88
Commission in issuing a temporary injunction upon legal counsel but it shall be the duty of the
hearing after notice. Such a temporary restraining order Chairman, any Presiding Commissioner or
shall be effective for no longer than twenty (20) days and Commissioner or any Labor Arbiter to exercise
shall become void at the expiration of said twenty (20) complete control of the proceedings at all stages.
days. No such temporary restraining order or temporary
injunction shall be issued except on condition that Any provision of law to the contrary
complainant shall first file an undertaking with adequate notwithstanding, the Labor Arbiter shall exert all
security in an amount to be fixed by the Commission efforts towards the amicable settlement of a labor
sufficient to recompense those enjoined for any loss, dispute within his jurisdiction on or before the first
expense or damage caused by the improvident or hearing. The same rule shall apply to the
erroneous issuance of such order or injunction, including Commission in the exercise of its original
all reasonable costs, together with a reasonable attorney's jurisdiction
fee, and expense of defense against the order or against
the granting of any injunctive relief sought in the same
proceeding and subsequently denied by the Commission.
Article 228, Labor Code. Appearances and Fees.
- (a) Non-lawyers may appear before the
The undertaking herein mentioned shall be
Commission or any Labor Arbiter only:
understood to constitute an agreement entered
1. If they represent themselves; or 2. If they
into by the complainant and the surety upon which
represent their organization or members thereof.
an order may be rendered in the same suit or
proceeding against said complainant and surety,
(b) No attorney's fees, negotiation fees or similar charges
upon a hearing to assess damages, of which
of any kind arising from any collective bargaining
hearing, complainant and surety shall have
agreement shall be imposed on any individual member of
reasonable notice, the said complainant and surety
the contracting union: Provided, However, that
submitting themselves to the jurisdiction of the
attorney’s fees may be charged against union funds in
Commission for that purpose. But nothing herein
an amount to be agreed upon by the parties. Any contract,
contained shall deprive any party having a claim or agreement or arrangement of any sort to the contrary shall
cause of action under or upon such undertaking be null and void.
from electing to pursue his ordinary remedy by suit
at law or in equity: Provided, further, That the
reception of evidence for the application of a writ of
injunction may be delegated by the Commission to Article 229, Labor Code. Appeal. - Decisions,
any of its Labor Arbiters who shall conduct such awards, or orders of the Labor Arbiter are final and
hearings in such places as he may determine to be executory unless appealed to the Commission by
accessible to the parties and their witnesses and any or both parties within ten (10) calendar days
shall submit thereafter his recommendation to the from receipt of such decisions, awards, or orders.
Commission. Such appeal may be entertained only on any of the
following grounds:

(a) If there is prima facie evidence of abuse of


Article 226, Labor Code. Ocular inspection. - The discretion on the part of the Labor Arbiter;
Chairman, any Commissioner, Labor Arbiter or their (b) If the decision, order or award was secured
duly authorized representatives, may, at any time through fraud or coercion, including graft and
during working hours, conduct an ocular inspection corruption;
on any establishment, building, ship or vessel, (c) If made purely on questions of law; and
place or premises, including any work, material, (d) If serious errors in the findings of facts are
implement, machinery, appliance or any object raised which would cause grave or irreparable
therein, and ask any employee, laborer, or any damage or injury to the appellant.
person, as the case may be, for any information or In case of a judgment involving a monetary award,
data concerning any matter or question relative to an appeal by the employer may be perfected only
the object of the investigation. upon the posting of a cash or surety bond issued by
a reputable bonding company duly accredited by
the Commission in the amount equivalent to the
Article 227, Labor Code. Technical rules not monetary award in the judgment appealed from.
binding and prior resort to amicable settlement. - In In any event, the decision of the Labor Arbiter
any proceeding before the Commission or any of reinstating a dismissed or separated employee,
the Labor Arbiters, the rules of evidence prevailing insofar as the reinstatement aspect is concerned,
in courts of law or equity shall not be controlling shall immediately be executory, even pending
and it is the spirit and intention of this Code that appeal. The employee shall either be admitted
the Commission and its members and the Labor back to work under the same terms and conditions
Arbiters shall use every and all reasonable means prevailing prior to his dismissal or separation or, at
to ascertain the facts in each case speedily and the option of the employer, merely reinstated in
objectively and without regard to technicalities of the payroll. The posting of a bond by the employer
law or procedure, all in the interest of due process. shall not stay the execution for reinstatement
In any proceeding before the Commission or any provided herein.
Labor Arbiter, the parties may be represented by

89
To discourage frivolous or dilatory appeals, the exclusive authority to act, at their own initiative or
Commission or the Labor Arbiter shall impose upon request of either or both parties, on all inter-
reasonable penalty, including fines or censures, union and intra-union conflicts, and all disputes,
upon the erring parties. grievances or problems arising from or affecting
labor-management relations in all workplaces,
In all cases, the appellant shall furnish a copy of whether agricultural or non-agricultural, except
the memorandum of appeal to the other party who those arising from the implementation or
shall file an answer not later than ten (10) calendar interpretation of collective bargaining agreements
days from receipt thereof. which shall be the subject of grievance procedure
and/or voluntary arbitration.
The Commission shall decide all cases within
twenty (20) calendar days from receipt of the The Bureau shall have fifteen (15) working days to
answer of the appellee. The decision of the act on labor cases before it, subject to extension by
Commission shall be final and executory after ten agreement of the parties.
(10) calendar days from receipt thereof by the
parties.
Article 273, Labor Code. Grievance machinery
Any law enforcement agency may be deputized by and voluntary arbitration. - The parties to a
the Secretary of Labor and Employment or the Collective Bargaining Agreement shall include
Commission in the enforcement of decisions, therein provisions that will ensure the mutual
awards or orders. observance of its terms and conditions. They shall
establish a machinery for the adjustment and
resolution of grievances arising from the
Article 230, Labor Code. Execution of decisions, interpretation or implementation of their Collective
orders or awards. - (a) The Secretary of Labor and Bargaining Agreement and those arising from the
Employment or any Regional Director, the interpretation or enforcement of company
Commission or any Labor Arbiter, or Med-Arbiter or personnel policies.
Voluntary Arbitrator may, motu proprio or on All grievances submitted to the grievance
motion of any interested party, issue a writ of machinery which are not settled within seven (7)
execution on a judgment within five (5) years from calendar days from the date of its submission shall
the date it becomes final and executory, requiring a automatically be referred to voluntary arbitration
sheriff or a duly deputized officer to execute or prescribed in the Collective Bargaining Agreement.
enforce final decisions, orders or awards of the For this purpose, parties to a Collective Bargaining
Secretary of Labor and Employment or regional Agreement shall name and designate in advance a
director, the Commission, the Labor Arbiter or med- Voluntary Arbitrator or panel of Voluntary
arbiter, or voluntary arbitrators. In any case, it shall Arbitrators, or include in the agreement a
be the duty of the responsible officer to separately procedure for the selection of such Voluntary
furnish immediately the counsels of record and the Arbitrator or panel of Voluntary Arbitrators,
parties with copies of said decisions, orders or preferably from the listing of qualified Voluntary
awards. Failure to comply with the duty prescribed Arbitrators duly accredited by the Board. In case
herein shall subject such responsible officer to the parties fail to select a Voluntary Arbitrator or
appropriate administrative sanctions. panel of Voluntary Arbitrators, the Board shall
designate the Voluntary Arbitrator or panel of
(b) The Secretary of Labor and Employment, and Voluntary Arbitrators, as may be necessary,
the Chairman of the Commission may designate pursuant to the selection procedure agreed upon in
special sheriffs and take any measure under the Collective Bargaining Agreement, which shall
existing laws to ensure compliance with their act with the same force and effect as if the
decisions, orders or awards and those of the Labor Arbitrator or panel of Arbitrators has been selected
Arbiters and voluntary arbitrators, including the by the parties as described above.
imposition of administrative fines which shall not
be less than P500.00 nor more than P10,000.00.
Article 274, Labor Code. Jurisdiction of Voluntary
Arbitrators or panel of Voluntary Arbitrators. - The
Article 231, Labor Code. Contempt powers of the Voluntary Arbitrator or panel of Voluntary
Secretary of Labor. - In the exercise of his powers Arbitrators shall have original and exclusive
under this Code, the Secretary of Labor may hold jurisdiction to hear and decide all unresolved
any person in direct or indirect contempt and grievances arising from the interpretation or
impose the appropriate penalties therefor. implementation of the Collective Bargaining
Agreement and those arising from the
interpretation or enforcement of company
Article 232, Labor Code. Bureau of Labor personnel policies referred to in the immediately
Relations. - The Bureau of Labor Relations and the preceding article. Accordingly, violations of a
Labor Relations Divisions in the regional offices of Collective Bargaining Agreement, except those
the Department of Labor, shall have original and which are gross in character, shall no longer be
treated as unfair labor practice and shall be

90
resolved as grievances under the Collective agreement to execute the final decision, order or
Bargaining Agreement. For purposes of this article, award.
gross violations of Collective Bargaining Agreement
shall mean flagrant and/or malicious refusal to
comply with the economic provisions of such Article 277, Labor Code. Cost of voluntary
agreement. arbitration and Voluntary Arbitrator's fee. - The
parties to a Collective Bargaining Agreement shall
The Commission, its Regional Offices and the provide therein a proportionate sharing scheme on
Regional Directors of the Department of Labor and the cost of voluntary arbitration including the
Employment shall not entertain disputes, Voluntary Arbitrator's fee. The fixing of fee of
grievances or matters under the exclusive and Voluntary Arbitrators, whether shouldered wholly
original jurisdiction of the Voluntary Arbitrator or by the parties or subsidized by the Special
panel of Voluntary Arbitrators and shall Voluntary Arbitration Fund, shall take into account
immediately dispose and refer the same to the the following factors:
Grievance Machinery or Voluntary Arbitration (a) Nature of the case;
provided in the Collective Bargaining Agreement. (b) Time consumed in hearing the case;
(c) Professional standing of the Voluntary
Arbitrator;
Article 275, Labor Code. Jurisdiction over other (d) Capacity to pay of the parties; and
labor disputes. - The Voluntary Arbitrator or panel (e) Fees provided for in the Revised Rules
of Voluntary Arbitrators, upon agreement of the of Court.
parties, shall also hear and decide all other labor
disputes including unfair labor practices and
bargaining deadlocks. Article 278(g), Labor Code.
(g) When, in his opinion, there exists a labor
dispute causing or likely to cause a strike or lockout
Article 276, Labor Code. Procedures. - The in an industry indispensable to the national
Voluntary Arbitrator or panel of Voluntary interest, the Secretary of Labor and Employment
Arbitrators shall have the power to hold hearings, may assume jurisdiction over the dispute and
receive evidences and take whatever action is decide it or certify the same to the Commission for
necessary to resolve the issue or issues subject of compulsory arbitration. Such assumption or
the dispute, including efforts to effect a voluntary certification shall have the effect of automatically
settlement between parties. enjoining the intended or impending strike or
lockout as specified in the assumption or
All parties to the dispute shall be entitled to attend certification order. If one has already taken place at
the arbitration proceedings. The attendance of any the time of assumption or certification, all striking
third party or the exclusion of any witness from the or locked out employees shall immediately return-
proceedings shall be determined by the Voluntary to-work and the employer shall immediately
Arbitrator or panel of Voluntary Arbitrators. Hearing resume operations and readmit all workers under
may be adjourned for cause or upon agreement by the same terms and conditions prevailing before
the parties. the strike or lockout. The Secretary of Labor and
Employment or the Commission may seek the
Unless the parties agree otherwise, it shall be assistance of law enforcement agencies to ensure
mandatory for the Voluntary Arbitrator or panel of compliance with this provision as well as with such
Voluntary Arbitrators to render an award or orders as he may issue to enforce the same.
decision within twenty (20) calendar days from the
date of submission of the dispute to voluntary In line with the national concern for and the highest
arbitration. respect accorded to the right of patients to life and
health, strikes and lockouts in hospitals, clinics and
The award or decision of the Voluntary Arbitrator or similar medical institutions shall, to every extent
panel of Voluntary Arbitrators shall contain the possible, be avoided, and all serious efforts, not
facts and the law on which it is based. It shall be only by labor and management but government as
final and executory after ten (10) calendar days well, be exhausted to substantially minimize, if not
from receipt of the copy of the award or decision by prevent, their adverse effects on such life and
the parties. health, through the exercise, however legitimate,
by labor of its right to strike and by management to
Upon motion of any interested party, the Voluntary lockout. In labor disputes adversely affecting the
Arbitrator or panel of Voluntary Arbitrators or the continued operation of such hospitals, clinics or
Labor Arbiter in the region where the movant medical institutions, it shall be the duty of the
resides, in case of the absence or incapacity of the striking union or locking-out employer to provide
Voluntary Arbitrator or panel of Voluntary and maintain an effective skeletal workforce of
Arbitrators, for any reason, may issue a writ of medical and other health personnel, whose
execution requiring either the sheriff of the movement and services shall be unhampered and
Commission or regular courts or any public official unrestricted, as are necessary to insure the proper
whom the parties may designate in the submission and adequate protection of the life and health of its

91
patients, most especially emergency cases, for the of the Municipal or City Courts and the Courts of
duration of the strike or lockout. In such cases, First Instance.
therefore, the Secretary of Labor and Employment
may immediately assume, within twenty four (24)
hours from knowledge of the occurrence of such a Article 304, Labor Code. Who are liable when
strike or lockout, jurisdiction over the same or committed by other than natural person. - If the
certify it to the Commission for compulsory offense is committed by a corporation, trust, firm,
arbitration. For this purpose, the contending parties partnership, association or any other entity, the
are strictly enjoined to comply with such orders, penalty shall be imposed upon the guilty officer or
prohibitions and/or injunctions as are issued by the officers of such corporation, trust, firm, partnership,
Secretary of Labor and Employment or the association or entity.
Commission, under pain of immediate disciplinary
action, including dismissal or loss of employment
status or payment by the locking-out employer of
backwages, damages and other affirmative relief, Article 305, Labor Code. Offenses. - Offenses
even criminal prosecution against either or both of penalized under this Code and the rules and
them. regulations issued pursuant thereto shall prescribe
in three (3) years.
The foregoing notwithstanding, the President of the
Philippines shall not be precluded from determining All unfair labor practice arising from Book V shall be
the industries that, in his opinion, are indispensable filed with the appropriate agency within one (1)
to the national interest, and from intervening at year from accrual of such unfair labor practice;
any time and assuming jurisdiction over any such otherwise, they shall be forever barred.
labor dispute in order to settle or terminate the
same.
Article 306, Labor Code. Money claims. - All
money claims arising from employer-employee
Article 278(h), Labor Code. relations accruing during the effectivity of this Code
(h) Before or at any stage of the compulsory shall be filed within three (3) years from the time
arbitration process, the parties may opt to submit the cause of action accrued; otherwise they shall
their dispute to voluntary arbitration. be forever barred.

All money claims accruing prior to the effectivity of


this Code shall be filed with the appropriate entities
Article 278(i), Labor Code. established under this Code within one (1) year
(i) The Secretary of Labor and Employment, the from the date of effectivity, and shall be processed
Commission or the voluntary arbitrator shall decide or determined in accordance with the
or resolve the dispute, as the case may be. The implementing rules and regulations of the Code;
decision of the President, the Secretary of Labor otherwise, they shall be forever barred.
and Employment, the Commission or the voluntary
arbitrator shall be final and executory ten (10) Workmen's compensation claims accruing prior to
calendar days after receipt thereof by the parties. the effectivity of this Code and during the period
from November 1, 1974 up to December 31, 1974,
shall be filed with the appropriate regional offices
Article 303, Labor Code. Penalties. - Except as of the Department of Labor not later than March
otherwise provided in this Code, or unless the acts 31, 1975; otherwise, they shall forever be barred.
complained of hinge on a question of interpretation The claims shall be processed and adjudicated in
or implementation of ambiguous provisions of an accordance with the law and rules at the time their
existing collective bargaining agreement, any causes of action accrued.
violation of the provisions of this Code declared to
be unlawful or penal in nature shall be punished
with a fine of not less than One Thousand Pesos Article 307, Labor Code. Institution of money
(P1,000.00) nor more than Ten Thousand Pesos claims. - Money claims specified in the immediately
(P10,000.00) or imprisonment of not less than preceding Article shall be filed before the
three months nor more than three years, or both appropriate entity independently of the criminal
such fine and imprisonment at the discretion of the action that may be instituted in the proper courts.
court.
Pending the final determination of the merits of
In addition to such penalty, any alien found guilty money claims filed with the appropriate entity, no
shall be summarily deported upon completion of civil action arising from the same cause of action
service of sentence. shall be filed with any court. This provision shall not
apply to employees compensation case which shall
Any provision of law to the contrary be processed and determined strictly in accordance
notwithstanding, any criminal offense punished in with the pertinent provisions of this Code.
this Code, shall be under the concurrent jurisdiction

92
Single Entry Approach the existence of an employer-employee relationship.
No procedure was laid down where the DOLE would
Read Republic Act no. 10396 and DO 151-16 only make a preliminary finding, and that the power
was primarily held by the NLRC. The DOLE, in
determining the existence of an employer-employee
Secretary / Regional Director relationship, can use the elements to determine the
existence of an employment relationship.[1] The use
People’s Broadcasting v. Secretary, May of this test is not solely limited to the NLRC. The DOLE
Secretary, or his or her representatives, can utilize
8, 2009 and March 6, 2012 the same test, even in the course of inspection,
May 8, 2009: Respondent filed a complaint against making use of the same evidence that would have
Bombo Radyo for illegal deduction, non-payment of been presented before the NLRC. Also, there is still a
service incentive leave, 13th month pay, premium threshold amount set by the Labor Code when money
pay for holiday and rest day and illegal diminution of claims are involved, in that if it is PhP 5,000 and
benefits, delayed payment of wages and non- below, the jurisdiction is with the regional director of
coverage of SSS, PAG-IBIG and Philhealth before the the DOLE, and if the amount involved exceeds PhP
DOLE. Bombo Radyo denied that there is an 5,000, the jurisdiction is with the labor arbiter.
employer-employee relationship with the respondent However, despite the wording of the Code, this would
since respondent is a drama talent hired on a per only apply in the course of regular inspections
drama "participation basis." DOLE ruled that undertaken by the DOLE. In conclusion, if a complaint
respondent is an employee of petitioner Bombo is brought before the DOLE to give effect to the labor
Radyo and that the former is entitled to his money standards provisions of the Labor Code or other labor
claims. Upon appeal to CA, Bombo Radyo reiterated legislation, and there is a finding by the DOLE that
its claim and added that the case was beyond the there is an existing employer-employee relationship,
jurisdiction of the DOLE and should have been the DOLE exercises jurisdiction to the exclusion of the
considered by the labor arbiter because respondent NLRC. If the DOLE finds that there is no employer-
Juenzan’s claim exceeded P5,000.00. CA ruled that employee relationship, the jurisdiction is with the
the DOLE Secretary had the power to order and NLRC. If a complaint is filed with the DOLE, and it is
enforce compliance with labor standard laws accompanied by a claim for reinstatement, the
irrespective of the amount of individual claims jurisdiction is with the Labor Arbiter, under Article
because the limitation imposed by Article 29 of the 217(3) of the Labor Code. If a complaint is filed with
Labor Code had been repealed by RA No. 7730. the NLRC, and there is still an existing employer-
ISSUE: Does the NLRC and not the DOLE Secretary, employee relationship, the jurisdiction is with DOLE.
has jurisdiction over respondent’s claim? HELD:
Clearly the law accords a prerogative to the NLRC [1] (1) The selection and engagement of the
over the claim when the employer-employee employee; (2) the payment of wages; (3) the power
relationship has terminated or such relationship has of dismissal; and (4) the employer’s power to control
not arisen at all. In the second situation especially, the employee’s conduct.
the existence of an employer-employee relationship is
a matter which is not easily determinable from an
ordinary inspection, because the elements of such a Republic of the Philippines v. Namboku
relationship are not verifiable from a mere ocular Peak, July 18, 2014
examination. The existence of an employer-employee Can the Secretary of Labor file a petition for review
relationship is a statutory prerequisite to and a on certiorari against the ruling of the CA, which
limitation on the power of the Secretary of Labor. The reversed it? The Secretary of Labor is a nominal party
rationale underlying this limitation is to eliminate the because of her decision but she is not the real party
prospect of competing conclusions of the Secretary of in interest vested with personality to file the present
Labor and the NLRC, on a matter of questions of fact petitions. A real party-in-interest is the party who
and law, which is best resolved by the quasi-judicial stands to be benefited or injured by the judgment in
body, NRLC, rather than an administrative official of the suit, or the party entitled to the avails of the suit.
the executive branch of the government.Therefore, The real party in interest would have been the unions
since evidence offered puts in doubt the existence of to appear and defend the ruling of the Secretary of
employer-employee relationship, DOLE should have Labor. Only real parties-in-interest who participated in
referred respondent Juezan to the NLRC for the proper the litigation of the case before the CA can avail of an
dispensation of his claims. appeal by certiorari.

March 6, 2012: SC said that no limitation in the law


was placed upon the power of the DOLE to determine

93
of the rights and conditions of union membership
provided for in the Code. The controversy in the case
South Cotabato Communications at bar is an intra-union dispute as it involves a
Corporation v. Hon. Sto. Tomas, June 15, dispute within FLAMES and the issue is propriety of
2016 disqualification of private respondents Daya et al.
DOLE conducted a Complaint Inspection at the Even as the dispute involves allegations that private
premises of DXCP Radio Station, owned by petitioner. respondents Daya, et al., sought the help of non-
The inspection yielded a finding of violations of labor members of the union in their election campaign to
standards.[1] Consequently, DOLE issued a Notice of the detriment of FLAMES, the same remains as one
Inspection Result directing the petitioner to effect which involves the grievance over the constitution
restitution and correction of the alleged violations. and bylaws of a union. Thus, petition is within
However, due to petitioner’s failure to comply, DOLE cognizance of BLR.
scheduled Summary Investigation. However,
petitioner failed to appear despite due notice. Thus, [1]ART. 226. BUREAU OF LABOR RELATIONS. The
DOLE Regional Director directed petitioners to pay Bureau of Labor Relations and the Labor Relations
private respondents their claim for wage differentials Divisions in the regional offices of the Department of
among other monetary claims. ISSUE: Does the DOLE Labor shall have original and exclusive authority to
have jurisdiction despite not having established yet act, at their own initiative or upon request of either or
EER relationship? HELD: While the existence of an both parties, on all inter-union and intra-union
employer-employee relationship is a statutory conflicts, and all disputes, grievances or problems
prerequisite to and a limitation on the power of the arising from or affecting labor-management relations
Secretary of Labor, one which the legislative branch in all workplaces whether agricultural or
is entitled to impose, like the NLRC, the DOLE has the nonagricultural, except those arising from the
authority to rule on the existence of an employer- implementation or interpretation of collective
employee relationship between the parties, bargaining agreements which shall be the subject of
considering that the existence of an employer- grievance procedure and/or voluntary arbitration.
employee relationship is a condition sine qua non for
the exercise of its visitorial power. Nevertheless, it
must be emphasized that without an employer- Labor Arbiter
employee relationship, or if one has already been
terminated, the Secretary of Labor is without Jaguar Security v. Sales, April 22, 2008
jurisdiction to determine if violations of labor Can a labor contractor file a reimbursement claim
standards provision had in fact been committed, and against its principal before the Labor Arbiter/NLRC?
to direct employers to comply with their alleged HELD: The contractor cannot claim reimbursement
violations of labor standards. from the principal through a cross-claim filed with the
labor court, because the claim is within the realm of
[1] Labor standards violations: Underpayment of civil law and jurisdiction over the case belongs to the
wages and 13th month pay, non-payment of the 5 regular courts. While the resolution of the issue
days SIL, non-payment of Rest Day Premium Pay and involves the application of labor laws, reference to
Holiday Premium Pay, non-remittance of SSS the labor code was only for the determination of the
Contributions, and some employees are paid on solidary liability of the petitioner to the respondent
commission basis aside from their allowances. where no employer-employee relation exists. In this
case, there is no employer-employee relationship
between Jaguar and Delta Milling. In its cross-claim,
Bureau of Labor Relations Jaguar is not seeking any relief under the Labor Code
but merely reimbursement of the monetary benefits
Diokno v. Cacdac, July 4, 2007 claims awarded and to be paid to the guard
In interpreting Article 226[1], BLR has the original and employees. There is no labor dispute involved in the
exclusive jurisdiction on all inter-union and intra- cross-claim against Delta Milling. Rather, the cross-
union conflicts. An intra-union dispute is defined, claim involves a civil dispute between petitioner and
under Section z, Rule I of Implementing Rules Book V, Delta Milling.
as referring to any conflict between and among union
members, and includes all disputes or grievances Pioneer Concrete Philippines v. Todaro,
arising from any violation of or disagreement over
any provision of the constitution and by-laws of a
June 8, 2007
union, including cases arising from chartering or As to the question of jurisdiction, this Court has
affiliation of labor organizations or from any violation consistently held that where no employer-employee
relationship exists between the parties and no issue
94
is involved which may be resolved by reference to the basis for the claim arises from or is necessarily
Labor Code, other labor statutes or any collective connected with the fact of termination, and should be
bargaining agreement, it is the Regional Trial Court entered as a counterclaim in the illegal dismissal
that has jurisdiction. In the present case, no case. In this case, Petitioners’ claim that they have
employer-employee relationship exists between the right to the immediate release of their benefits as
petitioners and respondent. In fact, in his complaint, employees separated from respondent Solid Mills is a
private respondent is not seeking any relief under the question arising from the employer-employee
Labor Code, but seeks payment of damages on relationship between the parties. Thus, there is
account of petitioners' alleged breach of their jurisdiction.
obligation under their agreement to employ him. It is
settled that an action for breach of contractual
obligation is intrinsically a civil dispute. In the
Indophil v. Adviento, August 4, 2014
alternative, respondent seeks redress on the basis of Not all claims involving employees can be resolved
the provisions of Articles 19 and 21 of the Civil Code. solely by labor courts, specifically when the law
Hence, it is clear that the present action is within the provides otherwise. The Court formulated the
realm of civil law, and jurisdiction over it belongs to “reasonable causal connection rule”, wherein if there
the regular courts. is a reasonable causal connection between claim
asserted and EE-ER relations, the case is within the
jurisdiction of the labor courts; and in the absence
Tegimenta Chemical Phils. v. Buensalida, thereof, its is the regular courts that have jurisdiction.
June 17, 2008 Indeed, jurisprudence has evolved the rule that
A party having more than one cause of action against claims for damaged under Art 217(a)(4) LC, to be
the other party arising out of the same relationship cognizable by LA, must have a reasonable causal
shall include all of them in one complaint or petition. connection with any of the claims provided for in the
As stated earlier, however, the inclusion of article. Only of there is such a connection with other
respondents cause of action for constructive illegal claims can a claim for damages be considered as
dismissal in the Davao case could not have been arising from EE-ER relationship. In this case, such
possible since the same arose only after the latter connection is nil. Adviento’s claim for damages is
case was filed. At the time of the filing of the Davao specifically grounded on Indophil’s gross negligence
case, respondent could not have yet claimed that to provide a safe, healthy and workable environment
petitioner committed acts that would amount to for its EEs – a case of quasi-delict – hence, within the
constructive illegal dismissal. Thus, the ambit of regular court’s jurisdiction. Adviento alleged
aforementioned rule has no application in this case. that due to continued and prolonged exposure to
textile dust seriously inimical to his health, he
suffered work-contracted disease which is now
Milan v. NLRC, February 4, 2015 irreversible and incurable, and deprived him of job
The NLRC has jurisdiction to determine, preliminarily, opportunities. Clearly, injury and damages were
the parties’ rights over a property, when it is suffered by Adviento, an element of quasi-delict. It
necessary to determine an issue related to rights or also bears stressing that the respondent is not
claims arising from an employer-employee praying for any relief under LC. He neither claims for
relationship. Article 217 provides that the Labor reinstatement nor backwages or separation pay
Arbiter, in his or her original jurisdiction, and the resulting from illegal dismissal. The cause of action
NLRC, in its appellate jurisdiction, may determine herein pertains to consequence of Indophil’s omission
issues involving claims arising from employer- which led to a work-related disease suffered by
employee relations. Claims arising from an employer- respondent, causing harm or damage to his person.
employee relationship are not limited to claims by an Such cause of action is within the realm of Civil Law,
employee. Employers may also have claims against and jurisdiction over controversy belongs to regular
the employee, which arise from the same courts. Where the resolution of dispute requires
relationship. Article 217 of the Labor Code also expertise, not in labor management relations nor in
applies to employers’ claim for damages, which wage structures and other terms and conditions of
arises from or is connected with the labor issue. Thus: employment, but rather in application of general civil
Whereas this Court in a number of occasions had law, such claim falls outside the area of competence
applied the jurisdictional provisions of Article 217 to of expertise ordinarily ascribed to LA and NLRC. RTC
claims for damages filed by employees, we hold that has jurisdiction over subject matter of Adviento’s
by the designating clause “arising from the employer- complaint praying for moral damages, exemplary
employee relations” Article 217 should apply with damages, compensatory damages, anchored on
equal force to the claim of an employer for actual Indophil’s alleged gross negligence in failing to
damages against its dismissed employee, where the provide a safe and healthy working environment.

95
Am-Phil Food Concepts v. Padilla, Malayan Insurance Company v.
October 1, 2014 Alibudbud, April 20, 2016
From the provisions of the 2002 Rules, it is clear that Alibudbud was dismissed from Malayan due to
a supplemental rejoinder is not a pleading which a redundancy. Malayan demanded that she surrender
labor arbiter is duty-bound to accept. Even following the possession of the car to the company however
changes to the NLRC Rules of Procedure in 2005 and she refused. Malayan instituted a Complaint for
2011, a rejoinder has not been recognized as a replevin and/or sum of money before the RTC and
pleading that labor arbiters must necessarily admit. prayed for the seizure of the car.
The 2005 and 2011 NLRC Rules of Procedure only go
so far as to recognize that a reply “may” be filed by Does the RTC have jurisdiction? Yes. A careful study of
the parties. Thus, Labor Arbiter Chuanico was under the case would reveal that the RTC correctly took
no obligation to grant Am-Phil’s motion for leave to cognizance of the action for replevin contrary to the
admit supplemental rejoinder and, hereby, consider pronouncement of the CA. Replevin is an action
the supplemental rejoinder’s averments and annexes. whereby the owner or person entitled to repossession
That Am-Phil had to file a motion seeking permission of goods or chattels may recover those goods or
to file its supplemental rejoinder (i.e., motion for chattels from one who has wrongfully distrained or
leave to file) is proof of its own recognition that the taken, or who wrongfully detains such goods or
labor arbiter is under no compulsion to accept any chattels. It is designed to permit one having right to
such pleading and that the supplemental rejoinder’s possession to recover property in specie from one
admission rests on the labor arbiter’s discretion. The who has wrongfully taken or detained the property.
requirements of due process in labor cases before a The term may refer either to the action itself, for the
Labor Arbiter is satisfied when the parties are given recovery of personalty, or to the provisional remedy
the opportunity to submit their position papers to traditionally associated with it, by which possession
which they are supposed to attach all the supporting of the property may be obtained by the plaintiff and
documents or documentary evidence that would retained during the pendency of the action. The
prove their respective claims, in the event that the present action involves the parties' relationship as
Labor Arbiter determines that no formal hearing debtor and creditor, not their "employer-employee"
would be conducted or that such hearing was not relationship. Malayan's demand for Alibudbud to pay
necessary. the 50% company equity over the car or, to surrender
its possession, is civil in nature. The trial court's
Pentagon Steel Corp. v. CA, June 26, ruling also aptly noted the Promissory Note and Deed
of Chattel Mortgage voluntarily signed by Alibudbud
2009
to secure her financial obligation to avail of the car
Information and statements made at conciliation
being offered under Malayan's Car Financing Plan.
proceedings shall be treated as privileged
Clearly, the issue in the replevin action is separate
communication and shall not be used as evidence in
and distinct from the illegal dismissal case. Even the
the Commission. Conciliators and similar officials
illegal dismissal case she heavily relied upon in
shall not testify in any court or body regarding any
moving for the suspension of the replevin action was
matters taken up at conciliation proceedings
settled in favor of Malayan which was merely found to
conducted by them.
have validly exercised its management prerogative in
order to improve its company sales.
Supra Multi-Services v. Tambunting,
August 3, 2016
As regards a managerial employee, the mere
Voluntary Arbitration
existence of a basis for believing that such employee
Negros Metal Corp. v. Lamayo, August
has breached the trust of his employer would suffice
for his dismissal. Hence, in the case of managerial 25, 2010
employees, proof beyond reasonable doubt is not The employee was terminated due to using a
required, it being sufficient that there is some basis different grinder, which was allegedly prohibited. He
for such loss of confidence, such as when the filed for illegal dismissal. The company argued that it
employer has reasonable ground to believe that the should be dismissed bc the LA did not have
employee concerned is responsible for the purported jurisdiction since under their CBA, the matter should
misconduct, and the nature of his participation have first been submitted before the company’s
therein renders him unworthy of the trust and grievance machinery. The particular section being
confidence demanded by his position. invoked by the company states that “the parties

96
hereto agree on principle that all disputes between Manila Pavilion v. Delada, January 25,
labor and management may be settled through
friendly negotiations that the parties have the same
2012
interest in the continuity of work until all points in Delada was originally the Head Waiter of the
dispute shall have been discussed and settled”. Rotisserie, MPH’s fine dining restaurant, but was later
reassigned as Head Waiter of MPH’s other restaurant.
Under Art. 217, it is clear that a labor arbiter has He declined the transfer and asked for a grievance
original and exclusive jurisdiction over termination meeting pursuant to the CBA. MPH told Delada to
disputes. On the other hand, under Article 261, a report to his new assignment temporarily, without
voluntary arbitrator has original and exclusive prejudice to the resolution of the grievance, but he
jurisdiction over grievances arising from the refused. Thus, MPH initiated administrative
interpretation or enforcement of company policies. As proceedings against him, and later sent him several
a general rule then, termination disputes should be memoranda requiring him to explain why he should
brought before a labor arbiter, except when the not be penalized for serious misconduct, willful
parties, under Art. 262, unmistakably express that disobedience, gross insubordination, gross and
they agree to submit the same to voluntary habitual neglect of duties and willful breach of trust.
arbitration. In the present case, the CBA provision on While Delada’s complaint concerning the validity of
grievance machinery being invoked by petitioner his transfer was pending before the Panel of
does not expressly state that termination disputes Voluntary Arbitrators (PVA), MPH continued with the
are included in the ambit of what may be brought disciplinary action against him for his refusal to report
before the company's grievance machinery. to his new post.

The voluntary arbitrator had plenary jurisdiction and


Albert Teng Fish Trading v. Pahagac, authority to interpret the agreement to arbitrate and
November 17, 2010 to determine the scope of his own authority—subject
As the owner of the business (Albert Teng Fish only, in a proper case, to the certiorari jurisdiction of
Trading), he claims that he customarily enters into this Court. In Ludo & Luym Corporation v. Saornido,
joint venture agreements with master fishermen Court recognized that voluntary arbitrators are
(maestros) who are skilled and are experts in deep generally expected to decide only those questions
sea fishing. The respondents were hired by the expressly delineated by the submission agreement;
maestros as checkers to determine the volume of the nevertheless, they can assume that they have the
fish caught. They filed a case for illegal dismissal necessary power to make a final settlement on the
claiming that an EE-relationship existed. The VA related issues, since arbitration is the final resort for
declared that there was no EE relationship. Thus, they the adjudication of disputes. Pursuant to the
filed a MR but this was denied by the VA who argued aforecited doctrines, the PVA was authorized to
that Sec. 6, Rule VII of the 1989 Procedural Guidelines assume jurisdiction over the related issue of
in the Conduct of Voluntary Arbitration Proceedings insubordination and willful disobedience of the
(1989 Procedural Guidelines) does NOT provide the transfer order. Nevertheless, the doctrine is
remedy of a motion for reconsideration to the party inapplicable here. The VAs assumed jurisdiction over
adversely affected by the VAs order or decision. the related issues and made rulings on the matter,
while here, PVA did not make a ruling on
VA’s decision is subject to a MR. Article 262-A of the insubordination & willful disobedience.
Labor Code does not prohibit the filing of a motion for
reconsideration. Notably, Article 262-A deleted the
word “unappealable” from Article 263. The deliberate
Honda Cars v. Honda Cars Technical
selection of the language in the amendatory act Specialists and Supervisors, November
differing from that of the original act indicates that 19, 2014
the legislature intended a change in the law, and the A CBA was entered. The union members had a
court should endeavor to give effect to such intent. In transport allowance of P3,300 a month. The same
a number of cases, SC ruled that this change means parties entered into a MOA whereby the
that the decision may still be reconsidered by the transportation allowance will be converted to a
Voluntary Arbitrator on the basis of a motion for gasoline allowance; the amount of unused gasoline
reconsideration duly filed during that period. These will be converted into cash subject to whatever tax
rulings fully establish that the absence of a may be applicable. Being income tax on
categorical language in Article 262-A does not compensation, it was subject to withholding tax and
preclude the filing of a motion for reconsideration of therefore the company deducted this from the union
the VAs decision within the 10-day period. members’ salaries. The jurisdiction of the Voluntary
Arbitrator is limited to labor disputes which the Labor
97
Code defines as “any controversy or matter pay to him. The second motion was granted. HIDECO
concerning terms and conditions of employment or filed a petition for certiorari in the CA which was
the association or representation of persons in granted in its favor.
negotiating, fixing, maintaining, changing, or
arranging the terms and conditions of employment, The proper remedy was to appeal to the CA by
regardless of whether the disputants stand in the petition for review under Rule 43 of the Rules of Court
proximate relation of employer and employee.” The whose Section 1 specifically provides: Section 1. 
issue on the taxability of the gas allowance and Scope.—This Rule shall apply to appeals from
propriety of the withholding tax are clearly tax judgments or final orders of the, X X X X voluntary
matters and do not in any way involve labor disputes. arbitrators authorized by law. On account of Article
262-A of the Labor Code, the period to appeal was
PHILEC v. CA, December 10, 2014 necessarily 10 days from receipt of the copy of the
Respondents were members of the exclusive award or decision of the Voluntary Arbitrator or panel
bargaining representative of Philec’s RNF EEs. They of Voluntary Arbitrators; otherwise, the order would
were selected for promotion and were required to become final and immutable, because only a timely
undergo training. EEs were to be given an allowance. appeal or motion for reconsideration could prevent
EEs claimed that the training allowance did not the award or decision from attaining finality and
conform to Article X of their CBA. Philec contends that immutability. Also, the Voluntary Arbitrator’s order of
they applied Modified SGV pay grades to avoid salary reinstatement of the petitioner was immediately
distortion. executory. Although the timely filing of a motion for
reconsideration or of an appeal forestalls the finality
The Voluntary Arbitrator did not gravely abuse its of the decision or award of the Voluntary Arbitrator,
discretion. The Voluntary Arbitrator correctly awarded the reinstatement aspect of the Voluntary Arbitrator’s
training allowances based on the amounts and decision or award remains executory regardless of
formula of the CBA. Moreover, there being no appeal the filing of such motion for reconsideration or
seasonably filed in this case, Voluntary Arbitrator’s appeal.
decision became final and executory after 10
calendar days from PHILEC’s receipt of the resolution
denying its motion for partial reconsideration. Since NLRC; Decision; Finality; Rule
the office of a Voluntary Arbitrator or a panel of
Voluntary Arbitrators is considered a quasi-judicial
65 Petition
agency, this court concluded that a decision or award
Jordan v. Grandeur Security, June 18,
rendered by a Voluntary Arbitrator is appealable
before the Court of Appeals. Under Section 9 of the 2014
Judiciary Reorganization Act of 1980, the Court of Jordan filed for illegal dismissal. Company denied the
Appeals has the exclusive original jurisdiction over allegation and claimed that claimed that it reassigned
decisions or awards of quasi-judicial agencies and Jordan from QC to Taguig and the latter abandoned
instrumentalities. Article 262-A of the Labor Code his work. The LA (May 27, 2008) held that Jordan had
provides that the award or decision of the Voluntary merely been transferred to another workplace. Thus,
Arbitrator “shall be final and executory after ten (10) the LA ordered Grandeur Security to "reinstate"
calendar days from receipt of the copy of the award Jordan in employment. However, on Dec 2010, the LA
or decision by the parties.” pronounced the proceedings terminated on the
ground Jordan’s waiver of his right to be reinstated
Baronda v. CA, October 14, 2015 despite receiving a letter from the company. The
NLRC set aside the Dec 2010 order and held that
Baroda was employed as a truck driver and was
Jordan is entitled to backwages and separation pay.
found guilty of negligence and was subsequently
The CA nullified the ruling of the NLRC.
terminated. He filed a case against the VA who then
declared that Baroda was dismissed illegally and
The NLRC gravely abused its discretion in
entitled to reinstatement. Baronda filed a motion for
substantively altering the dispositive part of the May
the issuance of the writ of execution and insisting on
27, 2008 decision. The CA correctly ruled that the
being entitled to backwages and other benefits.
NLRC acted outside of its jurisdiction in replacing the
HIDECO opposed the motion and stated that the VA
LA’s return to work order. The NLRC’s judgments
actually ruled Baroda to be liable to pay actual
ordering Grandeur Security to pay backwages,
damages. The first motion was denied on the ground
separation pay, and attorney’s fees are unwarranted,
that the decision did not award any backwages.
unprecedented, and arbitrary. These, in effect,
Baronda again filed another motion for execution
vacated the May 27, 2008 decision which already
praying that a writ of execution requiring HIDECO to
found the continued existence of Jordan’s
98
employment. To the point of being repetitive, we the court to correct any actual or perceived error
reiterate that backwages and separation pay are attributed to it by the re-examination of the legal and
mere consequences of illegal dismissal. factual circumstances of the case. The rationale of
the rule rests upon the presumption that the court or
Sarona v. NLRC, January 18, 2012 administrative body which issued the assailed order
Pet., hired as a security guard, was asked to resign by or resolution may amend the same, if given the
for applying to another company. During his floating chance to correct its mistake or error. As a rule, the
status, he was assigned twice in two different Court of Appeals, in the exercise of its original
companies. However, his assignment was jurisdiction, will not take cognizance of a petition for
subsequently cancelled which led to the filing of the certiorari under Rule 65, unless the lower court has
complaint. The LA held that he was illegally been given the opportunity to correct the error
dismissed. NLRC affirmed and res. to the CA and paid imputed to it. The Court of Appeals correctly ruled
Sarona the award. Upon reaching the SC, the res. that petitioners failure to file a motion for
claims that petitioner is barred from questioning the reconsideration against the assailed Resolution of the
manner by which his backwages and separation pay NLRC rendered its petition for certiorari before the
were computed. Pet. countered that despite the appellate court as fatally defective.
satisfaction of the award, this does not preclude the
SC from modifying the decision when it is tainted with PHILTRANCO v. PWU-AGLO, February 26,
grave abuse of discretion or issued without 2014
jurisdiction. Pet. retrenched its employees. The union staged a
strike claiming that the company engaged in unfair
The finality of the NLRC’s decision does not preclude labor practices. The case was thereafter referred to
the filing of a petition for certiorari under Rule 65 of the DOLE Secretary who ruled in favor of the union.
the ROC. That the NLRC issues an entry of judgment Pet. filed for a MR while the res. initiated a partial
after the lapse of 10 days from the parties’ receipt of appeal. The SOLE declined to rule on both prayers
its decision will only give rise to the prevailing party’s citing a DOLE regulation which provided that
right to move for the execution thereof, but will not voluntary arbitrators’ decisions, orders, resolutions or
prevent the CA from taking cognizance of a petition awards shall not be the subject of motions for
for certiorari on jurisdictional and due process reconsideration. Petitioner then filed before the CA an
considerations. Furthermore, if the NLRC’s decision or original Petition for Certiorari (R.65). CA dismissed
resolution was reversed and set aside for being and claimed that a petition for review (R.43) is the
issued with grave abuse of discretion by way of a proper remedy.
petition for certiorari to the CA or to the Court by way
of an appeal from the decision of the CA, it is Rule 65 is the proper remedy in assailing the decision
considered void ab initio and, thus, had never of the Secretary of Labor. It has long been settled that
become final and executory. the remedy of an aggrieved party in a decision or
resolution of the Secretary of Labor is to timely file a
Metro Transit Organization v. PIGLAS motion for reconsideration as a precondition for any
NFWU-KMU, April 14, 2008 further or subsequent remedy, and then seasonably
Petitioner entered into an operations agreement with file a special civil action for certiorari under Rule 65 of
the LRTA. A CBA was entered by the pet and the the 1997 Rules on Civil Procedure. The only way by
union. Later, Piglas won the CE and negotiated a new which a labor case may reach the Supreme Court is
CBA with higher benefits. However, due to a through a petition for certiorari under Rule 65 of the
deadlock, they staged a strike and the SOLE assumed Rules of Court alleging lack or excess of jurisdiction or
jurisdiction and rendered a return to work order. The grave abuse of discretion. Such petition may be filed
striking ees refused to heed the order and the LRTA within a reasonable time from receipt of the
informed the pet. that the LRTA would take over the resolution denying the motion for reconsideration of
operations which prompted the company to dismiss the NLRC decision.
the respondents. LA ruled that there was illegal
dismissal and the NLRC dismissed the appeal for
failing to post the required bond. The CA also Appeal
dismissed the petition for Certiorari for the failure to
file a MR. Prince Transport v. Garcia, January 12,
2011
The settled rule is that a motion for reconsideration is The Pres. of the company suspected that a union was
a condition sine qua non for the filing of a petition for about to be formed and made known to Garcia his
certiorari. Its purpose is to grant an opportunity for objection. In order to block the formation of a new
99
union, the respondent ees were transferred to the affirmative relief because it was inconsistent with the
sub-company which was later forced to cease doctrine that a party who has not appealed cannot
operations because of the lack of support from the obtain from the appellate court any affirmative relief
parent company. Company denied that the other than the ones granted in the appealed decision.
respondents were their ees. The LA ruled in favor of
the company and upon reaching the NLRC, it affirmed
the LA and denied the MR filed by the ees which led
to the filing of a R.65 petition. CA then ruled in favor Appeal Bond
of the ees. Pet alleges that the CA should not have
given due course to the petition as they failed to file McBurnie v. Ganzon, EGI-Managers, Inc.,
an appeal to the NLRC. October 17, 2013
LA found that there was illegal dismissal. Company
The Court held that the proper vehicle for such review
filed a notice of appeal with a memorandum
is a special civil action for certiorari under Rule 65 of
containing a motion to reduce bond averring that it
the said Rules, and that the case should be filed with
was encountering difficulty in raising the bond and
the CA in strict observance of the doctrine of
paying for the partial bond. The NLRC granted the
hierarchy of courts. Moreover, it is already settled
motion holding that there was substantial
that under Section 9 of Batas Pambansa Blg. 129, as
compliance. The CA reversed and claimed that the
amended by Republic Act No. 7902, the CA - pursuant
failure to post the required bond in an amount
to the exercise of its original jurisdiction over
equivalent to the monetary judgment impeded the
petitions for certiorari - is specifically given the power
perfection of the appeal and rendered the LA decision
to pass upon the evidence, if and when necessary, to
final and executory. The bond may be reduced upon
resolve factual issues. The CA can grant the petition
motion by the employer, this is subject to the
for certiorari if it finds that the NLRC, in its assailed
conditions: a) the motion to reduce the bond shall be
decision or resolution, made a factual finding not
based on meritorious grounds and b) a reasonable
supported by substantial evidence. It is within the
amount in relation to the monetary award is posted
jurisdiction of the CA, whose jurisdiction over labor
by the appellant, otherwise the filing of the motion to
cases has been expanded to review the findings of
reduce bond shall not stop the running of the period
the NLRC.
to perfect an appeal. The rule that the filing of a
motion to reduce bond shall not stop the running of
Unilever v. Rivera, June 3, 2013 the period to perfect an appeal is not absolute. The
Rivera was employed as the manager of sales Court may relax the rule under exceptional
activities in her area and was also supervising the circumstances.
third party service provider for the company’s
activation projects. The internal auditor of the pet. If the NLRC does eventually grant the motion for
conducted a random audit and found out that there reduction after the reglementary period has elapsed,
were fictitious billings and fabricated receipts from the correct relief would be to reduce the cash or
the third party under the instructions of Rivera. The surety bond already posted by the employer within
company found Rivera guilty of serious breach of the the 10-day period. As the Court remains firm on the
company’s Code of Business Principles compelling it importance of appeal bonds in appeals from
to sever their professional relations but was allowed monetary awards of LAs, we stress that the NLRC,
retirement benefits. LA dismissed the complaint and pursuant to Section 6, Rule VI of the NLRC Rules of
the NLRC affirmed but concluded that she is entitled Procedure, shall only accept motions to reduce bond
to separation pay. The CA deleted the award of that are coupled with the posting of a bond in a
retirement benefits. However, due to social justice, reasonable amount.
she was awarded separation pay. The pet. now
questions the awarding of affirmative reliefs despite Sara Lee v. Macatlang, June 4, 2014 and
the failure of Rivera to file a pet for certiorari to
question the NLRC resolutions. January 14, 2015
Aris Phil. permanently ceased operations which led
Rivera did not appeal the March 31, 2009 ruling of the ees to file a case for illegal dismissal. The LA
the NLRC disallowing the award of separation pay to found the dismissal illegal and awarded monetary
her. It was Unilever who elevated the case to the CA. benefits amounting to 3B. The corp. filed a notice of
It is axiomatic that a party who does not appeal, or appeal to with a motion to reduce the appeal bond.
file a petition for certiorari, is not entitled to any They posted a bond of 4.5 M. The NLRC required
affirmative relief. Due process prevents the grant of them to post an additional 4.5 M. The ees then filed a
additional awards to parties who did not appeal. petition for review before the CA insisting that the
Hence, it was erroneous for the CA to grant an
100
appeal was not perfected due to failure of the memorandum of appeal, the right of the employer to
Corporations to post the correct amount of the bond. appeal must be upheld. This is in recognition of the
importance of the remedy of appeal, which is an
The fact that the amount of 10% of the award is not a essential part of our judicial system and the need to
permissible bond but is only such amount that shall ensure that every party litigant is given the amplest
be deemed reasonable in the meantime that the opportunity for the proper and just disposition of his
appellant’s motion is pending resolution by the cause freed from the constraints of technicalities.
Commission. The actual reasonable amount yet to be
determined is necessarily a bigger amount. In an Hacienda Valentin-Balabag v. Secretary,
effort to strike a balance between the constitutional
obligation of the state to afford protection to labor on
February 11, 2008
the one hand, and the opportunity afforded to the Workers filed with the DOLE Bacolod a request for
employer to appeal on the other, the Court in that payroll inspection of Hacienda Valentin Balabag.
case considered the appeal bond in the amount of DOLE conducted an inspection and issued a notice of
P725M which is equivalent to 25% of the monetary inspection report, finding Yanson liable and directed
award sufficient to perfect the appeal. (2014 case the owner of the Hacienda to pay. Yanson filed with
doctrine) the Sec. of Labor a verified appeal and posted a
bond.
The Corporations should have followed the direction
of the Court and filed the additional amount In case said order by the DOLE involves a monetary
requested by the Courts for the perfection of the award, an appeal by the employer may be perfected
appeal so that the NLRC may proceed to try the only upon the posting of a cash or surety bond issued
merits of the case for illegal dismissal. The 10% by a reputable bonding company duly accredited by
requirement pertains to the reasonable amount which the Secretary of Labor in an amount equivalent to the
the NLRC would accept as the minimum of the bond monetary award in the order appealed from. When
that should accompany the motion to reduce bond in Yanson filed her verified appeal and supplement to
order to suspend the period to perfect an appeal the verified appeal, the Secretary of Labor rejected
under the NLRC rules. The 10% is based on the the appeal for insufficiency of the appeal bond. The
judgment award and should in no case be construed posting of the proper amount of the appeal bond
as the minimum amount of bond to be posted in under Article 128(b) is mandatory (based on the
order to perfect appeal. Should the NLRC, after usage of “only” in relation to requirements) for the
considering the merit of the Motion to Reduce Appeal perfection of an appeal from a monetary award in
Bond, determine that a greater amount or the full labor cases.
amount of the bond needs to be posted by the
appellant, then the party shall comply accordingly.
(2015 case doctrine) Execution; Updating of Award
Nacar v. Gallery Frames, August 13,
Balite v. SS Ventures, Feb. 2015
Balite et. al were all dismissed. There was a case for 2013
illegal dismissal. The Labor Arbiter in favored Balite et Nacar filed a case against the company for being
al., and charged SS Ventures for illegal dismissal for dismissed without cause. LA found the company
failing to comply with the procedural and substantive guilty. It then appealed all the way to the SC but the
requirement in terminating and awarded monetary SC affirmed the ruling of the LA and it became final
awards. The SS Ventures, however, instead of filing on May 27, 2002. After the finality of the SC decision,
the required appeal bond equivalent to the total Nacar filed a motion before the LA for re-computation
amount of the monetary award which is P490,308.00, as he alleged that his backwages should be
respondents filed a Motion to Reduce the Appeal computed from the time of his illegal dismissal until
Bond to P100,000.00. Upon reaching the CA, there the finality of the SC decision with interest. LA denied
was a relaxation of the rule on the appeal bound the motion claiming that it should be computed from
because even if the amount of the appeal bond the time of his illegal dismissal until the decision of
posted was not equivalent to the monetary award, the LA because Nacar did not appeal hence as to him,
the respondents were able to sufficiently prove their that decision became final and executory.
incapability to post the required amount of bond.
There are two parts of a decision when it comes to
With the employer’s demonstrated good faith in filing illegal dismissal cases. The second part is the ruling
the motion to reduce the bond on demonstrable on the award of backwages and/or separation pay. For
grounds coupled with the posting of the appeal bond backwages, it will be computed from the date of
in the requested amount, as well as the filing of the illegal dismissal until the date of the decision of the
101
Labor Arbiter. But if the employer appeals, then the to include the amount of the award as stated in the
end date shall be extended until the day when the LA’s decision and additional award computed until
appellate court’s decision shall become final. Hence, the date the CA denied petitioner’s motion for
as a consequence, the liability of the employer, if he reconsideration which the CA granted. Case was then
loses on appeal, will increase – this is just but a risk remanded to the LA. However, the CA resolution had
that the employer cannot avoid when it continued to confirmed that the amount of P170,520.31 awarded
seek recourse against the Labor Arbiter’s decision. by the LA is sufficient and that there’s no need to
compute for additional monetary awards.
University of Pangasinan v. Fernandez,
By the nature of an illegal dismissal case, the reliefs
November 12, 2014 continue to add up until full satisfaction, as expressed
The res. filed a case for illegal dismissal. Upon under Article 279 of the Labor Code. The
reaching the SC, it ruled that the res. were indeed recomputation of the consequences of illegal
illegally dismissed. The case became final and dismissal upon execution of the decision does not
executory. Subsequently, the res. filed for the re- constitute an alteration or amendment of the final
computation of their award to include backwages decision being implemented. The illegal dismissal
from the date of the decision of the LA until the ruling stands; only the computation of monetary
finality of the decision. The LA denied the claim consequences of this dismissal is affected, and this is
stating that the decision rendered by the CA not a violation of the principle of immutability of final
reinstating the decision of LA Gambito was declared judgments.
final and executory by no less than the Supreme
Court of the Philippines by its issuance of a final entry
of Judgment which amounts to the doctrine of
immutability of judgment.
Miscellaneous
Masmud v. NLRC, February 13, 2009
No essential change is made by a re-computation as
this step is a necessary consequence that flows from (attorney’s fees)
the nature of the illegality of dismissal declared in Masmud filed a complaint against the company for
that decision. A re-computation (or an original non-payment of permanent disability benefits,
computation, if no previous computation has been medical expenses, sickness allowance, moral and
made) is a part of the law—specifically, Article 279 of exemplary damages, and attorney’s fees. Masmud
the Labor Code and the established jurisprudence on agreed to pay to Att. Go, his counsel, attorney’s fees
this provision—that is read into the decision. By the on a contingent basis, as follows: 20% of total
nature of an illegal dismissal case, the reliefs monetary claims and 10% in case of appeal. It was
continue to add on until full satisfaction, as expressed likewise agreed that any award of attorney’s fees
under Article 279 of the Labor Code. The re- shall pertain to respondent’s law firm as
computation of the consequences of illegal dismissal compensation.
upon execution of the decision does not constitute an LA rendered a Decision granting the monetary claims.
alteration or amendment of the final decision being Masmud paid only the amount equivalent to 20% of
implemented. The illegal dismissal ruling stands; only the award as attorney’s fees, thus, leaving a balance
the computation of monetary consequences of this of 10%, plus the award pertaining to the counsel as
dismissal is affected and this is not a violation of the attorney’s fees. Masmud manifested that Atty. Go’s
principle of immutability of final judgments. . Article claim for attorney’s fees of 40% of the total monetary
279 provides for the consequences of illegal dismissal award was null and void based on Article 111 of the
in no uncertain terms, qualified only by jurisprudence Labor Code which provides 10% of the amount of the
in its interpretation of when separation pay in lieu of wages recovered.
reinstatement is allowed. When that happens, the
finality of the illegal dismissal decision becomes the There are two concepts of Attorney’s Fees. In the
reckoning point instead of the reinstatement that the ordinary sense, attorney’s fees represent the
law decrees reasonable compensation paid to a lawyer by his
client for the legal services rendered to the latter,
while in extraordinary sense, attorney’s fees may be
Metroguards Security v. Hilongo, March
awarded by the court as indemnity for damages to be
9, 2015 paid by the losing party to the prevailing party, and
LA entitled res. to backwages and separation pay for the amount is payable not to the lawyer but to the
being illegally dismissed. On appeal, NLRC reversed client, unless they have agreed that the award shall
the ruling. CA reinstated LA decision. MR denied. pertain to the lawyer as additional compensation or
Petitioner no longer appealed to the SC. Res. filed a as part thereof.
motion asking that the CA’s resolution be interpreted
102
the date of their dismissal. SC held that the
Article 111 of the Labor Code deals with the prescriptive period continues even after the
extraordinary concept of attorney’s fees—it regulates withdrawal of the case as though no action has been
the amount recoverable as attorney’s fees in the filed at all. Thus, it has prescribed. While the filing of
nature of damages sustained by and awarded to the the said case could have interrupted the running of
prevailing party and may not be used as the standard the four-year prescriptive period, the voluntary
in fixing the amount payable to the lawyer by his withdrawal of the petitioners effectively cancelled the
client for the legal services he rendered. tolling of the prescriptive period within which to file
their illegal dismissal case, leaving them in exactly
In this case, we apply the ordinary concept of the same position as though no labor case had been
attorney’s fees, or the compensation that Atty. Go is filed at all. The running of the four-year prescriptive
entitled to receive for representing Evangelina, in period not having been interrupted by the filing of
substitution of her husband, before the labor tribunals NLRC RAB-I-01-1007, the petitioners’ cause of action
and before the court. Contrary to Masmud’s had already prescribed in four years after their
proposition, Article 111 of the Labor Code deals with cessation of employment on Oct. 26, 1997 and Nov.
the extraordinary concept of attorney’s fees. It 24, 1997. Consequently, when the petitioners filed
regulates the amount recoverable as attorney’s fees their complaint for illegal dismissal, separation pay,
in the nature of damages sustained by and awarded retirement benefits, and damages in 2002, their
to the prevailing party. It may not be used as the claim, clearly, had already been barred by
standard in fixing the amount payable to the lawyer prescription
by his client for the legal services he rendered.
Phil. Carpet Manufacturing Corp. v.
Arriola v. Pilipino Star Ngayon, August Tagyamon, December 11, 2013
13, 2014 (prescription) (quitclaim)
George Arriola was a column writer for the newspaper PCMC's employees were affected by the company's
Pilipino Star Ngayon and he filed a case for illegal retrenchment and voluntary retirement program
dismissal averring that he was arbitrarily dismissed which was effected because of less demand in the
when his column was removed from publication by market of the products. The respondents received a
Pilipino Star. The newspaper company claimed that memorandum of dismissal stating that they were
Arriola abandoned his work because he went to work going to be retrenched. PCMC contends that
for a rival newspaper. Article 291 of the Labor Code retrenchment was a necessary management
does not cover “money claims” consequent to an prerogative, but the respondents claim that they were
illegal dismissal such as backwages. It also does not illegally dismissed and that their signing of a
cover claims for damages due to illegal dismissal. quitclaim is not a bar to the pursuit of an illegal
These claims are governed by Article 1146 of the Civil dismissal case. As the ground for termination of
Code which should be filed in 4 years. Further, in an employment was illegal, the quitclaims are deemed
illegal dismissal case, the claim for backwages, the illegal as the employees’ consent had been vitiated
money claim, is just but one of the reliefs that an by mistake or fraud. The law looks with disfavor upon
employee prays before the arbiter. As such, Arriola’s quitclaims and releases by employees pressured into
claim for backwages is still filed within the signing by unscrupulous employers minded to evade
prescriptive period of four years. However, Arriola’s legal responsibilities. The circumstances show that
case must still be dismissed because it was petitioner’s misrepresentation led its employees,
established that he in fact abandoned his work. The specifically respondents herein, to believe that the
removal of a certain column does not ipso facto mean company was suffering losses which necessitated the
the removal of the columnist. implementation of the voluntary retirement and
retrenchment programs, and eventually the execution
Montero v. Times Transport, March 16, of the deeds of release, waiver and quitclaim.
2015 (prescription)
Petitioners were dismissed from service on Oct. 26, Economic necessity constrained respondents to
1997 and Nove. 24, 1997. Some of the petitioners accept petitioners’ monetary offer and sign the deeds
filed several complaints on May 14, 1998 but the case of release, waiver and quitclaim. That respondents
was withdrawn on March 4, 1999. 4 years later, are supervisors and not rank-and- file employees
several complaints for ULP, illegal dismissal were filed does not make them less susceptible to financial
against TTCI on June 2002. In defense, TTCI asserted offers, faced as they were with the prospect of
that the petitioners’ cause of action had already been unemployment. The Court has allowed supervisory
barred by prescription because the complaints were employees to seek payment of benefits and a
filed only on June 2002 or after almost five years from manager to sue for illegal dismissal even though, for
103
a consideration, they executed deeds of quitclaims
releasing their employers from liability.

Am-Phil Food Concepts v. Padilla,


October 1, 2014 (quitclaim)
Padilla was informed by Am-Phil of a retrenchment
program because of serious adverse business
conditions that would affect him and 2 other
employees. Am-Phil gave him 2 options: be
retrenched with severance pay; or be transferred as a
waiter (a demotion). Am-Phil eventually sent him a
memorandum notifying him of his retrenchment and
was paid separation pay. Padilla also executed a
quitclaim and release in favor of Am-Phil. According
to the SC, Padilla was illegally dismissed despite the
quitclaim. Accordingly, there is no credible
explanation offered as to why the financial
statements were not presented when the evidence in
chief was being considered by the labor arbiter. It
follows that there is no clear and convincing evidence
to sustain the substantive ground on which the
supposed validity of Padilla’s retrenchment rests. As a
rule, deeds of release or quitclaim cannot bar
employees from demanding benefits to which they
are legally entitled or from contesting the legality of
their dismissal.

104

Das könnte Ihnen auch gefallen